SóProvas



Prova FUMARC - 2018 - SEE-MG - Professor de Educação Básica - Língua Portuguesa


ID
2688733
Banca
FUMARC
Órgão
SEE-MG
Ano
2018
Provas
Disciplina
Português
Assuntos

                                     Terrorismo lógico

                                                                                           Antônio Prata


      Said e Chérif Kouachi eram descendentes de imigrantes. Said e Chérif Kouachi são suspeitos do ataque ao jornal "Charlie Hebdo", na França. Se não houvesse imigrantes na França, não teria havido ataque ao "Charlie Hebdo".

      Said e Chérif Kouachi, suspeitos do ataque ao jornal "Charlie Hebdo", eram filhos de argelinos. Zinedine Zidane é filho de argelinos. Zinedine Zidane é terrorista.

      Zinedine Zidane é filho de argelinos. Said e Chérif Kouachi, suspeitos do ataque ao jornal "Charlie Hebdo", eram filhos de argelinos. Said e Chérif Kouachi sabiam jogar futebol.

      Muçulmanos são uma minoria na França. Membros de uma minoria são suspeitos do ataque terrorista. Olha aí no que dá defender minoria...

      A esquerda francesa defende minorias. Membros de uma minoria são suspeitos pelo ataque terrorista. A esquerda francesa é culpada pelo ataque terrorista.

      A extrema direita francesa demoniza os imigrantes. O ataque terrorista fortalece a extrema direita francesa. A extrema direita francesa está por trás do ataque terrorista.

      Marine Le Pen é a líder da extrema direita francesa. "Le Pen" é "O Caneta", se tomarmos o artigo em francês e o substantivo em inglês. Eis aí uma demonstração de apoio da extrema direita francesa à liberdade de expressão – e aos erros de concordância nominal.

      (Este último parágrafo não fez muito sentido. Os filmes do David Lynch não fazem muito sentido. Este último parágrafo é um filme do David Lynch.)

      O "Charlie Hebdo" zoava Maomé. Eu zoo negão, zoo as bichinhas, zoo gorda, zoo geral! "Je suis Charlie!"

      Humoristas brasileiros fazem piada racista, e as pessoas os criticam. "Charlie Hebdo" fez piada com religião, e terroristas o atacam. Criticar piada racista é terrorismo. 

      Numa democracia, é desejável que as pessoas sejam livres para se expressar. Algumas dessas expressões podem ofender indivíduos ou grupos. Numa democracia, é desejável que indivíduos ou grupos sejam ofendidos.

      O "Charlie Hebdo" foi atacado por terroristas. A editora Abril foi pichada por meia dúzia de jacus. A editora Abril é Charlie.

      Os terroristas que atacaram o jornal "Charlie Hebdo" usavam gorros pretos. "Black blocs" usam gorros pretos. "Black blocs" são terroristas.

      "Black blocs" não são terroristas. A polícia os trata como terroristas. Os "black blocs" têm o direito de tocar o terror.

      Os terroristas que atacaram o jornal "Charlie Hebdo" usavam gorros pretos. Drones não usam gorros pretos. Ataques com drones não são terrorismo.

      Ataques com drones matam inocentes mundo afora. O "Ocidente" usa drones. É justificável o terror contra o "Ocidente".

      O ataque terrorista contra o "Charlie Hebdo" foi no dia 7/1. A derrota brasileira para a Alemanha foi por 7 x 1. O 7 e o 1 devem ser imediatamente presos e submetidos a "técnicas reforçadas de interrogatório", tais como simulação de afogamento, privação de sono e alimentação via retal. Por via das dúvidas, o 6 e o 8 e o 0 e o 2 também.

      Todo abacate é verde. O Incrível Hulk é verde. O Incrível Hulk é um abacate. 

(Disponível em: http://www1.folha.uol.com.br/colunas/antonioprata/2015/01/1573334-terrorismo-logico.shtml. Acesso em: 2 fev. 2015.)

Leia as considerações abaixo, sobre o texto.


I. Os recursos de construção recorrentemente adotados em cada parágrafo do texto atuam diretamente na construção da ironia.

II. O texto toma como objeto central de reflexão os ataques terroristas na França.

III. Em alguns parágrafos do texto, revela-se, de forma explícita, a defesa do autor ao combate ao terror do Ocidente.

IV. Subjaz ao texto uma crítica à fragilidade das generalizações e conclusões apressadas ou inconsistentes.


Está CORRETO apenas o que se afirma em:

Alternativas
Comentários
  • isso é que eu chamo de um texto CHATO!!!

  • Gabarito letra B para os não assinantes.

     

    I. Os recursos de construção recorrentemente adotados em cada parágrafo do texto atuam diretamente na construção da ironia.

    Ironia: Ação de dizer o oposto do que se quer expressar: você foi super bem na prova, por isso, não passou nessa matéria.Sarcasmo; comentário insultuoso, mordaz; zombaria que ofende.

    Exemplo: Eu zoo negão, zoo as bichinhas, zoo gorda, zoo geral! "Je suis Charlie!",

     

    IV. Subjaz ao texto uma crítica à fragilidade das generalizações e conclusões apressadas ou inconsistentes.

    Subjaz: Ficar subjacente, implícito, não manifesto; dissimular. 

    Subjazer é sinônimo de: submeter, subordinar, dissimular, esconder, subjazem

    Exemplo: Todo abacate é verde. O Incrível Hulk é verde. O Incrível Hulk é um abacate. (uma generalização, uma conclusão precipitada);

    Os terroristas que atacaram o jornal "Charlie Hebdo" usavam gorros pretos. "Black blocs" usam gorros pretos. "Black blocs" são terroristas.

     

     

  • Por que a II está incorreta?

  • O erro da assertiva II é afirmar que o texto toma como objeto central de reflexão os ataques terroristas na França. Na verdade, o tema central do texto é terrorismo.

    O erro da assertiva III é afirmar que o autor defende o combate ao terrorismo pelo Ocidente, sendo que o termo "Ocidente" só aparece no parágrafo "Ataques com drones matam inocentes mundo afora. O "Ocidente" usa drones. É justificável o terror contra o "Ocidente." e o excerto deixa explícito que é justificável o terror contra o "Ocidente".

    Abraços!

  • Entendo que a II está errada porque o objeto central do texto é o ataque ao jornal Charlie Hebdo.

  • Fiz essa prova. E quando comecei a ler o texto, pensei: tô ferrada! rsrsrs Resultado: a prova de português me reprovou! =/


ID
2688736
Banca
FUMARC
Órgão
SEE-MG
Ano
2018
Provas
Disciplina
Português
Assuntos

                                     Terrorismo lógico

                                                                                           Antônio Prata


      Said e Chérif Kouachi eram descendentes de imigrantes. Said e Chérif Kouachi são suspeitos do ataque ao jornal "Charlie Hebdo", na França. Se não houvesse imigrantes na França, não teria havido ataque ao "Charlie Hebdo".

      Said e Chérif Kouachi, suspeitos do ataque ao jornal "Charlie Hebdo", eram filhos de argelinos. Zinedine Zidane é filho de argelinos. Zinedine Zidane é terrorista.

      Zinedine Zidane é filho de argelinos. Said e Chérif Kouachi, suspeitos do ataque ao jornal "Charlie Hebdo", eram filhos de argelinos. Said e Chérif Kouachi sabiam jogar futebol.

      Muçulmanos são uma minoria na França. Membros de uma minoria são suspeitos do ataque terrorista. Olha aí no que dá defender minoria...

      A esquerda francesa defende minorias. Membros de uma minoria são suspeitos pelo ataque terrorista. A esquerda francesa é culpada pelo ataque terrorista.

      A extrema direita francesa demoniza os imigrantes. O ataque terrorista fortalece a extrema direita francesa. A extrema direita francesa está por trás do ataque terrorista.

      Marine Le Pen é a líder da extrema direita francesa. "Le Pen" é "O Caneta", se tomarmos o artigo em francês e o substantivo em inglês. Eis aí uma demonstração de apoio da extrema direita francesa à liberdade de expressão – e aos erros de concordância nominal.

      (Este último parágrafo não fez muito sentido. Os filmes do David Lynch não fazem muito sentido. Este último parágrafo é um filme do David Lynch.)

      O "Charlie Hebdo" zoava Maomé. Eu zoo negão, zoo as bichinhas, zoo gorda, zoo geral! "Je suis Charlie!"

      Humoristas brasileiros fazem piada racista, e as pessoas os criticam. "Charlie Hebdo" fez piada com religião, e terroristas o atacam. Criticar piada racista é terrorismo. 

      Numa democracia, é desejável que as pessoas sejam livres para se expressar. Algumas dessas expressões podem ofender indivíduos ou grupos. Numa democracia, é desejável que indivíduos ou grupos sejam ofendidos.

      O "Charlie Hebdo" foi atacado por terroristas. A editora Abril foi pichada por meia dúzia de jacus. A editora Abril é Charlie.

      Os terroristas que atacaram o jornal "Charlie Hebdo" usavam gorros pretos. "Black blocs" usam gorros pretos. "Black blocs" são terroristas.

      "Black blocs" não são terroristas. A polícia os trata como terroristas. Os "black blocs" têm o direito de tocar o terror.

      Os terroristas que atacaram o jornal "Charlie Hebdo" usavam gorros pretos. Drones não usam gorros pretos. Ataques com drones não são terrorismo.

      Ataques com drones matam inocentes mundo afora. O "Ocidente" usa drones. É justificável o terror contra o "Ocidente".

      O ataque terrorista contra o "Charlie Hebdo" foi no dia 7/1. A derrota brasileira para a Alemanha foi por 7 x 1. O 7 e o 1 devem ser imediatamente presos e submetidos a "técnicas reforçadas de interrogatório", tais como simulação de afogamento, privação de sono e alimentação via retal. Por via das dúvidas, o 6 e o 8 e o 0 e o 2 também.

      Todo abacate é verde. O Incrível Hulk é verde. O Incrível Hulk é um abacate. 

(Disponível em: http://www1.folha.uol.com.br/colunas/antonioprata/2015/01/1573334-terrorismo-logico.shtml. Acesso em: 2 fev. 2015.)

Do ponto de vista do gênero, o texto é:

Alternativas
Comentários
  • Crônica é um tipo de texto narrativo curto, geralmente produzido para meios de comunicação, por exemplo, jornais, revistas, etc.

  • GABARITO - LETRA C

    Conto: É uma narrativa curta. O tempo em que se passa é reduzido e contém poucas personagens que existem em função de um núcleo. É o relato de uma situação que pode acontecer na vida das personagens, porém não é comum que ocorra com todo mundo. Pode ter um caráter real ou fantástico da mesma forma que o tempo pode ser cronológico ou psicológico.

    Crônica: Por vezes é confundida com o conto. A diferença básica entre os dois é que a crônica narra fatos do dia a dia, relata o cotidiano das pessoas, situações que presenciamos e já até prevemos o desenrolar dos fatos. A crônica também se utiliza da ironia e às vezes até do sarcasmo. Não necessariamente precisa se passar em um intervalo de tempo, quando o tempo é utilizado, é um tempo curto, de minutos ou horas, normalmente.

  • TEXTO GENIAL!

  • GABARITO C

    Características da crônica:

     ▪Estão presentes em jornais, revistas, livros e sites;

    ▪ Narração curta;

    ▪ Descreve fatos da vida cotidiana;

    ▪ Pode ter caráter humorístico, crítico, satírico e/ou irônico;

    ▪ Possui personagens comuns;

    ▪ Segue um tempo cronológico determinado;

    ▪ Uso da oralidade na escrita e do coloquialismo na fala das personagens;

    ▪ Linguagem simples.

  • CONTO - Narração breve, oral ou escrita, de um sucesso imaginário. Tem um número reduzido de personagens que participam de uma só ação, em um foco temático. Sua finalidade é provocar no leitor uma única resposta emocional. Originariamente, o conto é uma das formas mais antigas de literatura popular de transmissão oral

    CRÔNICA - Atualmente, é um gênero literário que explora qualquer assunto, principalmente os temas do cotidiano. Geralmente escritas para serem publicadas em jornais e revistas — que, mais tarde, podem ou não ser reunidas em livro — a crônica se caracteriza pelo tom humorístico ou crítico.

     

    Crônica:                                                                   Conto:

    Visão pessoal dos fatos                                          Narrativa centrada no essencial

    Apresenta fundo humorístico e crítico                    Elimina análises minunciosas

    Personaggens sem grandes descrições                Poucos personagens, mas bem construídos

    Padrão culto                                                           Padrão culto

    Narrativa curta e leve                                             Gira em torno de um único conflito, curto espaço de tempo e poucos fatos/lugares

     

    Fonte: 

    http://www.diaadiaeducacao.pr.gov.br/portals/roteiropedagogico/publicacao/4084_CONTO_OU_CRONICA.pdf

    https://rainhadaepifania.blogspot.com/2014/06/cronica-e-conto-qual-diferenca.html

     

  • . Conto: narrativa curta e concisa que se desenvolve em torno de um único núcleo. Pode ter características reais

    ou ser apenas fruto da imaginação do autor.


    Crônica: é o relato de acontecimentos do quotidiano, do nosso dia a dia, que se desenrolam em uma sucessão

    cronológica. Reflete a vida social, a política, os costumes de uma época



  • a crônica trabalha com temas do cotidiano: jornalístico, futebolístico...

    "o tom da crônica é predominantemente impressionista e as ideias se encaixam menos por nexos lógicos que imaginativos". (Teotônio Marque)


ID
2688739
Banca
FUMARC
Órgão
SEE-MG
Ano
2018
Provas
Disciplina
Português
Assuntos

                                     Terrorismo lógico

                                                                                           Antônio Prata


      Said e Chérif Kouachi eram descendentes de imigrantes. Said e Chérif Kouachi são suspeitos do ataque ao jornal "Charlie Hebdo", na França. Se não houvesse imigrantes na França, não teria havido ataque ao "Charlie Hebdo".

      Said e Chérif Kouachi, suspeitos do ataque ao jornal "Charlie Hebdo", eram filhos de argelinos. Zinedine Zidane é filho de argelinos. Zinedine Zidane é terrorista.

      Zinedine Zidane é filho de argelinos. Said e Chérif Kouachi, suspeitos do ataque ao jornal "Charlie Hebdo", eram filhos de argelinos. Said e Chérif Kouachi sabiam jogar futebol.

      Muçulmanos são uma minoria na França. Membros de uma minoria são suspeitos do ataque terrorista. Olha aí no que dá defender minoria...

      A esquerda francesa defende minorias. Membros de uma minoria são suspeitos pelo ataque terrorista. A esquerda francesa é culpada pelo ataque terrorista.

      A extrema direita francesa demoniza os imigrantes. O ataque terrorista fortalece a extrema direita francesa. A extrema direita francesa está por trás do ataque terrorista.

      Marine Le Pen é a líder da extrema direita francesa. "Le Pen" é "O Caneta", se tomarmos o artigo em francês e o substantivo em inglês. Eis aí uma demonstração de apoio da extrema direita francesa à liberdade de expressão – e aos erros de concordância nominal.

      (Este último parágrafo não fez muito sentido. Os filmes do David Lynch não fazem muito sentido. Este último parágrafo é um filme do David Lynch.)

      O "Charlie Hebdo" zoava Maomé. Eu zoo negão, zoo as bichinhas, zoo gorda, zoo geral! "Je suis Charlie!"

      Humoristas brasileiros fazem piada racista, e as pessoas os criticam. "Charlie Hebdo" fez piada com religião, e terroristas o atacam. Criticar piada racista é terrorismo. 

      Numa democracia, é desejável que as pessoas sejam livres para se expressar. Algumas dessas expressões podem ofender indivíduos ou grupos. Numa democracia, é desejável que indivíduos ou grupos sejam ofendidos.

      O "Charlie Hebdo" foi atacado por terroristas. A editora Abril foi pichada por meia dúzia de jacus. A editora Abril é Charlie.

      Os terroristas que atacaram o jornal "Charlie Hebdo" usavam gorros pretos. "Black blocs" usam gorros pretos. "Black blocs" são terroristas.

      "Black blocs" não são terroristas. A polícia os trata como terroristas. Os "black blocs" têm o direito de tocar o terror.

      Os terroristas que atacaram o jornal "Charlie Hebdo" usavam gorros pretos. Drones não usam gorros pretos. Ataques com drones não são terrorismo.

      Ataques com drones matam inocentes mundo afora. O "Ocidente" usa drones. É justificável o terror contra o "Ocidente".

      O ataque terrorista contra o "Charlie Hebdo" foi no dia 7/1. A derrota brasileira para a Alemanha foi por 7 x 1. O 7 e o 1 devem ser imediatamente presos e submetidos a "técnicas reforçadas de interrogatório", tais como simulação de afogamento, privação de sono e alimentação via retal. Por via das dúvidas, o 6 e o 8 e o 0 e o 2 também.

      Todo abacate é verde. O Incrível Hulk é verde. O Incrível Hulk é um abacate. 

(Disponível em: http://www1.folha.uol.com.br/colunas/antonioprata/2015/01/1573334-terrorismo-logico.shtml. Acesso em: 2 fev. 2015.)

Em “Este último parágrafo não fez muito sentido”, o enunciador remete o leitor

Alternativas
Comentários
  • Mais alguém achou dificil a questão ou apenas eu?

     

  • marcos vinicius, você voltou ao texto?!

    Este último parágrafo, refere-se:       "Marine Le Pen é a líder da extrema direita francesa. Le Pen" é "O Caneta", se tomarmos o artigo em francês e o substantivo em inglês. Eis aí uma demonstração de apoio da extrema direita francesa à liberdade de expressão – e aos erros de concordância nominal.' Entendi que o autor concluiu que o parágrafo não fez sentido, devido à expressão, Le Pen, que traduzindo, é O canetA, havendo erros de concordância nominal. Por isso a letra E, q fala das conclusões sobre o sobrenome de Le Pen.

    Bom, acho que é isso!

         

  • bob marley iria responder essa questão !! tranquilamente

  • Não vejo a hora de acabar com esta prova de português da fumarc. Estou ficando muito frustado kkkkkkkkkkkkkkkkkk.......

    Quarenta questões o loco meu.

  • KKKKK GENTE DO CÊU, QUE BANCA!

    A QUESTÃO PELO QUE ENTENDI É O SEGUINTE : MARINE LE PEN --- HOUVE NO MEIO DO TEXTO UMA COMPARAÇÃO DO NOME DA LÍDER DE EXTREMA DIREITA FRANCESA COM SUA ESCRITA EM INGLES QUE SERIA O CANETA, FAZENDO PORTANTO UMA REFERENCIA A LIBERDADE DE EXPRESSÃO ( TENDO EM VISTA QUE ELA É CONSERVADORA), SENDO TOTALMENTE CONTROVERSO .

    SERIAMENTE SE A FUMARC CONTINUAR COLOCANDO TEXTOS COMPLEXOS E ALTERNATIVAS MAIS COMPLEXAS AINDA, ESTAMOS FUDIDOS.


ID
2688742
Banca
FUMARC
Órgão
SEE-MG
Ano
2018
Provas
Disciplina
Português
Assuntos

                                     Terrorismo lógico

                                                                                           Antônio Prata


      Said e Chérif Kouachi eram descendentes de imigrantes. Said e Chérif Kouachi são suspeitos do ataque ao jornal "Charlie Hebdo", na França. Se não houvesse imigrantes na França, não teria havido ataque ao "Charlie Hebdo".

      Said e Chérif Kouachi, suspeitos do ataque ao jornal "Charlie Hebdo", eram filhos de argelinos. Zinedine Zidane é filho de argelinos. Zinedine Zidane é terrorista.

      Zinedine Zidane é filho de argelinos. Said e Chérif Kouachi, suspeitos do ataque ao jornal "Charlie Hebdo", eram filhos de argelinos. Said e Chérif Kouachi sabiam jogar futebol.

      Muçulmanos são uma minoria na França. Membros de uma minoria são suspeitos do ataque terrorista. Olha aí no que dá defender minoria...

      A esquerda francesa defende minorias. Membros de uma minoria são suspeitos pelo ataque terrorista. A esquerda francesa é culpada pelo ataque terrorista.

      A extrema direita francesa demoniza os imigrantes. O ataque terrorista fortalece a extrema direita francesa. A extrema direita francesa está por trás do ataque terrorista.

      Marine Le Pen é a líder da extrema direita francesa. "Le Pen" é "O Caneta", se tomarmos o artigo em francês e o substantivo em inglês. Eis aí uma demonstração de apoio da extrema direita francesa à liberdade de expressão – e aos erros de concordância nominal.

      (Este último parágrafo não fez muito sentido. Os filmes do David Lynch não fazem muito sentido. Este último parágrafo é um filme do David Lynch.)

      O "Charlie Hebdo" zoava Maomé. Eu zoo negão, zoo as bichinhas, zoo gorda, zoo geral! "Je suis Charlie!"

      Humoristas brasileiros fazem piada racista, e as pessoas os criticam. "Charlie Hebdo" fez piada com religião, e terroristas o atacam. Criticar piada racista é terrorismo. 

      Numa democracia, é desejável que as pessoas sejam livres para se expressar. Algumas dessas expressões podem ofender indivíduos ou grupos. Numa democracia, é desejável que indivíduos ou grupos sejam ofendidos.

      O "Charlie Hebdo" foi atacado por terroristas. A editora Abril foi pichada por meia dúzia de jacus. A editora Abril é Charlie.

      Os terroristas que atacaram o jornal "Charlie Hebdo" usavam gorros pretos. "Black blocs" usam gorros pretos. "Black blocs" são terroristas.

      "Black blocs" não são terroristas. A polícia os trata como terroristas. Os "black blocs" têm o direito de tocar o terror.

      Os terroristas que atacaram o jornal "Charlie Hebdo" usavam gorros pretos. Drones não usam gorros pretos. Ataques com drones não são terrorismo.

      Ataques com drones matam inocentes mundo afora. O "Ocidente" usa drones. É justificável o terror contra o "Ocidente".

      O ataque terrorista contra o "Charlie Hebdo" foi no dia 7/1. A derrota brasileira para a Alemanha foi por 7 x 1. O 7 e o 1 devem ser imediatamente presos e submetidos a "técnicas reforçadas de interrogatório", tais como simulação de afogamento, privação de sono e alimentação via retal. Por via das dúvidas, o 6 e o 8 e o 0 e o 2 também.

      Todo abacate é verde. O Incrível Hulk é verde. O Incrível Hulk é um abacate. 

(Disponível em: http://www1.folha.uol.com.br/colunas/antonioprata/2015/01/1573334-terrorismo-logico.shtml. Acesso em: 2 fev. 2015.)

Todas as alternativas abaixo trazem reformulações de trechos do texto sem comprometimento da orientação de sentido original, EXCETO em:

Alternativas
Comentários
  • Alguém me explica essa?

  • Gabarito letra C. 

     

    Ajudaria imensamente se a Fumar enumerasse as linhas. 

     

    Daniela, o texto reescrito pela banca muda de sentido, pois ela usa uma conjunção concessiva (ainda que) no lugar de uma que caberia uma adversativa (mas, porém, contudo...) Perceba:

     

    Numa democracia, é desejável que as pessoas sejam livres para se expressar. (PORÉM,/CONTUDO, MAS) Algumas dessas expressões podem ofender indivíduos ou grupos. 

     

    Numa democracia, é desejável que as pessoas sejam livres para se expressar, ainda que algumas dessas expressões podem ofender indivíduos ou grupos. 

  • Danielle, a frase do texto: "Numa democracia, é desejável que indivíduos ou grupos sejam ofendidos." , não indica que caberia uma adversativa com relação ao período anterior ... ainda resta dúvida sobre essa questão

  • Mais uma questão bem subjetiva da Fumarc

  • prefiro uma questão de gramatica pura ..que essa maconha...

  • As conjunções adversativas e concessivas são usadas com o mesmo propósito: ligar enunciados com orientação argumentativa contrária... não acredito q seja essa explicação dos colegas....sigo tentando entender a mentalidade da banca !!

  • Tive que jogar no word e comparar as questões. Muito estranhas essas questões.

  • Bem subjetiva mesmo !

    As letras A e B também não mudam o sentido, na minha opinião.

  • A reescrita na alternativa (C) não mantém o sentido original, pois se entende do texto que, numa democracia, é desejável que as pessoas sejam livres para se expressar. Porém, algumas dessas expressões podem ofender indivíduos ou grupos. Assim, o segundo segmento contrasta com o anterior e não o abona. Na substituição por uma oração subordinada adverbial concessiva, passa-se a uma noção de aceitabilidade, isto é, é desejável que as pessoas sejam livres para se expressar, mesmo que algumas dessas expressões podem ofender indivíduos ou grupos. Assim, há uma mudança de sentido.

    Compare:

    Numa democracia, é desejável que as pessoas sejam livres para se expressar. Algumas dessas expressões podem ofender indivíduos ou grupos.

    Numa democracia, é desejável que as pessoas sejam livres para se expressar, ainda que algumas dessas expressões possam ofender indivíduos ou grupos.

    https://www.estrategiaconcursos.com.br/blog/comentario-da-prova-de-portugues-see-mg-2018/

  • No trecho do texto, referente à letra C, podemos observar que há uma ideia de adversação entre as orações (Oração coordenadas sindética adversativa). Na reescrita da frase há a conjunção "ainda que", que expressa sentido de concessão (Oração subordinada adverbial de concessão).

    Numa democracia, é desejável que as pessoas sejam livres para se expressar. Algumas dessas expressões podem ofender indivíduos ou grupos.

    Numa democracia, é desejável que as pessoas sejam livres para se expressar, ainda que algumas dessas expressões possam ofender indivíduos ou grupos.

  • Cara na moral vou rodar na PC/MG. Maconha do cãooooo.


ID
2688745
Banca
FUMARC
Órgão
SEE-MG
Ano
2018
Provas
Disciplina
Português
Assuntos

                                     Terrorismo lógico

                                                                                           Antônio Prata


      Said e Chérif Kouachi eram descendentes de imigrantes. Said e Chérif Kouachi são suspeitos do ataque ao jornal "Charlie Hebdo", na França. Se não houvesse imigrantes na França, não teria havido ataque ao "Charlie Hebdo".

      Said e Chérif Kouachi, suspeitos do ataque ao jornal "Charlie Hebdo", eram filhos de argelinos. Zinedine Zidane é filho de argelinos. Zinedine Zidane é terrorista.

      Zinedine Zidane é filho de argelinos. Said e Chérif Kouachi, suspeitos do ataque ao jornal "Charlie Hebdo", eram filhos de argelinos. Said e Chérif Kouachi sabiam jogar futebol.

      Muçulmanos são uma minoria na França. Membros de uma minoria são suspeitos do ataque terrorista. Olha aí no que dá defender minoria...

      A esquerda francesa defende minorias. Membros de uma minoria são suspeitos pelo ataque terrorista. A esquerda francesa é culpada pelo ataque terrorista.

      A extrema direita francesa demoniza os imigrantes. O ataque terrorista fortalece a extrema direita francesa. A extrema direita francesa está por trás do ataque terrorista.

      Marine Le Pen é a líder da extrema direita francesa. "Le Pen" é "O Caneta", se tomarmos o artigo em francês e o substantivo em inglês. Eis aí uma demonstração de apoio da extrema direita francesa à liberdade de expressão – e aos erros de concordância nominal.

      (Este último parágrafo não fez muito sentido. Os filmes do David Lynch não fazem muito sentido. Este último parágrafo é um filme do David Lynch.)

      O "Charlie Hebdo" zoava Maomé. Eu zoo negão, zoo as bichinhas, zoo gorda, zoo geral! "Je suis Charlie!"

      Humoristas brasileiros fazem piada racista, e as pessoas os criticam. "Charlie Hebdo" fez piada com religião, e terroristas o atacam. Criticar piada racista é terrorismo. 

      Numa democracia, é desejável que as pessoas sejam livres para se expressar. Algumas dessas expressões podem ofender indivíduos ou grupos. Numa democracia, é desejável que indivíduos ou grupos sejam ofendidos.

      O "Charlie Hebdo" foi atacado por terroristas. A editora Abril foi pichada por meia dúzia de jacus. A editora Abril é Charlie.

      Os terroristas que atacaram o jornal "Charlie Hebdo" usavam gorros pretos. "Black blocs" usam gorros pretos. "Black blocs" são terroristas.

      "Black blocs" não são terroristas. A polícia os trata como terroristas. Os "black blocs" têm o direito de tocar o terror.

      Os terroristas que atacaram o jornal "Charlie Hebdo" usavam gorros pretos. Drones não usam gorros pretos. Ataques com drones não são terrorismo.

      Ataques com drones matam inocentes mundo afora. O "Ocidente" usa drones. É justificável o terror contra o "Ocidente".

      O ataque terrorista contra o "Charlie Hebdo" foi no dia 7/1. A derrota brasileira para a Alemanha foi por 7 x 1. O 7 e o 1 devem ser imediatamente presos e submetidos a "técnicas reforçadas de interrogatório", tais como simulação de afogamento, privação de sono e alimentação via retal. Por via das dúvidas, o 6 e o 8 e o 0 e o 2 também.

      Todo abacate é verde. O Incrível Hulk é verde. O Incrível Hulk é um abacate. 

(Disponível em: http://www1.folha.uol.com.br/colunas/antonioprata/2015/01/1573334-terrorismo-logico.shtml. Acesso em: 2 fev. 2015.)

Todas as alternativas a seguir apresentam sugestões de reformulação do texto, entre parênteses, em consonância com a norma padrão do português, EXCETO:

Alternativas
Comentários
  • Gabarito E.

     

    A Alternativa errada é a (E), pois o verbo “haver”, no sentido de existir, não se flexiona no plural. Veja a correção:

    Não teria havido ataque ao “Charlie Hebdo”, se não houvesse imigrantes na França.

     

    - Fonte: estratégia concursos


ID
2688748
Banca
FUMARC
Órgão
SEE-MG
Ano
2018
Provas
Disciplina
Português
Assuntos

                                     Terrorismo lógico

                                                                                           Antônio Prata


      Said e Chérif Kouachi eram descendentes de imigrantes. Said e Chérif Kouachi são suspeitos do ataque ao jornal "Charlie Hebdo", na França. Se não houvesse imigrantes na França, não teria havido ataque ao "Charlie Hebdo".

      Said e Chérif Kouachi, suspeitos do ataque ao jornal "Charlie Hebdo", eram filhos de argelinos. Zinedine Zidane é filho de argelinos. Zinedine Zidane é terrorista.

      Zinedine Zidane é filho de argelinos. Said e Chérif Kouachi, suspeitos do ataque ao jornal "Charlie Hebdo", eram filhos de argelinos. Said e Chérif Kouachi sabiam jogar futebol.

      Muçulmanos são uma minoria na França. Membros de uma minoria são suspeitos do ataque terrorista. Olha aí no que dá defender minoria...

      A esquerda francesa defende minorias. Membros de uma minoria são suspeitos pelo ataque terrorista. A esquerda francesa é culpada pelo ataque terrorista.

      A extrema direita francesa demoniza os imigrantes. O ataque terrorista fortalece a extrema direita francesa. A extrema direita francesa está por trás do ataque terrorista.

      Marine Le Pen é a líder da extrema direita francesa. "Le Pen" é "O Caneta", se tomarmos o artigo em francês e o substantivo em inglês. Eis aí uma demonstração de apoio da extrema direita francesa à liberdade de expressão – e aos erros de concordância nominal.

      (Este último parágrafo não fez muito sentido. Os filmes do David Lynch não fazem muito sentido. Este último parágrafo é um filme do David Lynch.)

      O "Charlie Hebdo" zoava Maomé. Eu zoo negão, zoo as bichinhas, zoo gorda, zoo geral! "Je suis Charlie!"

      Humoristas brasileiros fazem piada racista, e as pessoas os criticam. "Charlie Hebdo" fez piada com religião, e terroristas o atacam. Criticar piada racista é terrorismo. 

      Numa democracia, é desejável que as pessoas sejam livres para se expressar. Algumas dessas expressões podem ofender indivíduos ou grupos. Numa democracia, é desejável que indivíduos ou grupos sejam ofendidos.

      O "Charlie Hebdo" foi atacado por terroristas. A editora Abril foi pichada por meia dúzia de jacus. A editora Abril é Charlie.

      Os terroristas que atacaram o jornal "Charlie Hebdo" usavam gorros pretos. "Black blocs" usam gorros pretos. "Black blocs" são terroristas.

      "Black blocs" não são terroristas. A polícia os trata como terroristas. Os "black blocs" têm o direito de tocar o terror.

      Os terroristas que atacaram o jornal "Charlie Hebdo" usavam gorros pretos. Drones não usam gorros pretos. Ataques com drones não são terrorismo.

      Ataques com drones matam inocentes mundo afora. O "Ocidente" usa drones. É justificável o terror contra o "Ocidente".

      O ataque terrorista contra o "Charlie Hebdo" foi no dia 7/1. A derrota brasileira para a Alemanha foi por 7 x 1. O 7 e o 1 devem ser imediatamente presos e submetidos a "técnicas reforçadas de interrogatório", tais como simulação de afogamento, privação de sono e alimentação via retal. Por via das dúvidas, o 6 e o 8 e o 0 e o 2 também.

      Todo abacate é verde. O Incrível Hulk é verde. O Incrível Hulk é um abacate. 

(Disponível em: http://www1.folha.uol.com.br/colunas/antonioprata/2015/01/1573334-terrorismo-logico.shtml. Acesso em: 2 fev. 2015.)

Observe os excertos retirados do texto, atentando para os itens grifados.


I. [...] não teria havido ataque ao "Charlie Hebdo".

II. Marine Le Pen é a líder da extrema direita francesa.

III. Ataques com drones matam inocentes mundo afora.

IV. É justificável o terror contra o "Ocidente".


Exercem a mesma função sintática os itens grifados em:

Alternativas
Comentários
  • Em I. a locução verbal “teria havido” é transitiva direta e a expressão “ataque ao ‘Charlie Hebdo’” é o objeto direto. Note que “ao ‘Charlie Hebdo’” é uma extensão do objeto direto, pois é um complemento nominal.

     

    Em II. o verbo “é” é de ligação e “a líder da extrema direita francesa” é o predicativo.

     

    Em III, o verbo “matam” é transitivo direto e “inocentes” é o objeto direto.

     

    Em IV. “É” é verbo de ligação, “justificável” é o predicativo e “o terror contra o ‘Ocidente’” é o sujeito.

     

    GabaritoB

     

    - Fonte: Professor Décio Terror estratégia concursos

  • GABARITO B


    Pessoal, lembrando que o verbo HAVER não tem sujeito e sim complemento verbal (objeto direto), sempre.


    bons estudos

  • I. [...] não teria havido ataque ao "Charlie Hebdo".  (objeto direto)

    o verbo haver com sentido de existir é um verbo impessoal e transitivo direto, ou seja, não possui sujeito, apenas objeto direto;

     

    II. Marine Le Pen é a líder da extrema direita francesa. (predicativo do sujeito)

    trata-se de um predicativo do sujeito que qualifica o mesmo e separado dele por um VL. Lembrando que o PS acontece com qualquer verbo;

     

    III. Ataques com drones matam inocentes mundo afora. (objeto direto)

    o termo inocentes completa o sentido do VTD matar (geralmente é intransitivo, mas nesse caso pede objeto);

     

    IV. É justificável o terror contra o "Ocidente". (sujeito)

    trata-se do sujeito da oração. Basta perguntar ao verbo "o que é justificável?"

     

    Quem escolheu a busca não pode recusar a travessia - Guimarães Rosa

    ------------------- 

    Gabarito: B

  • GABARITO: LETRA B

    I. [...] não teria havido ataque ao "Charlie Hebdo".

    II. Marine Le Pen é a líder da extrema direita francesa.

    III. Ataques com drones matam inocentes mundo afora.

    IV. É justificável o terror contra o "Ocidente".

  • Em I, o termo "ataque ao 'Charlie Hebdo'" atua como objeto direto da construção impessoal - sem sujeito - "teria havido".

    Em II, o termo "a líder da extrema direita francesa" atua como predicativo do sujeito. Note a presença do verbo de ligação "é".

    Em III, o termo "inocentes" atua como objeto direto de "matam".

    Por fim, em IV, o termo "o terror contra o 'Ocidente'" funciona como sujeito do predicado "É justificável".

    Exercem, portanto, a mesma função sintática os itens destacados em I e III.

    Resposta: Letra B

  • Se todas as questões da FUMARC fossem assim (difíceis, mas com uma explicação lógica)... Estaríamos no Paraíso.

    1. Objeto direto
    2. Predicativo do Sujeito
    3. Objeto direto
    4. Sujeito

    1 e 3 possuem a mesma função sintática.

    gabarito, letra B


ID
2688751
Banca
FUMARC
Órgão
SEE-MG
Ano
2018
Provas
Disciplina
Português
Assuntos

                                     Terrorismo lógico

                                                                                           Antônio Prata


      Said e Chérif Kouachi eram descendentes de imigrantes. Said e Chérif Kouachi são suspeitos do ataque ao jornal "Charlie Hebdo", na França. Se não houvesse imigrantes na França, não teria havido ataque ao "Charlie Hebdo".

      Said e Chérif Kouachi, suspeitos do ataque ao jornal "Charlie Hebdo", eram filhos de argelinos. Zinedine Zidane é filho de argelinos. Zinedine Zidane é terrorista.

      Zinedine Zidane é filho de argelinos. Said e Chérif Kouachi, suspeitos do ataque ao jornal "Charlie Hebdo", eram filhos de argelinos. Said e Chérif Kouachi sabiam jogar futebol.

      Muçulmanos são uma minoria na França. Membros de uma minoria são suspeitos do ataque terrorista. Olha aí no que dá defender minoria...

      A esquerda francesa defende minorias. Membros de uma minoria são suspeitos pelo ataque terrorista. A esquerda francesa é culpada pelo ataque terrorista.

      A extrema direita francesa demoniza os imigrantes. O ataque terrorista fortalece a extrema direita francesa. A extrema direita francesa está por trás do ataque terrorista.

      Marine Le Pen é a líder da extrema direita francesa. "Le Pen" é "O Caneta", se tomarmos o artigo em francês e o substantivo em inglês. Eis aí uma demonstração de apoio da extrema direita francesa à liberdade de expressão – e aos erros de concordância nominal.

      (Este último parágrafo não fez muito sentido. Os filmes do David Lynch não fazem muito sentido. Este último parágrafo é um filme do David Lynch.)

      O "Charlie Hebdo" zoava Maomé. Eu zoo negão, zoo as bichinhas, zoo gorda, zoo geral! "Je suis Charlie!"

      Humoristas brasileiros fazem piada racista, e as pessoas os criticam. "Charlie Hebdo" fez piada com religião, e terroristas o atacam. Criticar piada racista é terrorismo. 

      Numa democracia, é desejável que as pessoas sejam livres para se expressar. Algumas dessas expressões podem ofender indivíduos ou grupos. Numa democracia, é desejável que indivíduos ou grupos sejam ofendidos.

      O "Charlie Hebdo" foi atacado por terroristas. A editora Abril foi pichada por meia dúzia de jacus. A editora Abril é Charlie.

      Os terroristas que atacaram o jornal "Charlie Hebdo" usavam gorros pretos. "Black blocs" usam gorros pretos. "Black blocs" são terroristas.

      "Black blocs" não são terroristas. A polícia os trata como terroristas. Os "black blocs" têm o direito de tocar o terror.

      Os terroristas que atacaram o jornal "Charlie Hebdo" usavam gorros pretos. Drones não usam gorros pretos. Ataques com drones não são terrorismo.

      Ataques com drones matam inocentes mundo afora. O "Ocidente" usa drones. É justificável o terror contra o "Ocidente".

      O ataque terrorista contra o "Charlie Hebdo" foi no dia 7/1. A derrota brasileira para a Alemanha foi por 7 x 1. O 7 e o 1 devem ser imediatamente presos e submetidos a "técnicas reforçadas de interrogatório", tais como simulação de afogamento, privação de sono e alimentação via retal. Por via das dúvidas, o 6 e o 8 e o 0 e o 2 também.

      Todo abacate é verde. O Incrível Hulk é verde. O Incrível Hulk é um abacate. 

(Disponível em: http://www1.folha.uol.com.br/colunas/antonioprata/2015/01/1573334-terrorismo-logico.shtml. Acesso em: 2 fev. 2015.)

Assinale a alternativa em que o hífen tenha sido CORRETAMENTE utilizado na formação de compostos e na indicação de divisão silábica, em situação de escrita de um texto, tendo em conta que a barra sinaliza final de linha.

Alternativas
Comentários
  • Erros marcados em vermelho:

    a) malcom-/portado – cor-de-/rosa – mal-/-sucedido (malsucedido)

    b) mesoclí-/tico – dois-/-pontos – pré-/datado (pré-datado)

    c) pon/to-e-vírgula (Ponto e vírgula) – anti-/-infeccioso – ante-/-projeto (anteprojeto)

    d) subu-/mano – hiper-/-realismo – mãe-d’á-/gua

    e) sub-/-locatário (sublocatário) – pree-/xistente – geo-/histórico (ge·o·is·tó·ri·co)

  • Que questão confusa...

  • Subumano ou Sub-humano?

     

    Com o advento do Novo Acordo Ortográfico, algumas situações estranhas aconteceram. Uma delas é referente à dupla que dá título a esse post.

     

    Antes do Acordo, a palavra era grafada da primeira forma “subumano”. Com a adoção da nova regra, ficou estabelecido que os prefixos terminados em “b” trariam hífen sempre que acompanhassem palavras iniciadas em “b” ,“r” ou “h”.

     

    O primeiro caso acontece porque a nova regra aplica o hífen quando há coincidência da última letra do prefixo com a primeira da palavra subsequente. Ou seja, no caso de “sub”, não podemos ter um emprego de dois “b”. 

    Ex: Sub-base

     

    Já no segundo caso, o hífen aparece para evitar uma confusão na pronúncia. Tente ler a palavras “subreitor” ou “subregião”. Logo, optou-se pelo uso do hífen: “sub-reitor”, “sub-região”.

     

    Mas e o caso de “subumano”? Não seria “sub-humano” para seguir a regra?

     

    Resposta: as duas formas são aceitas pelo Vocabulário Oficial da Língua Portuguesa (Volp).

     

    A forma “sub-humano” é aceita, porque segue corretamente o estabelecido pelo Acordo Ortográfico. Porém, optou-se por manter também a segunda forma (“subumano”) por se entender que já é consagrada pelo uso.

     

    Fonte: http://dicasdeportugues.tumblr.com/post/37880312374/subumano-ou-sub-humano

  • que lixo de questão quem fez tava drogado

  • Nos prefixos SUB, HIPER, INTER e SUPER, permanece o hífen se a palavra seguinte for iniciada por H ou R.

    inter-racial

    inter-hospitalar

    hiper-realista

    hiper-requintado

    sub-hepático

    sub-humano

     

    LIVRO PORTUGUÊS DESCOMPLICADO - FLÁVIA RITA

  • não entendi essa questão

  • Algumas regrinhas

    * A letra “H” não deve aparecer encostada em prefixos (pré-história)

    * Letras iguais, separa; letras diferentes, junta (Super-requintado / Anti-inflamatório / neoliberalismo)

    * Quanto ao "R" e o "S", se o prefixo terminar em vogal, a consoante deverá ser dobrada (minissaia / contrarregra)

    * Se o prefixo terminar em consoante, não se unem de jeito nenhum (Sub-reino)

    * Com os prefixos vice, vizo, grã, grão, além, aquém, ex (anterior), recém, sem, soto(a), para (exceto paraquedas e derivados dele), usa-se sempre o hífen (vice-rei / vice-almirante)

    * Nos prefixos sub, hiper, inter e super permanece o hífen se a palavra seguinte for iniciada por H ou R (inter-racial / inter-hospitalar)

     

    FONTE: https://www.infoescola.com/portugues/uso-do-hifen-descomplicado/

  • Corrigindo a correção do colega que postou anteriormente:

     

    Erros marcados em vermelho:

    a) malcom-/portado – cor-de-/rosa (cor-de-/-rosa) – mal-/-sucedido (mal-/sucedido)

    b) mesoclí-/tico – dois-/-pontos – pré-/datado (pré-/-datado)

    c) pon/to-e-vírgula (pon-/to e vírgula) – anti-/-infeccioso – ante-/-projeto (ante-/projeto)

    d) subu-/mano – hiper-/-realismo – mãe-d’á-/gua

    e) sub-/-locatário (sub-/locatário) – pree-/xistente (pre-/existente) geo-/histórico (geo-/-histórico)

     

    O detalhe dessa questão, pessoal, é que a / significa o final de uma linha. Então, devemos entender que ao final de cada linha, caso não dê para escrever a palavra toda, usase o hífen não é? Mas se a palavra já tiver hífen justamente onde será separada? Usa-se dois hífens, um no final da linha e outro no começo da linha seguinte.

  • Não sabia que tanto é possível usar sub-humano, como também subumano. As duas formas estão certas de acordo com o Novo Acordo Ortográfico

  • Cuida com os comentários, mal-sucedido POSSUÍ HÍFEN, tanto mal como bem...

  • difícil 

  • Com o prefixo sub", diante de palavras iniciadas por r usa-se o hífen . exemplos: sub-regional / sub-raça / sub- reino.

  • CUIDADO COM OS COMENTÁRIOS SEM FONTE!Repare que algumas palavras, DE ACORDO COM A GRAMÁTICA DE NÍLSON TEIXEIRA DE ALMEIDA, com prefixo( mal) são escritas sem hífen,como malsucedido,malcheiroso,malcozido,malcriado,malcomportado,maldormido e malresolvido.É comum  as pessoas ignorarem esse fato não só por não estarem acostumadas com essa grafia,como também por estabelecerem uma comparação com os antônimos de tais palavras,grafadas(bem-sucedido,bem-cozido,bem-criado, bem-comportado,bem-dormido e bem-resolvido). Fé em Deus!BONS  ESTUDOS.

  • decorar: BI.SA.VÓ BIS.NE.TO

  • Frustrante, não sei separar palavras

  • Agora que entendi a barra indicando final de linha.

  • Não entendi nada dessa questão
  • Questao looooool. Tendi nada
  • não entendi nada

  • GABARITO: LETRA D

  • https://youtu.be/9rIFTS9bi8Y

  • adjetivo

    Fracassado; que não é bem-sucedido; sem sucesso; em que há fracasso: negócio malsucedido; empresário malsucedido; projeto malsucedido.

    Etimologia (origem da palavra malsucedido). Mal + sucedido.

  • Ho loco meu, primeira que marquei como errado foi Subumano. Caí igual patinho na conversa da Banca.

  • Q loucura é essa vei

  • sem noção

  • não entendi nada

  • Uso do hífem na translineação

    Caso seja uma palavra hifenizada e o fim da linha coincidir com o hífen, este deverá ser repetido no início da outra linha.

    disseram-

    -me;

    couve-

    -flor.

  • Ino-varam!

  • Questão confusa.

    Atenção:

    Sub-humano e Subumano são aceitos pela nova ortografia da língua portuguesa

  • EU ACERTEI PORQUE RESPONDI OUTRAS QUESTOES E APRENDI A RESPEITO DA PALAVRA SUBUMANO......OS COMENTARIOS DE VOCES AJUDA MUITO

  • Gabarito: D

    "A questão cobra a chamada “translineação”, ou seja, a passagem de uma linha para outra.

    Na translineação (ou seja, na passagem para a linha seguinte quando se está escrevendo um texto)

    de uma palavra composta ou de uma combinação de palavras em que há um hífen, ou mais, se a partição

    coincide com o final de um dos elementos ou membros, por clareza gráfica, se deve repetir o hífen no início

    da linha seguinte: vice-/-almirante (a barra sinaliza final de linha).

    Assim, só cabe hífen antes e após a barra quando houver divisão silábica na palavra composta já com hífen.

    A alternativa (D) é a correta, pois “subumano” não apresenta hífen. Assim, na translineação, há

    apenas um hífen antes da barra. Na palavra “hiper-realismo”, já há hífen, por isso há hífen antes e depois da

    barra. Na palavra “mãe-d’água”, há hífen apenas entre a primeira e a segunda palavra, por isso cabe apenas

    um hífen antes da barra.

    Veja a correção de cada alternativa em negrito:

    (A) malcom-/portado – cor-de-/-rosa – mal-/sucedido

    (B) mesoclí-/tico – dois-/-pontos – pré-/-datado

    (C) pon-/to-e-vírgula – anti-/-infeccioso – ante-/projeto

    (E) sub-/locatário – pree-/xistente – geo-/-histórico"

    Fonte: Estratégia Concursos.

    Professor Décio Terror.

  • Sub-humano / Subumano = ok

    ________________________________________________________________________________________________________________________________

    hífen & translineação:

    hifen + fim da linha repete =

    hiper-

    -super

  • enunciado paia

  • Entendi foi nada.

  • Errei por falta de atenção; Pré-/datado - tem hífen - Pré-/-datado

  • Uma última dica: “pre” é diferente de “pré“. As palavras iniciadas com o prefixo “pre” serão sempre sem hífen, porque não têm autonomia fonética (é um prefixo átono), isto é, precisa se apoiar na sílaba seguinte.

    Ex.: preanunciação, predeterminação, preexistência.

    Por outro lado, quando as palavras começam com “pré-“, que é um prefixo tônico, usa-se o hífen para separar qualquer termo que venha posteriormente, pois há autônomia fonética.

    Ex.: pré-história, pré-natal, pré-adolescência.

  • QUE P0RR@ É ESSA ?

  •  Uso do hífen com o prefixo “sub-“?

    ->Segundo o novo acordo ortográfico, a única alteração é o fato de o uso do hífen ter-se tornado facultativo antes de palavras começadas por “h”: subumano ou sub-humano, subepático ou sub-hepático.

    ->Nos demais casos, a regra é a mesma anterior ao novo acordo: só há hífen se a palavra seguinte começar por “b” ou “r”: sub-base, sub-bibliotecária; sub-raça, sub-reino, sub-reitor...

    ->Isso significa que, diante de palavras começadas por qualquer letra diferente de “h”, “b” e “r”, devemos escrever sem hífen: subaquático, subemprego, subeditor, subitem, subchefe, subprefeitura, subsolo, subsecretário...


ID
2688754
Banca
FUMARC
Órgão
SEE-MG
Ano
2018
Provas
Disciplina
Português
Assuntos

                                     Terrorismo lógico

                                                                                           Antônio Prata


      Said e Chérif Kouachi eram descendentes de imigrantes. Said e Chérif Kouachi são suspeitos do ataque ao jornal "Charlie Hebdo", na França. Se não houvesse imigrantes na França, não teria havido ataque ao "Charlie Hebdo".

      Said e Chérif Kouachi, suspeitos do ataque ao jornal "Charlie Hebdo", eram filhos de argelinos. Zinedine Zidane é filho de argelinos. Zinedine Zidane é terrorista.

      Zinedine Zidane é filho de argelinos. Said e Chérif Kouachi, suspeitos do ataque ao jornal "Charlie Hebdo", eram filhos de argelinos. Said e Chérif Kouachi sabiam jogar futebol.

      Muçulmanos são uma minoria na França. Membros de uma minoria são suspeitos do ataque terrorista. Olha aí no que dá defender minoria...

      A esquerda francesa defende minorias. Membros de uma minoria são suspeitos pelo ataque terrorista. A esquerda francesa é culpada pelo ataque terrorista.

      A extrema direita francesa demoniza os imigrantes. O ataque terrorista fortalece a extrema direita francesa. A extrema direita francesa está por trás do ataque terrorista.

      Marine Le Pen é a líder da extrema direita francesa. "Le Pen" é "O Caneta", se tomarmos o artigo em francês e o substantivo em inglês. Eis aí uma demonstração de apoio da extrema direita francesa à liberdade de expressão – e aos erros de concordância nominal.

      (Este último parágrafo não fez muito sentido. Os filmes do David Lynch não fazem muito sentido. Este último parágrafo é um filme do David Lynch.)

      O "Charlie Hebdo" zoava Maomé. Eu zoo negão, zoo as bichinhas, zoo gorda, zoo geral! "Je suis Charlie!"

      Humoristas brasileiros fazem piada racista, e as pessoas os criticam. "Charlie Hebdo" fez piada com religião, e terroristas o atacam. Criticar piada racista é terrorismo. 

      Numa democracia, é desejável que as pessoas sejam livres para se expressar. Algumas dessas expressões podem ofender indivíduos ou grupos. Numa democracia, é desejável que indivíduos ou grupos sejam ofendidos.

      O "Charlie Hebdo" foi atacado por terroristas. A editora Abril foi pichada por meia dúzia de jacus. A editora Abril é Charlie.

      Os terroristas que atacaram o jornal "Charlie Hebdo" usavam gorros pretos. "Black blocs" usam gorros pretos. "Black blocs" são terroristas.

      "Black blocs" não são terroristas. A polícia os trata como terroristas. Os "black blocs" têm o direito de tocar o terror.

      Os terroristas que atacaram o jornal "Charlie Hebdo" usavam gorros pretos. Drones não usam gorros pretos. Ataques com drones não são terrorismo.

      Ataques com drones matam inocentes mundo afora. O "Ocidente" usa drones. É justificável o terror contra o "Ocidente".

      O ataque terrorista contra o "Charlie Hebdo" foi no dia 7/1. A derrota brasileira para a Alemanha foi por 7 x 1. O 7 e o 1 devem ser imediatamente presos e submetidos a "técnicas reforçadas de interrogatório", tais como simulação de afogamento, privação de sono e alimentação via retal. Por via das dúvidas, o 6 e o 8 e o 0 e o 2 também.

      Todo abacate é verde. O Incrível Hulk é verde. O Incrível Hulk é um abacate. 

(Disponível em: http://www1.folha.uol.com.br/colunas/antonioprata/2015/01/1573334-terrorismo-logico.shtml. Acesso em: 2 fev. 2015.)

Em todas as alternativas, o hífen foi utilizado de forma incorreta ao menos uma vez, EXCETO em:

Alternativas
Comentários
  • Há hífen:

    a) Antes de H (sub-humano)

    b) Vogal/Consoante igual (micro-ondas)

    c) Sub+R/B (sub-remunerado)

  • Então, letras IGUAIS, SEPARA  Letras DIFERENTES, JUNTA.

    Anti-inflamatório                       

    neoliberalismo

    Quanto ao "R" e o "S", se o prefixo terminar em vogal, a consoante deverá ser dobrada:

    minissaia                                         

    antisséptico

    Há hifén antes de H 

    super-homem

  • Erros marcados em vermelho:

    a) sub-humano, micro-ondas, socioeconômico, sub-remunerado (O Volp registra as duas formas para: subumano e sub-humano)

    b) hiper-sensibilidade, ultravioleta, infravermelho, anticorrupção

    c) hipersensibilidade, inter-regional, super-aquecimento, inter-sindical

    d) contracheque, contragolpe, contra-reforma, contra-senso

    e) anti-inflamatório, anteprojeto, antiabortivo, anti-social

  • Fonte: https://www.estrategiaconcursos.com.br/blog/comentario-da-prova-de-portugues-see-mg-2018/

    A alternativa (A) é a correta, pois as palavras estão corretamente grafadas.

    A alternativa (B) está errada, pois “hipersensibilidade” não apresenta hífen.

    A alternativa (C) está errada, pois “superaquecimento” e “intersindical” não apresentam hífen.

    A alternativa (D) está errada, pois “contrarreforma” e “contrassenso” não apresentam hífen.

    A alternativa (E) está errada, pois “antissocial” não apresenta hífen.

    GabaritoA

     

  • Algumas regrinhas

    * A letra “H” não deve aparecer encostada em prefixos (pré-história)

    * Letras iguais, separa; letras diferentes, junta (Super-requintado / Anti-inflamatório / neoliberalismo)

    * Quanto ao "R" e o "S", se o prefixo terminar em vogal, a consoante deverá ser dobrada (minissaia / contrarregra)

    * Se o prefixo terminar em consoante, não se unem de jeito nenhum (Sub-reino)

    * Com os prefixos vice, vizo, grã, grão, além, aquém, ex (anterior), recém, sem, soto(a), para (exceto paraquedas e derivados dele), usa-se sempre o hífen (vice-rei / vice-almirante)

    * Nos prefixos sub, hiper, inter e super permanece o hífen se a palavra seguinte for iniciada por H ou R (inter-racial / inter-hospitalar)

     

    FONTE: https://www.infoescola.com/portugues/uso-do-hifen-descomplicado/

  • Subumano ou Sub-humano?

     

    Antes do Acordo, a palavra era grafada da primeira forma “subumano”. Com a adoção da nova regra, ficou estabelecido que os prefixos terminados em “b” trariam hífen sempre que acompanhassem palavras iniciadas em “b” ,“r” ou “h”.

     

    O primeiro caso acontece porque a nova regra aplica o hífen quando há coincidência da última letra do prefixo com a primeira da palavra subsequente. Ou seja, no caso de “sub”, não podemos ter um emprego de dois “b”. 

    Ex: Sub-base

     

    Já no segundo caso, o hífen aparece para evitar uma confusão na pronúncia. Tente ler a palavras “subreitor” ou “subregião”. Logo, optou-se pelo uso do hífen: “sub-reitor”, “sub-região”.

     

    Mas e o caso de “subumano”? Não seria “sub-humano” para seguir a regra?

     

    Resposta: as duas formas são aceitas pelo Vocabulário Oficial da Língua Portuguesa (Volp).

     

    A forma “sub-humano” é aceita, porque segue corretamente o estabelecido pelo Acordo Ortográfico. Porém, optou-se por manter também a segunda forma (“subumano”) por se entender que já é consagrada pelo uso.

     

    Fonte: http://dicasdeportugues.tumblr.com/post/37880312374/subumano-ou-sub-humano

  • Resposta: Letra A: sub-humano; micro-ondas; socioeconômico; sub-remunerado
    Letra B: hipersensibilidade; ultravioleta; infravermelho; anticorrupção
    Letra C: hipersensibilidade; inter-regional; superaquecimento; intersindical 
    Letra D: contracheque; contragolpe; contrarreforma; contrassenso
    Letra E: anti-inflamatório, anteprojeto, antiabortivo, antissocial 

  • Letra Asub-humano; micro-ondas; socioeconômico; sub-remunerado
    Letra B: hipersensibilidade; ultravioleta; infravermelho; anticorrupção
    Letra C: hipersensibilidade; inter-regional; superaquecimento; intersindical 
    Letra D: contracheque; contragolpe; contrarreforma; contrassenso
    Letra E: anti-inflamatório, anteprojeto, antiabortivo, antissocial 

  • GABARITO LETRA A

     

    B: hiper-sensibilidade (ERRADO); ultravioleta; infravermelho; anticorrupção  = hipersensibilidade (CORRETO) - Com os prefixos HIPER, INTER e SUPER Só haverá hífen se a palavra seguinte começar por H ou R.


     C: hipersensibilidade; inter-regional; super-aquecimento (ERRADO)inter-sindical (ERRADO) = superaquecimento (CORRETO), intersindical (CORRETO)Com os prefixos HIPER, INTER e SUPER Só haverá hífen se a palavra seguinte começar por H ou R.


     D: contracheque; contragolpe; contra-reforma (ERRADO)contra-senso (ERRADO) = contrarreforma (CORRETO), contrassenso (CORRETO) - Com os prefixos AUTO, CONTRA, EXTRA, INFRA, INTRA, NEO, PROTO, PSEUDO, SEMI, SUPRA, ULTRA, ANTE, ANTI, ARQUI e SOBRE
    Se o segundo elemento começa por S ou R, devemos dobrar as consoantes, em vez de usar o hífen.


     E: anti-inflamatório, anteprojeto, antiabortivo, anti-social = antissocial (CORRETO) - Não se usa hífen se o prefixo terminar com letra diferente daquela com que se inicia outra palavra. OBS.: SE O PREFIXO TERMINAR POR VOGAL E A OUTRA PALAVRA COMEÇAR POR R OU S, DOBRAM-SE ESSAS LETRAS. Ex.: ultrassom.

  • Não concordo ser a letra A, porque a única execeção do hífen antes do H é na palavra sub-humano que passa a ficar subumano

     

     

  • Taiza Vieira, as dua formas estão corretas. "Subumano" e "Sub-humano".

  • SUB/SOB - mantiveram o hífen com H, B, R. Atentar para a possibilidade do uso de subumano.


    Sub-reino; Sob-homenagem

  • fez sentido nenhum esse gabarito, acho que nao entendi direito o enunciado

  • GABARITO: LETRA A

  • FUMAC 2018: Sub-humano

    FUNRIO 2018: Subumano

    Regras: (Prof Duda)

    1) Tem hífen os prefixos SUB e SOB, mesmo que o segundo elemento inicie por R:Sub-região, Sob-roda

    2) Usa-se hífen no segundo elemento iniciadas por H: anti-higiênico, sobre-humano

    3) Não tem hífen se o segundo elemento perdeu o H inicial: Desumano, Inábil

    Sites:

    Dicio.com.br: Subumano

    educacao.uol.com.br: Subumano ou sub-humano

    Professor Sérgio Nogueira: O hífen é facultativo antes das palavras começadas por H (Subumano ou sub-humano, Subepático ou Sub-hepático) Nos demais casos, a regra é a anterior ao acordo: só há hífen se a palavra seguinte começar por “b” ou “r”

    http://g1.globo.com/educacao/blog/dicas-de-portugues/27.html

  • https://youtu.be/9rIFTS9bi8Y

  • Fui eliminando uma por uma, até que eliminei todas, kkk. Voltei ao início e lembrei que são aceitos subumano e sub-humano.

  • Na letra B, deve-se grafar "hipersensibilidade", haja vista que, na junção de prefixo e palavras, vale a máxima: "os diferentes se atraem".

    Na letra C, deve-se grafar "superaquecimento" e "intersindical", haja vista que, na junção de prefixo e palavras, vale a máxima: "os diferentes se atraem".

    Na letra D, deve-se grafar "contrarreforma" e "contrassenso", haja vista que, na junção de prefixo e palavras, vale a máxima: "os diferentes se atraem". Nesse caso, é preciso dobrar o R e o S.

    Na letra E, deve-se grafar "antissocial", haja vista que, na junção de prefixo e palavras, vale a máxima: "os diferentes se atraem". Nesse caso, é preciso dobrar o S.

    Na letra A, todas as palavras foram grafadas corretamente!

  • PRA QUEM NÃO ENTENDEU O ENUNCIADO:

    A QUESTÃO QUER SABER QUAL DAS ALTERNATIVAS ESTA 100% CORRETO.

  • os opostos se atraem e os iguais se repelem......,

    ex; Micro- ondas (o/o), hipersensibilidade (R/S)

    vogaais diante de R/S, voce dobra a consoante

    ex; anti social= Antissocial

    contra reforma= Contrarreforma

    Mini saia= Minissaia

  • A sub-humano(OU SUBUMANO), micro-ondas, socioeconômico, sub-remunerado

    B hiper-sensibilidade, ultravioleta, infravermelho, anticorrupção

    C hipersensibilidade, inter-regional, super-aquecimento, inter-sindical

    Dcontracheque, contragolpe, contra-reforma, contra-senso

    Eanti-inflamatório, anteprojeto, antiabortivo, anti-social

  • Para responder esta questão, exige-se do candidato conhecimento em regra de hífen. O candidato deve indicar qual assertiva está correta. Vejamos:

    a) Correta.

    Sub-humano⇢ o prefixo sub com palavras que começam com letras H, B ou R, leva hífen.

    Micro-ondas⇢ no prefixo “micro” se coloca hífen quando a palavra seguinte iniciar com H ou O.

    Socioeconômico⇢ O prefixo “socio” apenas aceita o hífen quando a palavra seguinte iniciar com “o” ou “h”.

    Sub-remunerado⇢ o prefixo sub com palavras que começam com letras H, B ou R, leva hífen.

    Portanto, todas estão corretas.

    b) Incorreta.

    hiper-sensibilidade⇢ o prefixo "hiper” apenas se usa hífen quando a palavra seguinte tiver H ou R. “hipersensibilidade".

    c) Incorreta

    Super-aquecimento⇢ o prefixo sub com palavras que começam com letras H ou R, leva hífen, portanto, o correto é "superaquecimento".

    Intersindical⇢o prefixo "inter" só se coloca hífen quando a palavra seguinte terminar com R ou H.

    d) Incorreta.

    Contra-reforma, contra-sens⇢ o prefixo contra apenas é hifenizado quando a palavra seguinte começar com A ou H, portanto, o correto era a escrita ser sem hífen.

    e) Incorreta.

    Anti-social⇢ o prefixo “anti” apenas fica com hífen quando se une com a palavra que inicia com “i” ou “h”, portanto, o correto é “antissocial”. O S foi dobrado, porque quando a palavra seguinte começar com essa consoante, o correto é SS.

    Gabarito do monitor: A


ID
2688757
Banca
FUMARC
Órgão
SEE-MG
Ano
2018
Provas
Disciplina
Português
Assuntos

                                     Terrorismo lógico

                                                                                           Antônio Prata


      Said e Chérif Kouachi eram descendentes de imigrantes. Said e Chérif Kouachi são suspeitos do ataque ao jornal "Charlie Hebdo", na França. Se não houvesse imigrantes na França, não teria havido ataque ao "Charlie Hebdo".

      Said e Chérif Kouachi, suspeitos do ataque ao jornal "Charlie Hebdo", eram filhos de argelinos. Zinedine Zidane é filho de argelinos. Zinedine Zidane é terrorista.

      Zinedine Zidane é filho de argelinos. Said e Chérif Kouachi, suspeitos do ataque ao jornal "Charlie Hebdo", eram filhos de argelinos. Said e Chérif Kouachi sabiam jogar futebol.

      Muçulmanos são uma minoria na França. Membros de uma minoria são suspeitos do ataque terrorista. Olha aí no que dá defender minoria...

      A esquerda francesa defende minorias. Membros de uma minoria são suspeitos pelo ataque terrorista. A esquerda francesa é culpada pelo ataque terrorista.

      A extrema direita francesa demoniza os imigrantes. O ataque terrorista fortalece a extrema direita francesa. A extrema direita francesa está por trás do ataque terrorista.

      Marine Le Pen é a líder da extrema direita francesa. "Le Pen" é "O Caneta", se tomarmos o artigo em francês e o substantivo em inglês. Eis aí uma demonstração de apoio da extrema direita francesa à liberdade de expressão – e aos erros de concordância nominal.

      (Este último parágrafo não fez muito sentido. Os filmes do David Lynch não fazem muito sentido. Este último parágrafo é um filme do David Lynch.)

      O "Charlie Hebdo" zoava Maomé. Eu zoo negão, zoo as bichinhas, zoo gorda, zoo geral! "Je suis Charlie!"

      Humoristas brasileiros fazem piada racista, e as pessoas os criticam. "Charlie Hebdo" fez piada com religião, e terroristas o atacam. Criticar piada racista é terrorismo. 

      Numa democracia, é desejável que as pessoas sejam livres para se expressar. Algumas dessas expressões podem ofender indivíduos ou grupos. Numa democracia, é desejável que indivíduos ou grupos sejam ofendidos.

      O "Charlie Hebdo" foi atacado por terroristas. A editora Abril foi pichada por meia dúzia de jacus. A editora Abril é Charlie.

      Os terroristas que atacaram o jornal "Charlie Hebdo" usavam gorros pretos. "Black blocs" usam gorros pretos. "Black blocs" são terroristas.

      "Black blocs" não são terroristas. A polícia os trata como terroristas. Os "black blocs" têm o direito de tocar o terror.

      Os terroristas que atacaram o jornal "Charlie Hebdo" usavam gorros pretos. Drones não usam gorros pretos. Ataques com drones não são terrorismo.

      Ataques com drones matam inocentes mundo afora. O "Ocidente" usa drones. É justificável o terror contra o "Ocidente".

      O ataque terrorista contra o "Charlie Hebdo" foi no dia 7/1. A derrota brasileira para a Alemanha foi por 7 x 1. O 7 e o 1 devem ser imediatamente presos e submetidos a "técnicas reforçadas de interrogatório", tais como simulação de afogamento, privação de sono e alimentação via retal. Por via das dúvidas, o 6 e o 8 e o 0 e o 2 também.

      Todo abacate é verde. O Incrível Hulk é verde. O Incrível Hulk é um abacate. 

(Disponível em: http://www1.folha.uol.com.br/colunas/antonioprata/2015/01/1573334-terrorismo-logico.shtml. Acesso em: 2 fev. 2015.)

Tendo em conta o Acordo Ortográfico de 1990, assinale a afirmativa CORRETA.

Alternativas
Comentários
  • Comentário: A alternativa (A) é a correta, justamente porque “creem”, “leem” e “deem” perderam o acento circunflexo com o Acordo Ortográfico de 1990.

    A alternativa (B) está errada, pois nos hiatos, somente o “i” e o “u” tônicos nas palavras paroxítonas deixaram de ser acentuados graficamente sempre que antecedidos de ditongos. As oxítonas, por exemplo, como “Piauí”, “tuiuiú”, são acentuadas.

    A alternativa (C) está errada, pois o trema, em palavras como “Müller” e seus derivados “mülleriano”, permanece.

    A alternativa (D) está errada, pois acentos diferenciais continuam a ser empregados, como em “têm”, “vêm”, “pôr”, “pôde”.

    A alternativa (E) está errada, pois os ditongos abertos “eu”, “ei” e “oi” são acentuados graficamente, nas palavras oxítonas e monossilábicas tônicas.

  • Jurava que a mudança do item A tinha acontecido em 2009.
  • Eles creem que até o final da tarde o problema será resolvido.

    As crianças veem o filme enquanto esperam para ir embora.

    As alunas leem o livro que o professor indicou.

  • Basta lembrar do "credeleve" - creem, deem, leem e veem não mais se acentuam.


ID
2688760
Banca
FUMARC
Órgão
SEE-MG
Ano
2018
Provas
Disciplina
Português
Assuntos

                                     Terrorismo lógico

                                                                                           Antônio Prata


      Said e Chérif Kouachi eram descendentes de imigrantes. Said e Chérif Kouachi são suspeitos do ataque ao jornal "Charlie Hebdo", na França. Se não houvesse imigrantes na França, não teria havido ataque ao "Charlie Hebdo".

      Said e Chérif Kouachi, suspeitos do ataque ao jornal "Charlie Hebdo", eram filhos de argelinos. Zinedine Zidane é filho de argelinos. Zinedine Zidane é terrorista.

      Zinedine Zidane é filho de argelinos. Said e Chérif Kouachi, suspeitos do ataque ao jornal "Charlie Hebdo", eram filhos de argelinos. Said e Chérif Kouachi sabiam jogar futebol.

      Muçulmanos são uma minoria na França. Membros de uma minoria são suspeitos do ataque terrorista. Olha aí no que dá defender minoria...

      A esquerda francesa defende minorias. Membros de uma minoria são suspeitos pelo ataque terrorista. A esquerda francesa é culpada pelo ataque terrorista.

      A extrema direita francesa demoniza os imigrantes. O ataque terrorista fortalece a extrema direita francesa. A extrema direita francesa está por trás do ataque terrorista.

      Marine Le Pen é a líder da extrema direita francesa. "Le Pen" é "O Caneta", se tomarmos o artigo em francês e o substantivo em inglês. Eis aí uma demonstração de apoio da extrema direita francesa à liberdade de expressão – e aos erros de concordância nominal.

      (Este último parágrafo não fez muito sentido. Os filmes do David Lynch não fazem muito sentido. Este último parágrafo é um filme do David Lynch.)

      O "Charlie Hebdo" zoava Maomé. Eu zoo negão, zoo as bichinhas, zoo gorda, zoo geral! "Je suis Charlie!"

      Humoristas brasileiros fazem piada racista, e as pessoas os criticam. "Charlie Hebdo" fez piada com religião, e terroristas o atacam. Criticar piada racista é terrorismo. 

      Numa democracia, é desejável que as pessoas sejam livres para se expressar. Algumas dessas expressões podem ofender indivíduos ou grupos. Numa democracia, é desejável que indivíduos ou grupos sejam ofendidos.

      O "Charlie Hebdo" foi atacado por terroristas. A editora Abril foi pichada por meia dúzia de jacus. A editora Abril é Charlie.

      Os terroristas que atacaram o jornal "Charlie Hebdo" usavam gorros pretos. "Black blocs" usam gorros pretos. "Black blocs" são terroristas.

      "Black blocs" não são terroristas. A polícia os trata como terroristas. Os "black blocs" têm o direito de tocar o terror.

      Os terroristas que atacaram o jornal "Charlie Hebdo" usavam gorros pretos. Drones não usam gorros pretos. Ataques com drones não são terrorismo.

      Ataques com drones matam inocentes mundo afora. O "Ocidente" usa drones. É justificável o terror contra o "Ocidente".

      O ataque terrorista contra o "Charlie Hebdo" foi no dia 7/1. A derrota brasileira para a Alemanha foi por 7 x 1. O 7 e o 1 devem ser imediatamente presos e submetidos a "técnicas reforçadas de interrogatório", tais como simulação de afogamento, privação de sono e alimentação via retal. Por via das dúvidas, o 6 e o 8 e o 0 e o 2 também.

      Todo abacate é verde. O Incrível Hulk é verde. O Incrível Hulk é um abacate. 

(Disponível em: http://www1.folha.uol.com.br/colunas/antonioprata/2015/01/1573334-terrorismo-logico.shtml. Acesso em: 2 fev. 2015.)

Todas as alternativas trazem ocorrência(s) que contraria(m) o Acordo Ortográfico de 1990, EXCETO:

Alternativas
Comentários
  • Na alternativa (A), a paroxítona “hífen” deve ser acentuada.

    Na alternativa (B), a paroxítona “hifens” não tem acento e “polo”, com o novo Acordo Ortográfico de 1990, perdeu o acento.

    Na alternativa (C), “pôr do sol” tem acento e “contracheque” não tem hífen.

    A alternativa (D) é a correta, pois “raízes” tem acento no hiato, “papéis” tem acento por ser oxítona terminada em ditongo aberto tônico “éi”, seguido de “s”.

    Na alternativa (E), “ideias” não deve receber acento, por ser paroxítona terminada em “a”, seguido de “s”.

  • Apenas complementando o raciocínio do Carlos Lopes e retificando a explicação da última alternativa :

    Alternativa (A) -  a palavra "hífen" deve ser acentuada por ser paroxítona terminada em "n".

    Alternativa (B) - a palavra "hifens"  NÃO deve ser acentuada por ser uma paroxítona terminada em "em/ens".

    Alternativa (E) - a palavra "ideias" NÃO deve ser acentuada por ser paroxítona com ditongo aberto "ei/oi" e não pelo motivo que o colega explicou.

  • e o substantivo polo tambem nao se acentua mais 

  • Não acentua Paroxítonas terminadas em : EI,OI, EU


    Acentua Oxítonas terminadas em : EI,OI, EU

  • Na letra D a palavra "averigue" não deveria ter acento por ser uma paroxítona terminada em ditongo?

    "A-ve-rí-gue" ?



  • Errei por achar que averigui era paroxítona e na realidade é uma oxítona.

  • Fernando, sobre "averigue", o VOLP não consta essa palavra, mas no site do conjuga-me.net parece admitir as duas formas, "averigue" e "averígue".

  • Letra A - ERRADA - A palavra "hífen" deveria ser acentuada, pois é paroxítona terminada em N.

    Letra B - ERRADA - A palavra "hifens" não deveria possuir acento, pois é uma paroxítona terminada em ENS. O acento diferencial em "polo" foi abolido com o Novo Acordo Ortográfico.

    Letra C - ERRADA - Em "pôr do sol", devemos empregar o acento diferencial em "pôr", por se tratar do verbo. Além disso, a grafia correta deveria ser "contracheque" (na ligação de prefixo e palavra, vale a máxima dos "diferentes se atraem").

    Letra D - CERTA - As duas grafias - "averigue" e "averígue" - são corretas.

    Letra E - ERRADA - O vocábulo "ideias" não é mais acentuado - não mais se acentuam ditongos abertos em paroxítonas.

    Resposta: Letra D

  • RAÍZES -> HIATO

    PAPÉIS -> OXÍTONA EM DITONGO ABERTO

    AVERIGUE (SEM ACENTO)

  • Para responder esta questão, exige-se conhecimento no novo Acordo Ortográfico. O candidato deve indicar em qual assertiva todas as palavras estão corretamente escritas. Vejamos:

    a) Incorreta.

    Hi-fen⇢ ⇢ está errada, porque é acentuada por ter a penúltima sílaba mais forte e terminar em "N", ou seja, entra na regra da paroxítona terminada em "N". As paroxítonas são aquelas que têm sua sílaba mais forte na penúltima sílaba. Acentua-se paroxítona terminada em: i, is, us, um, uns, /, n, r, x, ons, ps, ei, eis, ã, ãs, ão, ãos, guam e as terminadas em ditongo. Ditongo é a união da semivogal com a vogal. Os semivogais são tradicionalmente "i" e "u". (Hífen)

    Tem⇢ correta, porque o verbo "ter" na terceira pessoa do singular não se usa acento.

    He-rói⇢ acentuada por ter a última sílaba mais forte e terminar com "ÓI", ou seja, entra na regra das oxítonas terminadas com "ÓI". Acentua-se a oxítona terminada em: a, e, o, seguidos ou não de s, em, ens (em palavras de duas ou mais sílabas), éis, éu(s), ói (s).

    Portanto, somente a primeira palavra contraria a regra de acentuação.

    b) Incorreta.

    -fens⇢ não se acentua a penúltima sílaba forte quando terminar em "ns". (Hifens).

    Cre-em⇢ essa é a escrita correta do verbo "crer" na terceira pessoa do plural.

    -lo⇢ essa palavra perdeu o acento com a reforma ortográfica. (Polo)

    Portanto, somente a primeira palavra e a terceira palavra contrariam a regra de acentuação.

    c) Incorreta.

    por do sol⇢ essa locução não perdeu o acento com a reforma ortográfica. (Pôr do sol)

    Contra-cheque⇢ o prefixo "contra" apenas é hifenizado quando a palavra que se junta inicia com "a" ou "h", assim, o correto é "contracheque".

    Es-car-céu⇢ acentuada por ter a última sílaba mais forte e terminar com "ÉU", ou seja, entra na regra das oxítonas terminadas com "ÉU". Acentua-se a oxítona terminada em: a, e, o, seguidos ou não de s, em, ens (em palavras de duas ou mais sílabas), éis, éu(s), ói (s).

    Portanto, somente a primeira palavra e a segunda palavra contrariam a regra de acentuação e de hifenização respectivamente.

    d) Correta.

    Ra-í-zes⇢ ⇢ acentuada por ser um hiato formado por "I" que fica sozinho na sílaba. Acentua-se o hiato quando formado pelas vogais "I" ou "U" sozinho na sílaba e não seguido de "NH".

    Pa-péis⇢ acentuada por ter a última sílaba mais forte e terminar com "ÉIS", ou seja, entra na regra das oxítonas terminadas com "ÓI". Acentua-se a oxítona terminada em: a, e, o, seguidos ou não de s, em, ens (em palavras de duas ou mais sílabas), éis, éu(s), ói (s).

    A-ve-ri-gue⇢ forma de escrever a conjugação do verbo 'averiguar’ no presente do subjuntivo.

    Portanto, todas as palavras estão corretamente escritas.

    e) Incorreta.

    I-déi-as⇢ não se acentua a penúltima sílaba quando formada por ditongo oral "ÉI" "ÓI".

    Vo-o⇢ essa é a forma correta de escrever voo, pois não leva mais acento dupla vogal.

    Cha-péu⇢ ⇢ acentuada por ter a última sílaba mais forte e terminar com "ÉU", ou seja, entra na regra das oxítonas terminadas com "ÉU". Acentua-se a oxítona terminada em: a, e, o, seguidos ou não de s, em, ens (em palavras de duas ou mais sílabas), éis, éu(s), ói (s).

    Portanto, somente a primeira palavra e a terceira palavra contrariam a regra de acentuação.

    Gabarito do monitor: D


ID
2688763
Banca
FUMARC
Órgão
SEE-MG
Ano
2018
Provas
Disciplina
Matemática
Assuntos

Sabe-se que a massa do Sol é de 1,989 ·1027 t, a massa da Terra é de 5,972 · 1021 t e a massa da Lua é de 7,348 ·1019 t.


Aproximadamente, quantas vezes a massa da Terra é maior que a massa da Lua?

Alternativas
Comentários
  • 597 

    7

  • como a diferença entre a massa da Terra e a massa da Lua é de 10², então comparei as duas medidas, ficando assim:

    597,2 x 10(^19)

    7,348 x 10(^19)

    neste caso, para simplificar, dividi o 597 por 7, chegando num resultado mais próximo, que seria o da alternativa "b".

  • 1° Passo é Retirar as vírgulas:

                             5972 * 10^18 e 7348 *10^16

    2° Passo é Dividir os dois valores:

                             5972* 10^18 / 7348* 10^16=

                          = 0,8127.. *10^2= 82 aproximadamente

    obs: na divisão os expoentes são subtraidos, pegando o exemplo da própria questão, temos: 10^18 - 10^16 = 10^2

  • como a diferenças entre as potencias e 10^2 = 100 ai vc multiplica 6/7 q resultara em numero menor q 100

  • Esta questão envolve notações científicas que nada mais são representações de números muito grandes na forma de potência de base 10.

    Para saber quantas vezes a massa da Terra é maior que a massa da Lua, precisamos dividir uma pela outra:

    n= 5.972 x 10^21

    7.348 x 10^19

    Para dividir estes valores, precisamos dividir primeiro os valores "independentes" (5972 e 7348) e depois dividir as potências. A divisão de potências de mesma base é feita conservando a base e subtraindo os expoentes:

    n= 5.972 x 10^21 = 0,8127 x 10^21-19

    7.348 x 10^19

    n= 0,8127 x 10^2

    n= 81,27

    A massa da Terra é aproximadamente 82 vezes maior que a massa da Lua.

    Resposta: letra D

  • Gente, eu fiz aproximado pra facilitar (visto que na hora da prova não tem tanto tempo)

    A massa da Terra é de 5,972 x 10^21 t = 6 x 10^21

    A massa da Lua é de 7,348 x 10^19 t = 7 x 10^19

    Para passar a massa da lua para a mesma notação: 0,07 x 10^19

    Dividindo: 6 / 0,07 = 85,7.

    Todas as alternativas tem muita diferença numérica, então eu arredondo sem medo. Deixo para fazer conta quando tem alternativas próximas.

    Portanto: 82 vezes maior.


ID
2688766
Banca
FUMARC
Órgão
SEE-MG
Ano
2018
Provas
Disciplina
Matemática
Assuntos

Um determinado medicamento é vendido em cartela com 4, 5 ou 6 comprimidos. O médico receitou a Bernardo 20 comprimidos desse medicamento. De quantas maneiras Bernardo pode comprar exatamente 20 comprimidos?

Alternativas
Comentários
  • Pensei dessa forma:

    4 caixas de 5: 5-5-5-5

    5 caixas de 4: 4-4-4-4-4

    2 caixas de 4 e 2 de 6: 4-4-6-6

    2 de 5, 1 de 4 e 1 de 6: 5-5-4-6

  • MMC: 4 ,5, 6 = 60

    SOMA AS POSSIBILIDADES DAS CARTELAS: 4, 5, 6 = 15

    DIVIDE: 60/15 = 4

  • Por que devo somar as possibilidades da cartela, Helda? Qual sentido?

  • Será mesmo que pode fazer isso, Helda? Não foi coincidência, será? Se ele pedisse exatamente 30 comprimidos, por exemplo, como ficaria então?

  • Discordo do gabarito, visto que posso comprar 4 cartelas com 5 comprimidos e ainda 5 cartelas com 4 comprimidos. Seriam então 5 possibilidades.

  • https://www.youtube.com/watch?v=evnCS47y2qY

  • mdc = 5

    divide pelo numero de comprimidos 20

    r = 4

     

     

  • EU multipliquei e fui dividindo pelas possibilidades 120/6 = 20 errei bonito.

  • Helda, comentário esclarecedor! Obrigada.

  • não entende muita essa resposta da Helda, essa possibilidade dela se aplica a outra tipo de questão parecida?

  • eu fiz assim:

    Pensei: eu tenho que comprimidos 20 comprimidos no total e eles so vender em cartelas com 4,5 e 6 comprimidos.

    entao se,,,

    opçao 1

    posso comprar 4 cartelas com 5 comprimidos cada `= 4 x 5= 20

    opçao 2

    posso comprar 5cartelas com 4 comprimidos cada = 5 x 4 = 20

    opçao 3

    posso comprar 2 cartelas com 5 comprimidos 2 x 5= 10

    + 1 cartela com 6 comprimidos 1 x 6 = 6

    + 1 cartea com 4 comprimidos 1 x 4 = 4

    total = 10+6+4 = 20

    opçao 4

    posso comprar 2 cartelas com 6 comprimidos 2 x 6 = 12

    + 2 com 4 comprimidos = 2 x 4 =8

    total 12+8 =20

    eu consegui assim, so nao sei se é a mais correta ou mais facil

  • O que a Helda respondeu não faz o melhor sentido. Onde entra a parte que a soma tem que dá 20 nas contas dela?

  • Eu tirei o MMC de 4,5,6 que deu 60, e dividi por 20 (que é o número de comprimidos) o resultado foi 3. Nao entendi o gabarito ser 4.

  • EU FIZ MMC... DEU 60 E DEPOIS DIVIDIR PELA SOMA DOS FATORES 4,5,OU 6 FORMAS DE TOMAR COMPRIMIDOS AÍ DEU 60/15 = 4

  • 4 MANEIRAS

    PODERIA COMPRAR :

    1ª 5 CARTELAS DE 4 COMPRIMIDOS 

    2ª 4 CARTELAS DE 5 COMPRIMIDOS 

    3ª 2 CARTELAS DE 4 COMPRIMIDOS E 2 CARTELAS DE 6 COMPRIMIDOS

    4ª 1 CARTELAS DE 4 COMPRIMIDOS, 2 CARTELAS DE 5 COMPRIMIDOS  E 1 CARTELA DE 6 COMPRIMIDOS

  • A resolução da Helda foi apenas uma coincidência, pessoal. Cuidado na hora de curtirem comentários sem entenderem se está realmente certo.

    Se pegarmos um outro exemplo, veremos que o cálculo que ela usou não se aplica e que foi apenas uma coincidência. E isso acontece em muitas questões por aqui, onde as pessoas não sabem resolver da forma correta e criam formas para chegar no valor do gabarito.

    Veja o exemplo:

    Aqui as cartelas tem 4, 5 e 10 comprimidos. Queremos que eles somem 20 comprimidos.

    Vejamos quais opções temos:

    1ª opção: 5+5+10=20

    2ª opção: 10+10=20

    3ª opção: 5+5+5+5=20

    4ª opção: 4+4+4+4+4=20

    Portanto, temos 4 opções.

    O MMC entre 4, 5 e 10 é 20. A soma de 4, 5 e 10 é 19. Portanto, seguindo o cálculo da Helda, teríamos 20/19=1,05, que nem mesmo é um número inteiro.

    Observem que tivemos 4 soluções e não apenas uma, comprovando que foi apenas uma coincidência.

    Na dúvida, sempre testem outros valores para conferir se a resolução da pessoa se aplica.


ID
2688781
Banca
FUMARC
Órgão
SEE-MG
Ano
2018
Provas
Disciplina
Matemática
Assuntos

Um trem faz uma viagem de 279 quilômetros a uma velocidade constante de 54 km/h, sem paradas. Qual o tempo gasto para essa viagem?

Alternativas
Comentários
  • Em caso de dúvida quanto à transformação do decimal em tempo, traduza os valores para m/s.

     

    279 km = 279.000 m

    54 km/h ---> /3,6 ---> 15 m/s

     

    d = v.t --> 279.000 = 15 . t --> t = 18.600s

     

    60 s ---- 1 min

    18.600 s ---- x

    x = 310 min

     

    300 min = 5h ----> 310 min = 5h e 10 min

     

  • Gab: D

    Distância: 279km 
    Velocidade: 54km/h

    279/54 = 5,16666... horas (mas isso não significa 5h e 16min e sim 05h e 0,1666666...h)

    1h = 60min
    0,16h = x

    x=60*0,16
    x=9,60 min (aproximadamente 10min - se fizer o cálculo com todos os 6 necessários dá10)

    Resposta: 5h e 10min


     


ID
2688790
Banca
FUMARC
Órgão
SEE-MG
Ano
2018
Provas
Disciplina
Matemática
Assuntos

Um avião fez uma viagem de 3h e 30min com 285 passageiros. A companhia estimou que o custo operacional total dessa viagem foi de R$ 596.534,00.


Aproximadamente, qual o custo médio por hora viajada para cada passageiro?

Alternativas
Comentários
  • Primeiro, encontramos o gasto total para 1 hora. Depois, dividimos esse gasto entre os passageiros.

     

    ~ 600.000 reais ------ 3,5h

    x ----------------------- 1h

     

    x = ~ 171.000

     

    -----> 171.000/285 = ~600

     

  • Consegui assim:

    S285 = 3.5 logo a soma é = 997,50

    Valor total do gasto dividido pela a média da soma.

    596.534 / 997,50 = 598

     


ID
2688796
Banca
FUMARC
Órgão
SEE-MG
Ano
2018
Provas
Disciplina
Pedagogia
Assuntos

Segundo a Convenção da Organização da Nações Unidas – (ONU) sobre a Eliminação de Todas as Formas de Discriminação Racial (1966), os Estados Parte condenam a discriminação racial e comprometem-se a adotar uma política de eliminação da discriminação racial em todas as suas formas e de promoção de entendimento entre todas as raças. Para esse fim, cada Estado Parte compromete-se a não efetuar ato ou prática de discriminação racial praticada por uma pessoa ou organização qualquer, a tomar as medidas eficazes, a fim de rever as políticas governamentais nacionais e locais e para modificar, ab-rogar ou anular qualquer disposição regulamentar que tenha como objetivo criar a discriminação ou perpetrá-la onde já existir; a adotar as medidas legislativas, proibir e pôr fim à discriminação racial praticada por pessoas, por grupos ou organizações; favorecer, quando for o caso, as organizações e movimentos multirraciais e outros meios próprios e eliminar as barreiras entre as raças e desencorajar o que tende a fortalecer a divisão racial. Os Estados Parte comprometem-se a proibir e a eliminar a discriminação racial em todas suas formas e a garantir o direito de cada um à igualdade perante a lei, sem distinção de raça, de cor ou de origem nacional ou étnica.

(Fonte: Convenção Internacional sobre a Eliminação de Todas as Formas de Discriminação Racial. ONU,1998. p. 02-03. Disponível em: http://unesdoc.unesco.org/images/0013/001393/139390 por.pdf> Acesso 26 fev 2018).


Assim, a todos os cidadãos, deve ser garantido:


I. Direito de um tratamento diferenciado, perante os tribunais ou outro órgão que administre justiça; direito à segurança ou à proteção do Estado, contra violência ou lesão corporal cometida, seja por funcionários de Governo, seja por qualquer indivíduo, grupo ou instituição.

II. Direito de participar das eleições - votar e ser votado - de acordo com o sistema de sufrágio universal e igual direito de tomar parte no Governo, assim como na direção de dois assuntos públicos, em qualquer grau, e o direito de acesso, em igualdade de condições, às funções públicas.

III. Direito de circular parcialmente e de escolher residência dentro das fronteiras do Estado; direito de deixar qualquer país, inclusive o seu, e de retornar a seu país.

IV. Direito a uma nacionalidade; direito de casar-se e escolher o cônjuge; direito tanto individual como coletivo à propriedade.

V. Direito ao trabalho, à escolha do trabalho, a condições equivalentes e satisfatórias de trabalho, à proteção contra o desemprego, a um salário igual para um trabalho igual e a uma remuneração equitativa e satisfatória.


Está CORRETO apenas o que se afirma em:

Alternativas
Comentários
  • GABARITO: letra D

    Conforme ARTIGO V da Convenção Internacional sobre a Eliminação de Todas as Formas de Discriminação Racial:

    I - ERRADO: "a) direito de recorrer a um tribunal ou a qualquer outro órgão de administração da justiça (sem tratamento diferenciado, mas igualitário);

    b) direito à segurança da pessoa e à proteção do Estado contra violência ou lesão corporal cometida por funcionários do Governo ou por qualquer pessoa, grupo ou instituição;"

    II - CERTO: "c) direitos políticos, especialmente o de participar de eleições - votando e sendo votado - através de sufrágio universal e igual, direito de tomar parte no governo assim como na direção dos assuntos públicos em todos os escalões, e direito de ter acesso em igualdade de condições às funções públicas;"

    III - ERRADO: "d)... (i) direito de circular livremente e de escolher sua residência no interior de um Estado;
    (ii) direito de deixar qualquer país, inclusive o seu, e de regressar ao mesmo;"

    IV - CERTO: "d)... (iii) direito a uma nacionalidade;
    (iv) direito ao casamento e à escolha do cônjuge;
    (v) direito de qualquer pessoa, tanto individualmente como em associação com outras, à propriedade;"

    V - CERTO: "e)... (i) direitos ao trabalho, à livre escolha do trabalho, a condições equitativas e satisfatórias de trabalho, à proteção contra o desemprego, a um salário igual para um trabalho igual, a uma remuneração eqüitativa e satisfatória;"


ID
2688799
Banca
FUMARC
Órgão
SEE-MG
Ano
2018
Provas
Disciplina
Pedagogia
Assuntos

A obrigatoriedade de inclusão da História e Cultura Afro-Brasileira e Africana nos currículos da Educação Básica trata-se de decisão política, com fortes repercussões pedagógicas, inclusive na formação de professores. Com esta medida, reconhece-se que, além de garantir vagas para negros nos bancos escolares, é preciso valorizar devidamente a história e a cultura de seu povo, buscando reparar danos, que se repetem há cinco séculos, à sua identidade e a seus direitos. A relevância do estudo de temas decorrentes da história e da cultura afro-brasileira e africana não se restringe à população negra, ao contrário, diz respeito a todos os brasileiros, uma vez que devem educar-se enquanto cidadãos atuantes no seio de uma sociedade multicultural e pluriétnica, capazes de construir uma nação democrática.

(Fonte: Resolução Nº 1, de 17 de junho de 2004. Institui Diretrizes Curriculares Nacionais para a Educação das Relações Étnico-Raciais e para o Ensino de História e Cultura Afro-Brasileira e Africana. p.17. Disponível em: http://portal.mec.gov.br/cne/arquivos/pdf/res012004.pdf> Acesso 26 fev 2018).


É urgente que professores, gestores e todos os que fazem parte do contexto escolar tenham, em sua formação, condições para o trabalho pedagógico com toda e qualquer expressão de diversidade cultural. Cabe a todos que educam exigir a educação para a diversidade, reconhecendo as alteridades presentes nas relações étnico-raciais, além de pensar nos grupos que foram e são excluídos da sociedade. Sabendo desses princípios, é CORRETO afirmar: 

Alternativas
Comentários
  • Gabarito: (c)

  • Discordo que o "discurso histórico carregado de estereótipo" tenha criado a desigualdade entre negros e branco. Esse discurso pode ter agravado ou servido de instrumento de manutenção dessa desigualdade... 

  • Achei errada a questão C

  • Daniel Dias concordo com vc

  • Não foi o discurso histórico que criou a desigualdade, mas o próprio sistema escravista que criou a desigualdade entre negros e brancos, visto que decorrido desse sistema, originou-se, mais tarde, o discurso racista.

    Outro ponto controverso da alternativa: não havia a necessidade de colonizar os negros, muito pelo contrário, havia a necessidade de ESCRAVIZÁ-LOS. Havia a necessidade de colonizar os territórios e não os negros.

    Devido as contradições apresentada pela alternativa, deveria ser anulada.

    INFELIZMENTE, GABARITO C


ID
2688802
Banca
FUMARC
Órgão
SEE-MG
Ano
2018
Provas
Disciplina
Pedagogia

Arroyo, ao discorrer sobre a política da Educação Integral e Integrada, explica que uma visão negativa persistente na escola e na gestão escolar ainda precisa ser superada. Ele aponta que um risco que a Educação Integral e Integrada corre é o de perder seu significado político ao se limitar a uma oferta de “mais tempo da mesma escola, ou mais um turno – turno extra – ou mais educação do mesmo tipo de educação”. (Fonte: MOLL. Jaqueline. [et al]. Caminhos da Educação Integral no Brasil. [recurso eletrônico]: direito a outros tempos e espaços educativos. Porto Alegre: Penso, 2012, p. 33).


Sobre a política de Educação Integral e Integrada, analise as asserções a seguir:


I. A Educação Integral e Integrada na Educação Básica assegura jornada escolar igual ou superior a sete horas diárias ou trinta e cinco semanais, durante o período letivo.

II. O decreto reconhece e valoriza a diversidade das populações do campo, quilombola, indígena e situação de itinerância e estimula a gestão democrática e a articulação entre a educação básica e o ensino superior.

III. São princípios da Educação Integral e Integrada: igualdade de condições para o acesso e permanência na escola; valorização do profissional da educação; vinculação entre a educação escolar, trabalho e práticas sociais; singularismo de ideias e de concepções pedagógicas.

IV. Um dos objetivos da Educação Integral é fortalecer a rede de educação profissional, com vistas ao aumento da escolarização e à melhoria da qualidade da formação do jovem e adulto trabalhador, tendo como centralidade o estudante, considerando como dimensões indissociáveis o trabalho, a ciência, a cultura e a tecnologia.

V. É competência da Secretaria Municipal de Educação tomar as providências para a ampliação gradativa da Educação Integral e Integrada na rede de ensino, considerando as metas estabelecidas no Plano Nacional de Educação e nos demais instrumentos legais.


Está CORRETO apenas o que se afirma em:

Alternativas

ID
2688805
Banca
FUMARC
Órgão
SEE-MG
Ano
2018
Provas
Disciplina
Pedagogia
Assuntos

Numere a Coluna 2 conforme a Coluna 1, levando em consideração os recursos de acessibilidade que o estudante com Necessidades Educacionais Especiais precisa ter, com a adaptação no seu currículo, garantindo-lhe o direito à aprendizagem e efetividade na vida escolar.


Coluna 1

Aluno com NEE

1 Aluno Surdo

2 Aluno Cego

3 Aluno Superdotado

4 Aluno com Transtorno do Espectro Autista

5 Aluno com Deficiência Intelectual

6 Aluno com Paralisia Cerebral


Coluna 2

Recursos necessários

( ) Rotina diária organizada, agenda com esquema de aulas com símbolos ou desenhos, ordens claras e cartões de comunicação.

( ) Teclados especiais, programas para uso no computador (como instrumento para escrever), tesoura adaptada, engrossadores e material pedagógico adaptado.

( ) Material pedagógico adaptado para facilitar sua compreensão, material concreto para auxiliar na matemática, adaptações curriculares nas provas e no material da aula.

( ) Uso de tecnologias computacionais: softwares educativos, enciclopédias digitais, jogos pedagógicos e simuladores.

( ) Aprendizagem do Português como segunda língua e aprendizagem da LIBRAS como língua materna para aperfeiçoar.

( ) Recursos didáticos em alto relevo, reglete e punção, recursos ópticos e programas leitores de tela


A sequência CORRETA, de cima para baixo, é:

Alternativas
Comentários
  • Coluna 1

    Aluno com NEE

    1 Aluno Surdo

    2 Aluno Cego

    3 Aluno Superdotado

    4 Aluno com Transtorno do Espectro Autista

    5 Aluno com Deficiência Intelectual

    6 Aluno com Paralisia Cerebral

    Coluna 2

    Recursos necessários

    ( 4) Rotina diária organizada, agenda com esquema de aulas com símbolos ou desenhos, ordens claras e cartões de comunicação.

    ( 6) Teclados especiais, programas para uso no computador (como instrumento para escrever), tesoura adaptada, engrossadores e material pedagógico adaptado.

    ( 5) Material pedagógico adaptado para facilitar sua compreensão, material concreto para auxiliar na matemática, adaptações curriculares nas provas e no material da aula.

    ( 3) Uso de tecnologias computacionais: softwares educativos, enciclopédias digitais, jogos pedagógicos e simuladores.

    ( 1) Aprendizagem do Português como segunda língua e aprendizagem da LIBRAS como língua materna para aperfeiçoar.

    ( 2) Recursos didáticos em alto relevo, reglete e punção, recursos ópticos e programas leitores de tela

  • Aluno com Transtorno do Espectro Autista: Rotina diária organizada, agenda com esquema de aulas com símbolos ou desenhos, ordens claras e cartões de comunicação.

    Aluno com Paralisia Cerebral: Teclados especiais, programas para uso no computador (como instrumento para escrever), tesoura adaptada, engrossadores e material pedagógico adaptado.

    Aluno com Deficiência Intelectual: Material pedagógico adaptado para facilitar sua compreensão, material concreto para auxiliar na matemática, adaptações curriculares nas provas e no material da aula.

    Aluno Superdotado: Uso de tecnologias computacionais: softwares educativos, enciclopédias digitais, jogos pedagógicos e simuladores.

    Aluno Surdo: Aprendizagem do Português como segunda língua e aprendizagem da LIBRAS como língua materna para aperfeiçoar.

    Aluno Cego: Recursos didáticos em alto relevo, reglete e punção, recursos ópticos e programas leitores de tela.

    Bons estudos!!!

    Se estiver com sono, lembre-se que nosso maior concorrente, somos nos mesmos.....cansaço,desânimo, pensamentos aleatórios....porém o desejo de vencer prevalece.


ID
2688808
Banca
FUMARC
Órgão
SEE-MG
Ano
2018
Provas
Disciplina
Pedagogia
Assuntos

De acordo com Vasconcelos (2005), Projeto Político-Pedagógico é o plano global da instituição. Pode ser entendido como a sistematização, nunca definitiva, de um processo de planejamento participativo, que se aperfeiçoa e se concretiza na caminhada, que define claramente o tipo de ação educativa que se quer realizar. É um instrumento teórico-metodológico para a intervenção e mudança da realidade. É um elemento de organização e integração da atividade prática da instituição nesse processo de transformação.

(Fonte: VASCONCELLOS, Celso dos S. Planejamento: projeto de ensino–aprendizagem e projeto político-pedagógico. São Paulo, Libertad Editora, 2005. Coleção Cadernos Pedagógicos).


Diante do texto, analise as asserções a seguir:


I. O Projeto Político-Pedagógico está relacionado com a organização do trabalho pedagógico em dois níveis: na organização da escola como um todo e na organização da sala de aula, incluindo sua relação com o contexto social imediato, procurando preservar a visão de totalidade.


PORQUE


II. O Projeto Político-Pedagógico, sendo a sistematização de um processo de planejamento participativo, substitui o Regimento Escolar e dá o devido suporte para a elaboração dos planos de ensino e dos planos de aula.


Está CORRETO o que se afirma em:

Alternativas
Comentários
  • Eu agredito que o erro II está na palavra "substitui "

  • Projeto tem prazo detrminado, regimento não. Um não substiui o outro, inclusive não pode haver contradição entre eles. O regimento é mais estatico por ser uma " especie " de lei da escola, regra de conduta. O projeto é mais dinamico pois vai e adaptando ao logo do ano...

  • O Regimento da Escola, complementa o PPP, em hipotese alguma pode substituir...

    PPP---> Identidade da escola

    Regimento Escolar---> São as regras escolares.

  • EI é uma proposição verdadeira e II é uma proposição falsa


ID
2688811
Banca
FUMARC
Órgão
SEE-MG
Ano
2018
Provas
Disciplina
Pedagogia
Assuntos

O Projeto Político-Pedagógico de cada unidade de ensino deve ser elaborado e atualizado em conformidade com a legislação, assegurada a participação de todos os segmentos representativos da escola, com assessoramento do Serviço de Inspeção Escolar e Equipes Pedagógicas Central e Regional, e aprovado pelo Colegiado de cada escola, implementado e amplamente divulgado na comunidade escolar.

(Fonte: MINAS GERAIS. Secretaria de Estado de Educação. Resolução 2197/2012. Dispõe sobre a organização e o funcionamento do ensino nas Escolas Estaduais de Educação Básica de Minas Gerais. Belo Horizonte, 2012.)


Considerando o tema abordado e o contexto em que se insere, é CORRETO afirmar:

Alternativas
Comentários
  • b) O Plano de Intervenção Pedagógica elaborado, anualmente, pela equipe Pedagógica da Escola é parte integrante do Projeto Político-Pedagógico da Escola.


ID
2688814
Banca
FUMARC
Órgão
SEE-MG
Ano
2018
Provas
Disciplina
Pedagogia
Assuntos

A organização e o funcionamento do ensino nas Escolas Estaduais de Educação Básica de Minas Gerais, regulamentada pela Resolução n. 2.197/2012, estabelece princípios éticos, políticos e estéticos que deverão ser adotados para nortear as ações pedagógicas nelas desenvolvidas.


Sobre o processo de avaliação, esse documento estabelece:


I. A avaliação da aprendizagem dos estudantes será realizada pelos professores, em conjunto com toda equipe pedagógica da escola, e deverá assumir um caráter processual, formativo e participativo, prevalecendo os aspectos quantitativos do aprendizado do estudante sobre os qualitativos.

II. A avaliação da aprendizagem deverá prover, obrigatoriamente, intervenções pedagógicas, ao longo do ano letivo, para garantir a aprendizagem no tempo certo, e assegurar tempos e espaços diversos para aqueles com menor rendimento, para que tenham condição de ser devidamente atendido.

III. A avaliação da aprendizagem dos estudantes será contínua, cumulativa, diagnóstica e possibilitar a aceleração de estudos para aqueles com distorção idade-série e para aqueles que tiverem frequência superior a 75%, no final do ano letivo.

IV. As formas e procedimentos utilizados pela escola para diagnosticar, acompanhar e intervir pedagogicamente no processo de aprendizagem dos estudantes devem expressar, com clareza, o que é esperado deles, em relação a sua aprendizagem e ao que foi realizado pela escola, devendo ser registrado para subsidiar as decisões e informações sobre sua vida escolar.

V. No caso de desempenho satisfatório dos estudantes e de frequência inferior a 75%, no final do período letivo, a escola deve usar o recurso de reclassificação para posicionar o aluno no ano seguinte de seu percurso escolar.


Está CORRETO apenas o que se afirma em:

Alternativas

ID
2688817
Banca
FUMARC
Órgão
SEE-MG
Ano
2018
Provas
Disciplina
Pedagogia
Assuntos

“A Base Nacional Comum Curricular (BNCC) é um documento de caráter normativo que define o conjunto orgânico e progressivo de aprendizagens essenciais que todos os alunos devem desenvolver ao longo das etapas e modalidades da Educação Básica, de modo a que tenham assegurados seus direitos de aprendizagem e desenvolvimento, em conformidade com o que preceitua o Plano Nacional de Educação (PNE)”. (Fonte: BRASIL, 2017, p. 7).


Considerando a concepção presente no texto, analise as afirmativas a seguir:


I. A BNCC reconhece que a Educação Básica deve visar à formação e ao desenvolvimento humano global, o que implica compreender que esse desenvolvimento é linear.

II. A dimensão conceitual da BNCC permite que os estudantes desenvolvam aproximações e compreensões sobre os saberes científicos e os presentes nas situações cotidianas.

III. A noção de competência é definida na BNCC como a mobilização de conhecimentos, habilidades, atitudes e valores para resolver demandas complexas da vida cotidiana, do pleno exercício da cidadania e do mundo do trabalho.

IV. Ao dizer que os conteúdos curriculares estão a serviço do desenvolvimento de competências, a LDBEN orienta a definição das aprendizagens dos conteúdos mínimos a serem ensinados na proposta da BNCC.


Está CORRETO apenas o que se afirma em:

Alternativas
Comentários
  • II. A dimensão conceitual da BNCC permite que os estudantes desenvolvam aproximações e compreensões sobre os saberes científicos e os presentes nas situações cotidianas.

    III. A noção de competência é definida na BNCC como a mobilização de conhecimentos, habilidades, atitudes e valores para resolver demandas complexas da vida cotidiana, do pleno exercício da cidadania e do mundo do trabalho.

    Resposta E

  • I - O erro está em dizer que o conhecimento é linear

    IV - LDBEN o normatiza, quem define os conteúdos é o PNE

  • I - "Educação Básica deve visar à formação e ao desenvolvimento humano global, o que implica compreender a complexidade e a não linearidade desse desenvolvimento, rompendo com visões reducionistas..."

    IV- "Ao dizer que os conteúdos curriculares estão a serviço do desenvolvimento de competências, a LDB orienta a definição das aprendizagens essenciais, e não apenas dos conteúdos mínimos a ser ensinados. Essas são duas noções fundantes da BNCC"

  • A BNCC define conteúdos a serem estudados e competências e habilidades que os alunos devem demonstrar a cada passo da vida escolar.

  • I - Não existe conhecimento linear

    IV - A BNCC ( o próprio nome já diz: BASE NACIONAL COMUM CURRICULAR) orienta os conteúdos mínimos a serem estudados, a LDB somente normatiza.

    Gabarito E

  • I- (…) a Educação Básica deve visar à formação e ao desenvolvimento humano global, o que implica compreender a complexidade e a não linearidade desse desenvolvimento, rompendo com visões reducionistas que privilegiam ou a dimensão intelectual (cognitiva) ou a dimensão afetiva. (BNCC, 2018, pág. 14) 

    IV- Quem orientam e a LDB e as DCN.

  • Vocês arrasam na explicação !!!!


ID
2688820
Banca
FUMARC
Órgão
SEE-MG
Ano
2018
Provas
Disciplina
Pedagogia
Assuntos

O Atendimento Educacional Especializado (AEE) é o conjunto de atividades e recursos de acessibilidade, com objetivos pedagógicos, organizados institucional e continuamente para atender aos estudantes com algum tipo de necessidade especial escolar.


Quanto ao AEE (Atendimento Educacional Especializado), analise as afirmativas a seguir e identifique-as com (V) ou (F) conforme sejam verdadeiras ou falsas.


( ) O Atendimento Educacional Especializado deve integrar a proposta pedagógica da escola, envolver a participação da família para garantir pleno acesso e participação dos estudantes, atender às necessidades específicas do público-alvo da educação especial e ser realizado em articulação com as demais políticas públicas.

( ) A educação especial deve garantir os serviços de apoio especializado voltados a eliminar as barreiras que possam obstruir o processo de escolarização de estudantes com deficiência, mas não dos transtornos globais do desenvolvimento e altas habilidades ou superdotação.

( ) O Atendimento Educacional Especializado para o aluno com deficiência intelectual deve permitir que esse aluno saia de uma posição de “não saber”, para se apropriar de um saber que lhe é próprio, ou melhor, que ele tem consciência de que o construiu.

( ) O Atendimento Educacional Especializado existe para que os alunos possam aprender o que é diferente dos conteúdos curriculares do ensino comum, exceto o que é necessário para que possam ultrapassar as barreiras impostas pela deficiência.


A sequência CORRETA, de cima para baixo, é:

Alternativas
Comentários
  • a) V, F, V, F.

  • Questão bem elaborada..

    Caso fique em dúvida assim como eu, segue a estratégia da eliminação, sempre funciona.

    Bons estudos, nossa vez estar chegando!!!!


ID
2688823
Banca
FUMARC
Órgão
SEE-MG
Ano
2018
Provas
Disciplina
Português
Assuntos

TEXTO I:


                                 Nossa Senhora dos Destoantes

                                                                                    Luís Fernando Veríssimo 


      A pequena Capela de Nossa Senhora do Rosário do Padre Faria é uma das tantas joias arquitetônicas de Ouro Preto. O exterior despojado não prepara o visitante para a opulência barroca do interior. O campanário fica afastado do corpo da igreja, como a “casinha” numa morada sem banheiro, e nada tem de imponente. Os sinos da Capela de Padre Faria badalam em concerto com os outros sinos da região, cantando as horas e os eventos, e não soam nem melhor nem pior do que os outros. Mas os sinos da Capela do Padre Faria têm uma história diferente dos outros.

      Quando Tiradentes foi enforcado e esquartejado no Rio de Janeiro todos os outros sinos celebraram a notícia. Afinal, tratava-se da execução de um traidor, de um inimigo da sociedade. Os sinos de Ouro Preto festejaram o castigo exemplar de um réprobo e o triunfo da legalidade sobre a rebeldia. Mesmo que o toque festivo não tivesse sido recomendado pela Coroa, a celebração se justificaria. Mas os sinos da Capela do Padre Faria dobraram Finados. Pela primeira e única vez na história, talvez, os sinos da Capela do Padre Faria destoaram do concerto. Tocaram, sozinhos, uma batida fúnebre pelo martírio de Tiradentes.

      Não conheço bem a história e não sei o que motivou as badaladas subversivas. Um pedido de secretos simpatizantes da Inconfidência? Apenas uma manifestação de piedade cristã? Um sineiro bêbado? Não sei. Minha tese preferida é que alguém responsável pelos sinos teve um vislumbre histórico. Teve a presciência que ninguém mais teve e ordenou o toque plangente, em homenagem precoce ao futuro herói e pelo ocaso do poder colonial que seu sacrifício desencadearia.

      Nossa Senhora do Rosário serviria como padroeira, não necessariamente de quem consegue adivinhar a História, mas de quem entende o momento que está vivendo ou destoa da maioria, com ou sem razão. Destoantes deveriam ir regularmente em romaria à pequena capela e pedir a bênção dessa Nossa Senhora do Contexto Maior, para melhor poder enfrentar a maioria que badala o que não tem importância e o fato errado e menospreza qualquer batida diferente.

      Os outros sineiros de Ouro Preto não tinham como saber que estavam festejando a morte de um herói. Faltava-lhes a perspectiva histórica para entender o momento e só cumpriram o que se esperava deles. Estão perdoados. Mas que nos sirvam de lição.

(Disponível em https://jornalggn.com.br/noticia/nossa-senhora-dos-destoantes-por-luis-fernando-verissimo. Acesso em: 9 fev. 2018)

“É curioso como elas [as crônicas] mantêm o ar despreocupado, de quem está falando de coisas sem maior consequência e, no entanto, não apenas entram fundo no significado dos atos e sentimentos do homem, mas podem levar longe a crítica social”.

CÂNDIDO, Antônio. Para gostar de ler: crônicas. Volume 5. São Paulo: Ática, 2003. pp.89-99.


O trecho em destaque faz parte do texto de Antônio Cândido, A vida ao rés do chão, no qual o autor discorre sobre o gênero crônica. A produção de Luís Fernando Veríssimo ilustra as ideias de Antônio Cândido pelo fato de que

Alternativas
Comentários
  • Questão de interpretação mesmo. Note que os sinos mostram para um acontecimento de injustiça do passado que sequer era percebido devido a falta de sensibilidade das pessoas à época.

  • Gabarito Letra A

     

    Bem que a Fumar poderia numerar as linhas.

     

    Comprovamos a letra A nas passagens:

     

    Linha 10 do texto (...) Tocaram sozinhos (os sinos), uma batida fúnebre pelo martírio de Tiradentes (...)

     

    Linha 18/19 (...) Os outros sineiros de Ouro Preto não tinham como saber que estavam festejando a morte de um herói. Faltava-lhes a perspectiva histórica para entender o momento e só cumpriram o que se esperava deles. 

  • As questões de português dessa prova estão muito bem elaboradas, fugindo à linha tradicional da banca, que é fazer questões horríveis, extremamente subjetivas e pessimamente formuladas.

    Espero que a PC siga essa linha.


ID
2688826
Banca
FUMARC
Órgão
SEE-MG
Ano
2018
Provas
Disciplina
Português
Assuntos

TEXTO I:


                                 Nossa Senhora dos Destoantes

                                                                                    Luís Fernando Veríssimo 


      A pequena Capela de Nossa Senhora do Rosário do Padre Faria é uma das tantas joias arquitetônicas de Ouro Preto. O exterior despojado não prepara o visitante para a opulência barroca do interior. O campanário fica afastado do corpo da igreja, como a “casinha” numa morada sem banheiro, e nada tem de imponente. Os sinos da Capela de Padre Faria badalam em concerto com os outros sinos da região, cantando as horas e os eventos, e não soam nem melhor nem pior do que os outros. Mas os sinos da Capela do Padre Faria têm uma história diferente dos outros.

      Quando Tiradentes foi enforcado e esquartejado no Rio de Janeiro todos os outros sinos celebraram a notícia. Afinal, tratava-se da execução de um traidor, de um inimigo da sociedade. Os sinos de Ouro Preto festejaram o castigo exemplar de um réprobo e o triunfo da legalidade sobre a rebeldia. Mesmo que o toque festivo não tivesse sido recomendado pela Coroa, a celebração se justificaria. Mas os sinos da Capela do Padre Faria dobraram Finados. Pela primeira e única vez na história, talvez, os sinos da Capela do Padre Faria destoaram do concerto. Tocaram, sozinhos, uma batida fúnebre pelo martírio de Tiradentes.

      Não conheço bem a história e não sei o que motivou as badaladas subversivas. Um pedido de secretos simpatizantes da Inconfidência? Apenas uma manifestação de piedade cristã? Um sineiro bêbado? Não sei. Minha tese preferida é que alguém responsável pelos sinos teve um vislumbre histórico. Teve a presciência que ninguém mais teve e ordenou o toque plangente, em homenagem precoce ao futuro herói e pelo ocaso do poder colonial que seu sacrifício desencadearia.

      Nossa Senhora do Rosário serviria como padroeira, não necessariamente de quem consegue adivinhar a História, mas de quem entende o momento que está vivendo ou destoa da maioria, com ou sem razão. Destoantes deveriam ir regularmente em romaria à pequena capela e pedir a bênção dessa Nossa Senhora do Contexto Maior, para melhor poder enfrentar a maioria que badala o que não tem importância e o fato errado e menospreza qualquer batida diferente.

      Os outros sineiros de Ouro Preto não tinham como saber que estavam festejando a morte de um herói. Faltava-lhes a perspectiva histórica para entender o momento e só cumpriram o que se esperava deles. Estão perdoados. Mas que nos sirvam de lição.

(Disponível em https://jornalggn.com.br/noticia/nossa-senhora-dos-destoantes-por-luis-fernando-verissimo. Acesso em: 9 fev. 2018)

No título da crônica – Nossa Senhora dos Destoantes – a palavra “destoantes” se constitui como uma pungente metáfora que só NÃO pode ser relacionada

Alternativas
Comentários
  • Destoantes = Sinos da Capela do Padre Faria que tocaram, sozinhos, em homenagem precoce ao futuro herói - Tiradentes -. Ou seja, os outros sineiros de Ouro Preto não tinham como saber que estavam festejando a morte de um herói. O responsável pelos sinos da capela foi o único que teve um vislumbre histórico de Tiradentes. Teve a presciência que ninguém mais teve e ordenou o toque plangente, em homenagem precoce ao futuro herói e pelo ocaso do poder colonial que seu sacrifício desencadearia.

    GAB: D

  • metáfora

    designação de um objeto ou qualidade mediante uma palavra que designa outro objeto ou qualidade que tem com o primeiro uma relação de semelhança (p.ex., ele tem uma vontade de ferro, para designar uma vontade forte, como o ferro)

  • Achei difícil essa questão.

  • Errei mas....analisando a resposta de Fernando a luz do texto... a resposta está coerente D)


    pois o único que não poderia tocar seria aqueles que não faziam ideia da inocência..

    pois quem tocou tinha o vislumbre histórico...



  • Errei mas....analisando a resposta de Fernando a luz do texto... a resposta está coerente D)


    pois o único que não poderia tocar seria aqueles que não faziam ideia da inocência..

    pois quem tocou tinha o vislumbre histórico...



  • fui na ovelha negra.

  • Se você acertou essa, ta na hora de passar!

  • acertei essa, mas achei bem difícil...fui por eliminação. A única q não tinha muito haver com as outras era a alternativa D.

  • Seria pedir demais que toda questão de interpretação da FUMARC fosse coerente como essa?

  • eu acertei mas achei muito dificil

  • Misericórdia


ID
2688829
Banca
FUMARC
Órgão
SEE-MG
Ano
2018
Provas
Disciplina
Português
Assuntos

TEXTO I:


                                 Nossa Senhora dos Destoantes

                                                                                    Luís Fernando Veríssimo 


      A pequena Capela de Nossa Senhora do Rosário do Padre Faria é uma das tantas joias arquitetônicas de Ouro Preto. O exterior despojado não prepara o visitante para a opulência barroca do interior. O campanário fica afastado do corpo da igreja, como a “casinha” numa morada sem banheiro, e nada tem de imponente. Os sinos da Capela de Padre Faria badalam em concerto com os outros sinos da região, cantando as horas e os eventos, e não soam nem melhor nem pior do que os outros. Mas os sinos da Capela do Padre Faria têm uma história diferente dos outros.

      Quando Tiradentes foi enforcado e esquartejado no Rio de Janeiro todos os outros sinos celebraram a notícia. Afinal, tratava-se da execução de um traidor, de um inimigo da sociedade. Os sinos de Ouro Preto festejaram o castigo exemplar de um réprobo e o triunfo da legalidade sobre a rebeldia. Mesmo que o toque festivo não tivesse sido recomendado pela Coroa, a celebração se justificaria. Mas os sinos da Capela do Padre Faria dobraram Finados. Pela primeira e única vez na história, talvez, os sinos da Capela do Padre Faria destoaram do concerto. Tocaram, sozinhos, uma batida fúnebre pelo martírio de Tiradentes.

      Não conheço bem a história e não sei o que motivou as badaladas subversivas. Um pedido de secretos simpatizantes da Inconfidência? Apenas uma manifestação de piedade cristã? Um sineiro bêbado? Não sei. Minha tese preferida é que alguém responsável pelos sinos teve um vislumbre histórico. Teve a presciência que ninguém mais teve e ordenou o toque plangente, em homenagem precoce ao futuro herói e pelo ocaso do poder colonial que seu sacrifício desencadearia.

      Nossa Senhora do Rosário serviria como padroeira, não necessariamente de quem consegue adivinhar a História, mas de quem entende o momento que está vivendo ou destoa da maioria, com ou sem razão. Destoantes deveriam ir regularmente em romaria à pequena capela e pedir a bênção dessa Nossa Senhora do Contexto Maior, para melhor poder enfrentar a maioria que badala o que não tem importância e o fato errado e menospreza qualquer batida diferente.

      Os outros sineiros de Ouro Preto não tinham como saber que estavam festejando a morte de um herói. Faltava-lhes a perspectiva histórica para entender o momento e só cumpriram o que se esperava deles. Estão perdoados. Mas que nos sirvam de lição.

(Disponível em https://jornalggn.com.br/noticia/nossa-senhora-dos-destoantes-por-luis-fernando-verissimo. Acesso em: 9 fev. 2018)

“O contexto maior não absolve, exatamente, o contexto imediato, a triste realidade de revelações e escândalos de todos os dias, mas consola. Nossa inspiração deve ser o historiador francês Fernand Braudel, que — principalmente no seu monumental estudo sobre as civilizações do Mediterrâneo — ensinou que, para se entender a História, é preciso concentrar-se no que ele chamava de lalonguedurée, que é outro nome para o contexto maior. Braudel partia do particular e do individual para o social e daí para o nacional e o generacional, se é que existe a palavra, e na sua história da região, o indivíduo e seu cotidiano eram reduzidos a “poeira” (palavra dele também, que incluía até papas e reis) em contraste com a longuedurée, o longo prazo da história verdadeira. Assim na sua obra se encontram as minúcias da vida diária nos países do Mediterrâneo mas compreendidas sub specieaeternitatis, do ponto de vista da eternidade, que é o contexto maior pedante.”

(Disponível em https://jornalggn.com.br/noticia/o-contexto-maior-por-luis-fernando-verissimo. Acesso em 09 fev. 2018).


O segmento em destaque também foi produzido por Luís Fernando Veríssimo, em sua crônica, de fevereiro de 2015, O contexto maior, título retomado no trecho da crônica Nossa Senhora dos Destoantes, publicada em 09 de fevereiro de 2018 em: “Destoantes deveriam ir regularmente em romaria à pequena capela e pedir a bênção dessa Nossa Senhora do Contexto Maior, para melhor poder enfrentar a maioria que badala o que não tem importância e o fato errado e menospreza qualquer batida diferente”.


Sobre essa referência de O contexto maior em Nossa Senhora dos Destoantes, pode-se afirmar que

Alternativas
Comentários
  • GAB: E

  • SERIAMENTE, PRA QUÊ COLOCAR UM TEXTO DESTE TAMANHO NA PROVA...


ID
2688832
Banca
FUMARC
Órgão
SEE-MG
Ano
2018
Provas
Disciplina
Português
Assuntos

Leia atentamente os textos II e III. A questão  refere-se a eles.


TEXTO II


              Peão morre após ser pisoteado por touro em festa de Alvorada

        Vítima participava de montaria quando caiu no chão e foi atingida pelas

                  patas do animal. Homem não usava capacete de segurança.

Por Pâmela Fernandes, G1 Ji-Paraná e Região Central

Globo.com 25/06/2017 17h03 Atualizado 25/06/2017 17h39


      Um peão de 24 anos morreu neste fim de semana após ser pisoteado por um touro, durante uma montaria no distrito de Terra Boa em Alvorada do Oeste (RO), a 460 quilômetros de Porto Velho. Conforme informações do registro policial, o rapaz participava da montaria em uma festa, quando caiu e acabou sendo atingido pelas patas do animal.

      Um vídeo gravado por espectadores do rodeio da festa mostra o momento em que o jovem, que era peão profissional, monta no touro e a porteira é aberta.

      Poucos segundos depois o peão se desequilibrou, caiu do animal e foi pisoteado. Os palhaços então tentam afastar o touro do jovem, que ficou caído no meio da arena.

      Na sequência o boi volta na direção dos palhaços e rodopia novamente sobre a vítima. Assustado, o público da arquibancada começa a gritar e o locutor pede ajuda médica.


TEXTO III


      Ezequiel voou parafusado. Quando estava de boca pro céu, as estrelas e as luzes da arena formaram um telegrama manchado nos seus olhos. Bateu chapado no chão. Ouvido apitando. Deu até vontade de rir... Mas não é que o touro desceu com uma pisada tão forte que as costelas se esmigalharam por cima do coração?! Foi menos que suspiro e mais que dolorido. Ele ainda levantou o chapéu e batendo a poeira das calças! Ezequiel, esse insistente... Os braços valeram pra isso. Mas também só pra isso, porque ao cair de novo já foi de cara... e completamente morto. (Barretos – Brasil – 1996)

(BONASSI, Fernando. Passaporte. São Paulo: Cosac & Naify Edições, 2001)

I. É possível que a jornalista Pâmela Fernandes não tenha presenciado o fato por ela noticiado, embora demonstre conhecer a vítima, conforme indica a informação “que era peão profissional”.

II. A jornalista Pâmela Fernandes narra com certo distanciamento a partir de informações colhidas de um registro policial no qual a situação fora narrada como algo já acontecido, conforme indicam os verbos no pretérito imperfeito.

III. Pâmela Fernandes narra a partir de imagens de um vídeo feito por espectadores, transpostas com verbos no presente do indicativo, como se houvesse simultaneidade entre o que vídeo mostra e o relato feito pela jornalista.


O texto II é uma notícia publicada em um site de grande circulação e apresenta um fato acontecido em Roraima. Quanto à posição da jornalista em relação ao fato narrado, é CORRETO o que se afirma apenas em:

Alternativas
Comentários
  • Gabarito da banca letra E, mas discordo. Se alguém souber explicar a afirmativa III fico agradecida. E se meu comentário estiver incorreto, chama no PRIVADO (pois não costumo voltar nas questões respondidas) que retifico o comentário. 

     

    Antes de qualquer coisa gostaria de esclarecer uma coisa: FUMARC RO = RONDÔNIA E NÃO RORAIMA

     

    No texto diz: (...) Terra Boa em Alvorada do Oeste (RO), a 460 quilômetros de Porto Velho. <----- capital

    O texto II é uma notícia publicada em um site de grande circulação e apresenta um fato acontecido em Roraima. 

     

    Agora vamos a questão: Não concordo  com o gabarito, pois a afirmativa III diz que: . Pâmela Fernandes narra a partir de imagens de um vídeo feito por espectadores, transpostas com verbos no presente do indicativo, como se houvesse simultaneidade entre o que vídeo mostra e o relato feito pela jornalista.

     

    verbos no presente do indicativo em azul /  vermelho o pretérito perfeito   /  verde pretérito imperfeito

     

       Um vídeo gravado por espectadores do rodeio da festa mostra o momento em que o jovem, que era peão profissional, monta no touro e a porteira é aberta.

          Poucos segundos depois o peão se desequilibrou, caiu do animal e foi pisoteado. Os palhaços então tentam afastar o touro do jovem, que ficou caído no meio da arena.

          Na sequência o boi volta na direção dos palhaços e rodopia novamente sobre a vítima. Assustado, o público da arquibancada começa a gritar e o locutor pede ajuda médica.

     

    Sendo assim na narrativa da Pâmela têm verbos no presente do indicativo, no pretérito perfeito e também no pretérito imperfeito e dessa forma dizer que a narrativa foi transpostas com verbos no presente do indicativo não seria incorreto?

  • uma dica matadora para pret imperfeito..de uma estudante do qconcurso que aprendi..

    verbo terminou em : VA IA NHA ERA

  • acredito que eles levaram em conta que :

    2-...a partir de informações colhidas de um registro policial

    3-...a partir de imagens de um vídeo feito por espectadores

    Não queira dizer que foi exclusivamente uma ou outra, não anulando uma a outra.

    Se alguém tiver alguma teoria melhor me avise,rs.

  • O item II está incorreto, apesar do Gabarito `E ́ considerá-la correta

    II. A jornalista Pâmela Fernandes narra com certo distanciamento a partir de informações colhidas de um registro policial no qual a situação fora narrada como algo já acontecido, conforme indicam os verbos no pretérito imperfeito.

    Carga semântica do verbo no pretérito IMPERFEITO= ação INACABADA (ação frequente no passado)

    "como algo já acontecido" = ação CONCLUÍDA no passado (passado concluído) = pretérito PERFEITO!

  • Oi????

    "Algo já acontecido" é pretérito imperfeito? Se algo já aconteceu é pretérito perfeito. Além disso, todas as informações decorrentes do registro policial claramente se referem ao passado (por óbvio) e retratam fatos já finalizados (por óbvio, já que o peão morreu!).

    Essas questões de interpretação de texto da FUMARC são muito ruins.


ID
2688835
Banca
FUMARC
Órgão
SEE-MG
Ano
2018
Provas
Disciplina
Português
Assuntos

Leia atentamente os textos II e III. A questão  refere-se a eles.


TEXTO II


              Peão morre após ser pisoteado por touro em festa de Alvorada

        Vítima participava de montaria quando caiu no chão e foi atingida pelas

                  patas do animal. Homem não usava capacete de segurança.

Por Pâmela Fernandes, G1 Ji-Paraná e Região Central

Globo.com 25/06/2017 17h03 Atualizado 25/06/2017 17h39


      Um peão de 24 anos morreu neste fim de semana após ser pisoteado por um touro, durante uma montaria no distrito de Terra Boa em Alvorada do Oeste (RO), a 460 quilômetros de Porto Velho. Conforme informações do registro policial, o rapaz participava da montaria em uma festa, quando caiu e acabou sendo atingido pelas patas do animal.

      Um vídeo gravado por espectadores do rodeio da festa mostra o momento em que o jovem, que era peão profissional, monta no touro e a porteira é aberta.

      Poucos segundos depois o peão se desequilibrou, caiu do animal e foi pisoteado. Os palhaços então tentam afastar o touro do jovem, que ficou caído no meio da arena.

      Na sequência o boi volta na direção dos palhaços e rodopia novamente sobre a vítima. Assustado, o público da arquibancada começa a gritar e o locutor pede ajuda médica.


TEXTO III


      Ezequiel voou parafusado. Quando estava de boca pro céu, as estrelas e as luzes da arena formaram um telegrama manchado nos seus olhos. Bateu chapado no chão. Ouvido apitando. Deu até vontade de rir... Mas não é que o touro desceu com uma pisada tão forte que as costelas se esmigalharam por cima do coração?! Foi menos que suspiro e mais que dolorido. Ele ainda levantou o chapéu e batendo a poeira das calças! Ezequiel, esse insistente... Os braços valeram pra isso. Mas também só pra isso, porque ao cair de novo já foi de cara... e completamente morto. (Barretos – Brasil – 1996)

(BONASSI, Fernando. Passaporte. São Paulo: Cosac & Naify Edições, 2001)

I. O narrador de rodeio é testemunha ocular do fato narrado, conforme indica a frase “Deu até vontade de rir...”.

II. O narrador de rodeio retrata a cena cruelmente, sem demonstrar constrangimento quanto a essa perspectiva, segundo a ironia presente em “Ezequiel, esse insistente...”.

III. A voz narrativa de rodeio banaliza a atuação do peão, demonstrando seu engajamento com relação à causa em favor de animais.


O texto III é um conto, parte da obra Passaporte, de Fernando Bonassi, publicado pela editora Cosac & Naify. Quanto à posição do narrador em relação ao fato narrado, é CORRETO o que se afirma apenas em:

Alternativas
Comentários
  • só eu quem achou que todas estavam erradas???

  • Gab.: D

    lll-A voz narrativa de rodeio banaliza a atuação do peão, demonstrando seu engajamento com relação à causa em favor de animais.

    Não pode ser, pois em nenhum momento ele fala que é a favor dos animais.

  • Gabarito letra D 

     

    Não Cindy também achei que estavam todas erradas, mas daí como não tinha essa opção e a afirmativa III estava claramente errada... fui por eliminação. 

  • a I) está certa ..fiquei em duvida da II)

  • II. O narrador de rodeio retrata a cena cruelmente, sem demonstrar constrangimento quanto a essa perspectiva, segundo a ironia presente em “Ezequiel, esse insistente...”.

  • Não consegui visualizar ironia no trecho. Alguém pode me ajudar?

  • Bruno Luiz, o narrador disse q, ao ser pisado, o peão havia ainda levantado o chapéu (como eles fazem no rodeio qdo estão tentando dominar o animal). Co

    Vejo aí a ironia do narrador ao dizer que o peão era insistente, como se este tivesse ainda no "jogo". Na vdd o peão já estava fraturado e talvez morto.

    Espero ter ajudado.

  • rsrs eu tinha entendi "o narrador" como o narrador do rodeio e não o do texto kkk que vacilo.

  • Cadê a crueldade na narração da cena?

    Meu, essa banca vai dar muito trabalho nas questões de português da PC-MG.


ID
2688838
Banca
FUMARC
Órgão
SEE-MG
Ano
2018
Provas
Disciplina
Português
Assuntos

Leia atentamente os textos II e III. A questão  refere-se a eles.


TEXTO II


              Peão morre após ser pisoteado por touro em festa de Alvorada

        Vítima participava de montaria quando caiu no chão e foi atingida pelas

                  patas do animal. Homem não usava capacete de segurança.

Por Pâmela Fernandes, G1 Ji-Paraná e Região Central

Globo.com 25/06/2017 17h03 Atualizado 25/06/2017 17h39


      Um peão de 24 anos morreu neste fim de semana após ser pisoteado por um touro, durante uma montaria no distrito de Terra Boa em Alvorada do Oeste (RO), a 460 quilômetros de Porto Velho. Conforme informações do registro policial, o rapaz participava da montaria em uma festa, quando caiu e acabou sendo atingido pelas patas do animal.

      Um vídeo gravado por espectadores do rodeio da festa mostra o momento em que o jovem, que era peão profissional, monta no touro e a porteira é aberta.

      Poucos segundos depois o peão se desequilibrou, caiu do animal e foi pisoteado. Os palhaços então tentam afastar o touro do jovem, que ficou caído no meio da arena.

      Na sequência o boi volta na direção dos palhaços e rodopia novamente sobre a vítima. Assustado, o público da arquibancada começa a gritar e o locutor pede ajuda médica.


TEXTO III


      Ezequiel voou parafusado. Quando estava de boca pro céu, as estrelas e as luzes da arena formaram um telegrama manchado nos seus olhos. Bateu chapado no chão. Ouvido apitando. Deu até vontade de rir... Mas não é que o touro desceu com uma pisada tão forte que as costelas se esmigalharam por cima do coração?! Foi menos que suspiro e mais que dolorido. Ele ainda levantou o chapéu e batendo a poeira das calças! Ezequiel, esse insistente... Os braços valeram pra isso. Mas também só pra isso, porque ao cair de novo já foi de cara... e completamente morto. (Barretos – Brasil – 1996)

(BONASSI, Fernando. Passaporte. São Paulo: Cosac & Naify Edições, 2001)

Assinale a afirmativa INCORRETA.

Alternativas
Comentários
  • Gabarito letra E

     

    Sinestesia: Consiste em mesclar, numa mesma expressão, as sensações percebidas por diferentes órgãos do sentido.

    Exemplo: Um grito áspero revelava tudo o que sentia. (grito = auditivo; áspero = tátil);

     

    Outras figuras de linguagem:

     

    Catacrese: Trata-se de uma metáfora que, dado seu uso contínuo, cristalizou-se. A catacrese costuma ocorrer quando, por falta de um termo específico para designar um conceito, toma-se outro "emprestado".

    Assim, passamos a empregar algumas palavras fora de seu sentido original. Exemplos:"asa da xícara","maçã do rosto"...

     

    Perífrase: Trata-se de uma expressão que designa um ser através de alguma de suas características ou atributos, ou de um fato que o celebrizou. Veja o exemplo:A Cidade Maravilhosa (= Rio de Janeiro) continua atraindo visitantes do mundo todo.

     

    https://www.soportugues.com.br/secoes/estil/estil4.php

  • Ambos os textos tratam sobre acidentes em rodeio (enredo) e acerca da notícia da morte de um peão (intenção de noticiar).

    Ao meu ver, a letra E está correta.

  • Ouvido apitando: sinestesia.


ID
2688841
Banca
FUMARC
Órgão
SEE-MG
Ano
2018
Provas
Disciplina
Português
Assuntos

TEXTO IV


Encostei meu ombro naquele céu curvo e terno

No lago as estrelas molhavam-se

Sussurravam que meu abraço

Contivera a terra inteira e os ares

ALVIM, Francisco. In: HOLLANDA, Heloísa Buarque de. 26 poetas hoje. Rio de Janeiro: Aeroplano, 2007. p. 23


No poema de Francisco Alvim, o lirismo se dá por meio da

Alternativas
Comentários
  • Sussurravam que meu abraço - audição

     

  • GAB: A

  • Funções da Linguagem

    1) Referencial- informativa: texto centrado na mensagem imposta o que se diz. Em geral mais formal. 

    2) Emotiva/Expressiva: centrada no autor. Apresenta traços de subjetividade. Abordagem do tema é mais significativa que a própria mensagem.

    3) Conativa-Apelativa: Típica dos textos PUBLICITÁRIOS, objetiva convencer, persuadir o leitor. Infinitivos e imperativos como estratégia.

    4) Fática: objetiva iniciar ou manter a comunicação. ex: Alô.

    5) Poética: não é exclusiva da poesia. Trata-se de uma representação estética de palavras. São explorados recursos expressivos como: rima, trocadilho, campo semântico, metáfora.

    6) Metalinguístico: fala de si mesmo. 

  • Analisemos letra a letra.

    Letra A - CERTA - De fato! Explora-se o tato (... as estrelas molhavam-se) e a audição (Sussurravam...).

    Letra B - ERRADA - Não se nota no eu-lírico um sentimento de melancolia (tristeza). Há apenas a expressão dos gestos.

    Letra C - ERRADA - Não se nota no eu-lírico um sentimento conflito. Há apenas a expressão dos gestos.

    Letra D - ERRADA - Em nenhum momento se faz referência a poetas no texto. Há sim menção a imagens e sentidos.

    Letra E - ERRADA - Não fica evidente a expressão de vocábulos que guardem relação com a ideia de sonho (onírico).

  • Onirismo (do grego oneiros que significa sonho) em medicina se refere a um estado mental que costuma ocorrer em síndromes confusionais e é constituído por um conjunto de alucinações visuais interagindo entre si e com o "sonhador" enquanto este está acordado. Eu lírico, eu poético ou sujeito lírico são nomenclaturas utilizadas para indicar a voz que enuncia o poema. O gênero lírico é aquele destinado a expressar emoções, sensações, disposições psíquicas, ou seja, a vivência de um eu em seu encontro com o mundo. ... O eu lírico é parte essencial do poema.
  • O rótulo “imagens sensoriais” é utilizado para mencionar o grande leque de representações ou ideais que tem relação com os sentidos. ... Estas imagens podem ser visuais, auditivas, táteis, gustativas ou olfativas. Todas elas são utilizadas como recurso literário para proporcionar beleza e expressividade a um texto.


ID
2688844
Banca
FUMARC
Órgão
SEE-MG
Ano
2018
Provas
Disciplina
Português
Assuntos

TEXTO V


De Sinal de menos

perdido entre signos

decifro devoro

persigo persigno

redecifroredevoro

entre signos perdido

devoro decifro

sigo poesigno

redevororedecifro

entreperdido paraíso

voraz cifro

desenho & desígnio

(ÁVILA, Carlos. Sinal de menos. Ouro Preto / MG: Tipografia do Fundo de Ouro Preto, 1989)


A leitura do poema de Carlos Ávila sugere que a criação literária se dá

Alternativas
Comentários
  • GAB: A-  Que persiste.

  • Ao meu ver é a letra c ... kkkk

  • Gabarito letra A para os não assinantes

     

    Esse sujeito deve estudar para concurso: Fazendo uma analogia entre uma matéria que não entendemos poderíamos ter: 

     

    Matéria: Português : 

     

    perdido entre signos (por mais que estudemos, nos sentimos perdidos)

    decifro devoro  (interpretamos, estudamos e o fazemos demasiadamente

    persigo persigno (perseguimos entender a matéria)

    redecifroredevoro (mas às vezes, não conseguimos, então é preciso redecifrar ou seja, fazer tudo novamente, logo temos persistencia)

    entre signos perdido (continua perdido, quem nunca ao estudar português?

    devoro decifro (mesmo com toda dificuldade, o concurseiro é persistente... continua devorando a matéria e decifrando-a)

    sigo poesigno

    redevororedecifro (sempre! até a nomeação!)

     

     

  • que banca escrot a. nunca vi igual.


ID
2688847
Banca
FUMARC
Órgão
SEE-MG
Ano
2018
Provas
Disciplina
Português
Assuntos

TEXTO VI


A terra é nossa


A terra é um bem comum

Que pertence a cada um

Com o seu poder além,

Deus fez a grande natura

Mas não passou escritura

Da terra para ninguém


Se a terra foi Deus quem fez

Se é obra da criação

Deve cada camponês

Ter uma faixa de chão.


Sei que o latifundiário

Egoísta e usuário

Da terra toda se apossa,

Causando crises fatais

Porém nas leis naturais

Sabemos que a terra é nossa.


Quando um agricultor solta

O seu grito de revolta

Tem razão de reclamar,

Não há maior padecer

Do que um camponês viver

Sem terra pra trabalhar.

(Disponível em: http://patativadoassare.com/a-terra-e-nossa/. Acesso em: 5 jan. 2018)


Considere a seguinte declaração de Patativa do Assaré e as afirmativas abaixo:


“Não é, então, o papel do poeta um papel neutro, de simples observador. O poeta nasceu não só com o dom da poesia, como também com o da verdade e o da justiça. O poeta comenta, critica, ensina...”.


I. A rigidez da métrica configura a rigidez ideológica que perpassa a composição das estrofes.

II. Busca pela igualdade social, injustiça social e crítica política são temas encadeados presentes nas estrofes.

III. A presença de discurso religioso contribui para a proposição da justiça defendida nos versos.

IV. O sujeito poético fala sobre o sofrimento de um homem que, sendo da terra, vê-se alijado de seu direito.


Anúncios


Ilustra(m) a declaração de Patativa do Assaré, apenas:

Alternativas
Comentários
  • Em minha opinião, DEUS é diferente de religião, não achei nada de religioso no texto....

  • II. Busca pela igualdade social, injustiça social e crítica política são temas encadeados presentes nas estrofes. (Verdadeira)

    III. A presença de discurso religioso contribui para a proposição da justiça defendida nos versos.(Verdadeira)

    IV. O sujeito poético fala sobre o sofrimento de um homem que, sendo da terra, vê-se alijado de seu direito.(Verdadeira)

    GAB:D

  • Sinceramente, não entendi! Na letra D, é forçar mto a barra!

  • Gabarito letra D para a banca,

     

    Mas discurso religioso foi meio forçado mesmo... 

  • Esse poeta foi um sertanejo nordestino, analfabeto, sem estudo, mas com um enorme entendimento sobre a vida. Como a maioria dos sertanejos nordestinos era muito religioso e nota-se muito essa influência em suas obras. Ao meu ver teria que conhecer o poeta.


ID
2688850
Banca
FUMARC
Órgão
SEE-MG
Ano
2018
Provas
Disciplina
Português
Assuntos

TEXTO VII


Discreta e formosíssima Maria

Gregório de Matos Guerra


Discreta e formosíssima Maria,

Enquanto estamos vendo a qualquer hora

Em tuas faces a rosada Aurora,

Em teus olhos, e boca o Sol, e o dia:


Enquanto com gentil descortesia

O ar, que fresco Adônis te namora,

Te espalha a rica trança voadora,

Quando vem passear-te pela fria:


Goza, goza da flor da mocidade,

Que o tempo trota a toda ligeireza,

E imprime em toda a flor sua pisada.

Oh não aguardes, que a madura idade

Te converta em flor, essa beleza

Em terra, em cinza, em pó, em sombra, em nada


O poema de Gregório de Matos salienta

Alternativas
Comentários
  • GAB: B - 

  • Efemeridade é aquilo que é transitório, brevemente.

    Vanidade é vão; vaidade, estultícia.

    "Goza, goza da flor da mocidade,

                    Que o tempo trota a toda ligeireza,

    E imprime em toda a flor sua pisada.

                  Oh não aguardes, que a madura idade

                     Te converta em flor, essa beleza

              Em terra, em cinza, em pó, em sombra, em nada

    GABARITO B

  • O texto traz várias elementos que evidenciam uma passagem do tempo, ou seja, algo efêmero, passageiro:

    "Enquanto estamos vendo a qualquer hora"

    "Enquanto com gentil descortesia"

    "Oh não aguardes, que a madura idade"

    "Te converta em flor, essa beleza".


ID
2688853
Banca
FUMARC
Órgão
SEE-MG
Ano
2018
Provas
Disciplina
Português
Assuntos

Os textos VIII e IX, motivadores, o(a) auxiliarão a responder à questão.


TEXTO VIII 


      Na busca por definir critérios para fixar uma escrita convencionalizada, observamos que as muitas línguas com notação alfabética enfrentaram, desde a Antiguidade, uma disputa entre opções (cf. BLANCHE-BENVENISTE; CHERVEL, 1974). Por um lado, desde a Roma e a Grécia antigas, já existia tendência de buscar respeitar o princípio fonográfico, segundo o qual a ortografia deveria estar o mais próxima possível da pronúncia das palavras. Apesar das boas intenções, isso envolvia um problema sem solução perfeita: se diferentes falantes de uma mesma língua – pertencentes a regiões, grupos socioculturais e épocas diferentes – pronunciam de forma distinta as mesmas palavras, a busca de uma correspondência “limpa” entre formas de falar e escrever teria sempre que partir de uma pronúncia idealizada, tomada como padrão. Por outro lado, encontramos há muitos séculos a defesa de um princípio etimológico, segundo o qual as palavras provenientes de outra língua deveriam preservar as grafias que tinham nas línguas de origem. Assim, no caso de línguas como português, francês e espanhol, as formas latinas e gregas seriam candidatas especiais à manutenção de suas notações originais (e a uma desobediência do princípio fonográfico).

      Finalmente, nessa disputa entre perspectivas diferentes, a história de evolução das normas ortográficas das línguas aqui mencionadas revela que não só tendeu-se a fazer um “casamento” dos dois princípios (fonográfico e etimológico) já citados, como a incorporar formas escritas que surgiram por mera tradição de uso. 

      Tudo em ortografia precisa ser visto, consequentemente, como fruto de uma convenção arbitrada / negociada ao longo da História. Mesmo a separação das palavras no texto, com espaços em branco, é uma invenção recente, bem como o emprego sistemático de sinais de pontuação. Até o século XVIII, quando predominava a leitura em voz alta, muitos textos eram notados com as palavras “pegadas”. Como também tinham poucos sinais de pontuação, cabia ao leitor, ao “preparar” sua leitura, definir como iria segmentar o texto. Numa língua como o português, vemos hoje que a norma ortográfica envolve não só a definição das letras autorizadas para escrever-se cada palavra, como também a segmentação destas no texto e o emprego da acentuação. 

      Diferentemente da pontuação – que permite opções / variações conforme o estilo ou interesse de quem escreve –, no caso da ortografia as convenções estabelecidas são avaliadas taxativamente: a grafia de uma palavra ou está certa ou errada, não se julgando sua qualidade em termos de “aproximação” do esperado (MORAIS, 1998; SILVA, 2004).


(MORAIS, Arthur Gomes. A norma ortográfica do português: o que é? para que serve? como está organizada? In: SILVA, Alexsandro; MORAIS, Arthur G.; MELO, Kátia L. R. Ortografia na sala de aula. Belo Horizonte: Autêntica, 2005. Disponível em: http://www.serdigital.com.br/gerenciador/clientes/ceel/arquivos/25.pdf. Acesso em: 5 jan. 2018)


TEXTO IX


      Nosso objeto de estudo, a linguagem, mostra-se diferente aos olhos do observador, conforme ele a investigue. Por exemplo, como representação do pensamento, e este como representação do mundo. Entretanto, sabemos que, no uso cotidiano da língua, não pensamos conscientemente em formas para traduzir conteúdos, nem em conteúdos preexistentes que buscam formas. Forma e pensamento nascem juntos; nossos pensamentos e representações são feitos de palavras e se constroem, ou na interação contextualizada com o outro ou no diálogo interno com outros discursos também feitos de palavras. A referência à decodificação, presente nos PCN+, não pode nos induzir também ao engano de reduzir as línguas naturais — em particular, a língua portuguesa — a um sistema de sinais, por meio do qual um emissor comunica a um recebedor determinada mensagem.

      A partir dessa concepção, aprender e ensinar língua seria dominar o código, e a compreensão e a produção de textos se reduziriam ao processo de decodificação e codificação: para cada sinal ou combinação de sinais corresponderia um sentido. Sabemos que os enunciados produzidos nas línguas naturais têm uma parte material - os sons, no caso da língua oral, e as formas, no caso da escrita -, mas têm também uma parte subentendida, essencial para a produção de sentido na interação. Essa parte subentendida, digamos, “invisível”, está no contexto de produção do enunciado, em sua enunciação e co-enunciação, nos conhecimentos de mundo e nos valores partilhados pelos interlocutores.

(SEE MG. Currículo Básico Comum. Proposta Curricular. Língua Portuguesa - Ensinos Fundamental e Médio. 2005, p. 11- 12)

Assinale a afirmativa INCORRETA:

Alternativas
Comentários
  • Gabarito E

  • Essa questão vence pelo cansaço. 

  • O trabalho com ortografia demanda treinos constantes, diários, visto que cada item que apresenta dificuldade precisa ser tratado de forma individualizada, num contexto específico. Isso inviabiliza a promoção de reflexões com base em regularidades, já que tais vocábulos apresentam motivações etimológicas e /ou fonológicas, ou seguem aspectos devidos à mera tradição do uso.


    Acredito que o erro está nesse seguimento que destaquei, porque o estudo da ortografia não pode ser individualizado num contexto específico, mas sim ao contrário, tem que se contextualizar, significar a palavra no texto para então saber a sua forma que ela vai ser escrita.

  • Gabarito Letra E para os não assinantes. 

     

    Silma, eu entendi que quando o comando fala que o trabalho com ortografia  deve ser tratado de forma individualizada, quer dizer, por exemplo, que aquele aluno que está com mais dificuldade, ou aquele assunto mais complexo, deve ser tratado de forma individualizada, de modo que o aluno consiga superar a dificuldade. Eu coloquei letra E pela parte em vermelho, que na minha opinião está errada.  Uma vez que, a reflexão e o trabalho individualizado com o aluno não INVIABILIZA reflexões, muito pelo contrário. 

     

     e) O trabalho com ortografia demanda treinos constantes, diários, visto que cada item que apresenta dificuldade precisa ser tratado de forma individualizada, num contexto específico. Isso inviabiliza a promoção de reflexões com base em regularidades, já que tais vocábulos apresentam motivações etimológicas e /ou fonológicas, ou seguem aspectos devidos à mera tradição do uso.

     

  • OHHHH QUESTAO DESGRANHADA.........KKKKKKKKKKKKKKKKKK TU NAO ME GANHOU PELO CANSAÇO

  • Gabarito letra E) sem duvida, no final da citar que é tradição e brincadeira !

  • Gabarito letra E) sem duvida, no final da citar que é tradição e brincadeira !

  • Gabarito letra E) sem duvida, no final da citar que é tradição e brincadeira !

  • E - A ortografia deve ser vista no contexto, não de forma isolada,e deve ser permeada de reflexões. A classe de palavras ,por exemplo,tem suas regras,mas nem sempre uma palavra se encaixa na classe a qual pertence, dependerá,entretanto, do contexto em que está inserida considerando a frase,a oração,o texto em si.


ID
2688856
Banca
FUMARC
Órgão
SEE-MG
Ano
2018
Provas
Disciplina
Português
Assuntos

Os textos VIII e IX, motivadores, o(a) auxiliarão a responder à questão.


TEXTO VIII 


      Na busca por definir critérios para fixar uma escrita convencionalizada, observamos que as muitas línguas com notação alfabética enfrentaram, desde a Antiguidade, uma disputa entre opções (cf. BLANCHE-BENVENISTE; CHERVEL, 1974). Por um lado, desde a Roma e a Grécia antigas, já existia tendência de buscar respeitar o princípio fonográfico, segundo o qual a ortografia deveria estar o mais próxima possível da pronúncia das palavras. Apesar das boas intenções, isso envolvia um problema sem solução perfeita: se diferentes falantes de uma mesma língua – pertencentes a regiões, grupos socioculturais e épocas diferentes – pronunciam de forma distinta as mesmas palavras, a busca de uma correspondência “limpa” entre formas de falar e escrever teria sempre que partir de uma pronúncia idealizada, tomada como padrão. Por outro lado, encontramos há muitos séculos a defesa de um princípio etimológico, segundo o qual as palavras provenientes de outra língua deveriam preservar as grafias que tinham nas línguas de origem. Assim, no caso de línguas como português, francês e espanhol, as formas latinas e gregas seriam candidatas especiais à manutenção de suas notações originais (e a uma desobediência do princípio fonográfico).

      Finalmente, nessa disputa entre perspectivas diferentes, a história de evolução das normas ortográficas das línguas aqui mencionadas revela que não só tendeu-se a fazer um “casamento” dos dois princípios (fonográfico e etimológico) já citados, como a incorporar formas escritas que surgiram por mera tradição de uso. 

      Tudo em ortografia precisa ser visto, consequentemente, como fruto de uma convenção arbitrada / negociada ao longo da História. Mesmo a separação das palavras no texto, com espaços em branco, é uma invenção recente, bem como o emprego sistemático de sinais de pontuação. Até o século XVIII, quando predominava a leitura em voz alta, muitos textos eram notados com as palavras “pegadas”. Como também tinham poucos sinais de pontuação, cabia ao leitor, ao “preparar” sua leitura, definir como iria segmentar o texto. Numa língua como o português, vemos hoje que a norma ortográfica envolve não só a definição das letras autorizadas para escrever-se cada palavra, como também a segmentação destas no texto e o emprego da acentuação. 

      Diferentemente da pontuação – que permite opções / variações conforme o estilo ou interesse de quem escreve –, no caso da ortografia as convenções estabelecidas são avaliadas taxativamente: a grafia de uma palavra ou está certa ou errada, não se julgando sua qualidade em termos de “aproximação” do esperado (MORAIS, 1998; SILVA, 2004).


(MORAIS, Arthur Gomes. A norma ortográfica do português: o que é? para que serve? como está organizada? In: SILVA, Alexsandro; MORAIS, Arthur G.; MELO, Kátia L. R. Ortografia na sala de aula. Belo Horizonte: Autêntica, 2005. Disponível em: http://www.serdigital.com.br/gerenciador/clientes/ceel/arquivos/25.pdf. Acesso em: 5 jan. 2018)


TEXTO IX


      Nosso objeto de estudo, a linguagem, mostra-se diferente aos olhos do observador, conforme ele a investigue. Por exemplo, como representação do pensamento, e este como representação do mundo. Entretanto, sabemos que, no uso cotidiano da língua, não pensamos conscientemente em formas para traduzir conteúdos, nem em conteúdos preexistentes que buscam formas. Forma e pensamento nascem juntos; nossos pensamentos e representações são feitos de palavras e se constroem, ou na interação contextualizada com o outro ou no diálogo interno com outros discursos também feitos de palavras. A referência à decodificação, presente nos PCN+, não pode nos induzir também ao engano de reduzir as línguas naturais — em particular, a língua portuguesa — a um sistema de sinais, por meio do qual um emissor comunica a um recebedor determinada mensagem.

      A partir dessa concepção, aprender e ensinar língua seria dominar o código, e a compreensão e a produção de textos se reduziriam ao processo de decodificação e codificação: para cada sinal ou combinação de sinais corresponderia um sentido. Sabemos que os enunciados produzidos nas línguas naturais têm uma parte material - os sons, no caso da língua oral, e as formas, no caso da escrita -, mas têm também uma parte subentendida, essencial para a produção de sentido na interação. Essa parte subentendida, digamos, “invisível”, está no contexto de produção do enunciado, em sua enunciação e co-enunciação, nos conhecimentos de mundo e nos valores partilhados pelos interlocutores.

(SEE MG. Currículo Básico Comum. Proposta Curricular. Língua Portuguesa - Ensinos Fundamental e Médio. 2005, p. 11- 12)

Levando em consideração as novas regras ortográficas vigentes, assinale a afirmação INCORRETA:

Alternativas
Comentários
  • desde que em paroxítonas? 
    não há este requisito.

  • A regra do I e U tônicos precedidos de vogal e formando sílaba sozinho ou com S, continua a mesma : são acentuados e constituem hiato;

    Ex - e-go-ís-mo.

    BIZU - Se o vogal I ou U forem seguidos de NH ou quanado vierem em palavras paroxitonas, depois de ditongo, o acento agudo desaparece, ex :

    Ra-i-nha, fei-u-ra.

     

  • Letra A incorreta

    As vogais I e U tônicas, quando precedidas de ditongos crescentes em paroxítonas, (feiura, boiuna, baiuca) NÃO têm mais acento. Lembrando que em oxítonas o acento permanece (tuiuiú, Piauí).

    As palavras “viúva” e “saúde” são acentuadas pela regra do hiato - I e U tônicos em sílaba separada, seguidos ou não de "s", e precedidos por vogal.

  • nossa, procurando o erro da b pelo resto da vida.... nunca ia achar mesmo :'((((((((((((( 

     

    assinale a INCORRETAAAAAAAAAAAAAAAAAAAAAAAAAAAAAAAAAAAAAAAAAAAAAAAAAAAAAAAAAAAAAAAAAAAAAAAAAAAAAAAAAAAAAAAAAAAAAAAAAAAAAAAAAAAAAAAAAAAAAAAAAAAAAAAAAAAAAAAAAAAAAAAHHHHHHHHHHHHHHHHHHHHHHHHHHHHHHHHHHHHHHHHHHHHHHHHHHHHHHHHHHHHHHHHHHHHHHHHHHHHHHHHHHHHHHHHHHHHHHHHHHHHHHHHHHHHHHHHHHHHHHHHHHHHHHHHHHHHHHHHHHHH!!!!!!!!!!!!!!!!!!!!!!!!!!!!!!!!!!!!!!!!!!!!!!!!!!!!!!!!!!!!!!!!!!!!!!!!!!!!!!!!!!!!!!!!!!!!!!!!!!!!!!!!!!!!!!!!!!!!!

     

    não importa, meu marido tem dois empregos!

  • Convém fazer uma ressalva sobre a alternativa E: Anti-inflamatório escreve com hífen.

  • Joyce, muito bom o seu comentário! Mas há apenas uma pequena correção nele. Os ditongos precedidos de I ou U para não serem acentuados, serão decrescentes, e não crescentes como você colocou.

  • viuva”, “SAÚDE”, “feiura”, “boiuna”, “baiuca

  • Se o cargo fosse para nível médio, a questão seria nível Harvard....

  • Na alternativa A é incorreta porque os hiatos devem ser vogais em silabas diferentes seguidas ou não de S, e não de ditongo

  • Na alternativa A é incorreta porque os hiatos devem ser vogais em silabas diferentes seguidas ou não de S, e não de ditongo

  • Note que a afirmação da letra A é INCORRETA.

    Os chamados falsos hiatos (que consistem nas vogais I e U tônicas precedidas de ditongos) não mais são acentuados caso a palavra seja paroxítona.

    É o caso de FEIURA, BOCAIUVA, GUAIUBA, etc.

    Essas palavras possuíam acento antes do Acordo Ortográfico. Hoje, não mais possuem.

    Resposta: Letra A

  • Alternativa E também está incorreta, visto que anti-inflamatório deve ser separado. Mas o QC que deve ter errado ao redigir.

  • "Feiúme", "feiúra", "baiúca", "bocaiúva", "boiúna" cascaram fora.

  • LETRA A:

    SA - Ú - DE

    VI - Ú - VA

    DIFERENTE DE

    FEI - U - RA

    BOI - U - NA

    BAI - U - CA

    BO - CAI - U - VA

  • Alternativa A) é a correta,sendo ela certa em partes o erro foi em citar queque a diferença é somente em palavras paroxítonas.


ID
2688859
Banca
FUMARC
Órgão
SEE-MG
Ano
2018
Provas
Disciplina
Português
Assuntos

Atente para os textos X e XI, excerto da BNCC e da obra de Travaglia (2003), para responder à questão.


TEXTO X


       A linguagem é “uma forma de ação interindividual orientada para uma finalidade específica; um processo de interlocução que se realiza nas práticas sociais existentes numa sociedade, nos distintos momentos de sua história” (BRASIL, 1998, p. 20).

           Tal proposta assume a centralidade do texto como unidade de trabalho e as perspectivas enunciativo-discursivas na abordagem, de forma a sempre relacionar os textos a seus contextos de produção e o desenvolvimento de habilidades ao uso significativo da linguagem em atividades de leitura, escuta e produção de textos em várias mídias e semioses. Na esteira do que foi proposto nos Parâmetros Curriculares Nacionais, o texto ganha centralidade na definição dos conteúdos, habilidades e objetivos, considerado a partir de seu pertencimento a um gênero discursivo que circula em diferentes esferas / campos sociais de atividade / comunicação / uso da linguagem. Os conhecimentos sobre os gêneros, sobre os textos, sobre a língua, sobre a norma-padrão, sobre as diferentes linguagens (semioses) devem ser mobilizados em favor do desenvolvimento das capacidades de leitura, produção e tratamento das linguagens, que, por sua vez, devem estar a serviço da ampliação das possibilidades de participação em práticas de diferentes esferas / campos de atividades humanas.

     Ao componente Língua Portuguesa cabe, então, proporcionar aos estudantes experiências que contribuam para a ampliação dos letramentos, de forma a possibilitar a participação significativa e nas diversas práticas sociais permeadas / constituídas pela oralidade, pela escrita e por outras linguagens.

(Disponível em: http://basenacionalcomum.mec.gov.br/images/BNCC_20dez_site.pdf. Acesso em: 19 fev. 2018)


TEXTO XI


                               A sistematização do ensino de gramática


      O trabalho com atividades de ensino de gramática dos tipos “gramática de uso”, “gramática reflexiva” e “gramática normativa” na proposta de Travaglia (1996) seria utilizado essencialmente para o desenvolvimento da competência comunicativa, ou seja, para conseguir que o aluno, como usuário da língua, seja capaz de usar cada vez um maior número de recursos da língua de maneira adequada à produção do(s) efeito(s) de sentido desejado(s) em situações específicas de interação comunicativa, o que inclui o uso das diferentes variedades linguísticas em termos de dialetos e registros e variedades de modo (oral e escrito). Já o trabalho com atividades do tipo “gramática teórica” seria utilizado para: a) atendendo certas posturas da sociedade sobre o domínio de conhecimentos, fornecer ao aluno informação cultural sobre a língua; b) instrumentalizar o aluno com um meio auxiliar aos demais tipos de atividades de ensino de gramática, dando ao aluno uma metalinguagem básica que serviria para facilitar a referência aos elementos da língua. Este objetivo faria parte de outro maior que seria a instrumentalização com recursos para aplicações práticas imediatas; c) desenvolver o raciocínio, para ensinar a pensar de forma organizada na produção de conhecimento sobre um fato, enfim, para ensinar a fazer ciência.

(TRAVAGLIA, Luiz Carlos. Gramática – Ensino Plural. São Paulo: Cortez, 2003. p. 58-59)

Assinale a opção que traz afirmação INCORRETA sobre os textos X e XI:

Alternativas
Comentários
  • GAB:C

  • Gabarito alternativa C. Veja o erro:

    "Os PCN tomam o texto como elemento central a ser focalizado no ensino linguístico, o que é respaldado pela BNCC, que prevê o desenvolvimento de inúmeras competências pelo aluno. No entanto, a orientação dada por Travaglia afasta-se desse pressuposto, já que o autor se preocupa com o ensino estrito de gramática e o desenvolvimento de uma única dimensão (a competência cognitiva) dos aprendizes."

    No texto, encontramos:

    "O trabalho com atividades de ensino de gramática dos tipos “gramática de uso”, “gramática reflexiva” e “gramática normativa” na proposta de Travaglia (1996) seria utilizado essencialmente para o desenvolvimento da competência comunicativa, ou seja, para conseguir que o aluno, como usuário da língua, seja capaz de usar cada vez um maior número de recursos da língua de maneira adequada à produção do(s) efeito(s) de sentido desejado(s) em situações específicas de interação comunicativa, o que inclui o uso das diferentes variedades linguísticas em termos de dialetos e registros e variedades de modo (oral e escrito). "


ID
2688862
Banca
FUMARC
Órgão
SEE-MG
Ano
2018
Provas
Disciplina
Português
Assuntos

Atente para os textos X e XI, excerto da BNCC e da obra de Travaglia (2003), para responder à questão.


TEXTO X


       A linguagem é “uma forma de ação interindividual orientada para uma finalidade específica; um processo de interlocução que se realiza nas práticas sociais existentes numa sociedade, nos distintos momentos de sua história” (BRASIL, 1998, p. 20).

           Tal proposta assume a centralidade do texto como unidade de trabalho e as perspectivas enunciativo-discursivas na abordagem, de forma a sempre relacionar os textos a seus contextos de produção e o desenvolvimento de habilidades ao uso significativo da linguagem em atividades de leitura, escuta e produção de textos em várias mídias e semioses. Na esteira do que foi proposto nos Parâmetros Curriculares Nacionais, o texto ganha centralidade na definição dos conteúdos, habilidades e objetivos, considerado a partir de seu pertencimento a um gênero discursivo que circula em diferentes esferas / campos sociais de atividade / comunicação / uso da linguagem. Os conhecimentos sobre os gêneros, sobre os textos, sobre a língua, sobre a norma-padrão, sobre as diferentes linguagens (semioses) devem ser mobilizados em favor do desenvolvimento das capacidades de leitura, produção e tratamento das linguagens, que, por sua vez, devem estar a serviço da ampliação das possibilidades de participação em práticas de diferentes esferas / campos de atividades humanas.

     Ao componente Língua Portuguesa cabe, então, proporcionar aos estudantes experiências que contribuam para a ampliação dos letramentos, de forma a possibilitar a participação significativa e nas diversas práticas sociais permeadas / constituídas pela oralidade, pela escrita e por outras linguagens.

(Disponível em: http://basenacionalcomum.mec.gov.br/images/BNCC_20dez_site.pdf. Acesso em: 19 fev. 2018)


TEXTO XI


                               A sistematização do ensino de gramática


      O trabalho com atividades de ensino de gramática dos tipos “gramática de uso”, “gramática reflexiva” e “gramática normativa” na proposta de Travaglia (1996) seria utilizado essencialmente para o desenvolvimento da competência comunicativa, ou seja, para conseguir que o aluno, como usuário da língua, seja capaz de usar cada vez um maior número de recursos da língua de maneira adequada à produção do(s) efeito(s) de sentido desejado(s) em situações específicas de interação comunicativa, o que inclui o uso das diferentes variedades linguísticas em termos de dialetos e registros e variedades de modo (oral e escrito). Já o trabalho com atividades do tipo “gramática teórica” seria utilizado para: a) atendendo certas posturas da sociedade sobre o domínio de conhecimentos, fornecer ao aluno informação cultural sobre a língua; b) instrumentalizar o aluno com um meio auxiliar aos demais tipos de atividades de ensino de gramática, dando ao aluno uma metalinguagem básica que serviria para facilitar a referência aos elementos da língua. Este objetivo faria parte de outro maior que seria a instrumentalização com recursos para aplicações práticas imediatas; c) desenvolver o raciocínio, para ensinar a pensar de forma organizada na produção de conhecimento sobre um fato, enfim, para ensinar a fazer ciência.

(TRAVAGLIA, Luiz Carlos. Gramática – Ensino Plural. São Paulo: Cortez, 2003. p. 58-59)

Considere as afirmativas feitas a seguir:


I. Uma boa atividade de ensino gramatical reside no consagrado tripé de exposição (ou explicitação) do conceito em estudo, correta exemplificação (preferivelmente retirados de boas obras literárias ou gêneros mais monitorados da linguagem jornalística) e exercitação exaustiva, a fim de que o aprendiz possa compreender bem o tópico enfocado.

II. Para Travaglia, o ensino de gramática teórica tem grande relevância social, pois há uma aplicação imediata de praticamente todos os tópicos enfocados, que possibilitam ao aprendiz ter melhores resultados no ensino de Ciências.

III. O ensino da vasta metalinguagem constitutiva das prescrições normativas sobre os tópicos que constam do plano curricular de Língua Portuguesa de cada ano / ciclo da educação básica é crucial para o aprendiz, a fim de que este possa ampliar sua desenvoltura nas práticas leitura e escrita.

IV. Ao afirmar que é preciso fornecer “informação cultural sobre a língua”, Travaglia remete-nos ao fato de que conhecer a história da língua (seu percurso de mudanças diacrônicas) é tão importante quanto saber utilizar adequadamente as estruturas consideradas válidas no estágio sincrônico atual, se o que se deseja é melhorar a proficiência linguística do aprendiz.


Verifica-se que estão INCORRETAS as afirmativas:

Alternativas
Comentários
  • Entendi foi nada kkkkk

  • Entendi foi nada 2 

     

    Gabarito para os não assinantes letra C (TUDO ERRADO)???????

  • Uma questão com alto índice de erros e sem comentários dos professores. =\

    Tá difícil, ein, QC!

  • Vamos pedir comentário do professor, galera..

  • Acho que nem os professores entenderam...ahahahahaha

  • As questões da Fumarc são uma piada kkkkk

  • I. Uma boa atividade de ensino gramatical reside no consagrado tripé de exposição (ou explicitação) do conceito em estudo, correta exemplificação (preferivelmente retirados de boas obras literárias ou gêneros mais monitorados da linguagem jornalística) e exercitação exaustiva, a fim de que o aprendiz possa compreender bem o tópico enfocado.

    NÃO MENCIONA ISSO NO TEXTO, PRINCIPALMENTE SOBRE AS OBRAS.

    II. Para Travaglia, o ensino de gramática teórica tem grande relevância social, pois há uma aplicação imediata de praticamente todos os tópicos enfocados, que possibilitam ao aprendiz ter melhores resultados no ensino de Ciências.

    Certas posturas da sociedade, e não grande relevância.

    III. O ensino da vasta metalinguagem constitutiva das prescrições normativas sobre os tópicos que constam do plano curricular de Língua Portuguesa de cada ano / ciclo da educação básica é crucial para o aprendiz, a fim de que este possa ampliar sua desenvoltura nas práticas leitura e escrita.

    Metalinguagem básica, e não vasta.

    IV. Ao afirmar que é preciso fornecer “informação cultural sobre a língua”, Travaglia remete-nos ao fato de que conhecer a história da língua (seu percurso de mudanças diacrônicas) é tão importante quanto saber utilizar adequadamente as estruturas consideradas válidas no estágio sincrônico atual, se o que se deseja é melhorar a proficiência linguística do aprendiz.

    NÃO MENCIONA ISSO NO TEXTO

    FOI ISSO QUE ENTENDI DA QUESTÃO..KKKKKKKK


ID
2688865
Banca
FUMARC
Órgão
SEE-MG
Ano
2018
Provas
Disciplina
Português
Assuntos

Leia a letra da música “Gramática”, a respeito da qual serão feitas a questão. 


TEXTO XII


                                         Gramática

                                                                              Sandra Peres e Luiz Tatti


                                       O substantivo

                            É o substituto do conteúdo

 

                                          O adjetivo

                    É a nossa impressão sobre quase tudo


                                       O diminutivo

                               É o que aperta o mundo

                                      E deixa miúdo


                                       O imperativo

                   É o que aperta os outros e deixa mudo


                               Um homem de letras

                                   Dizendo ideias

                                Sempre se inflama


                              Um homem de ideias

                                  Nem usa letras

                                  Faz ideograma


                               Se altera as letras

                              E esconde o nome

                                   Faz anagrama


                           Mas se mostro o nome

                              Com poucas letras

                                É um telegrama


                               Nosso verbo ser

                              É uma identidade

                               Mas sem projeto


                               E se temos verbo

                                   Com objeto

                              É bem mais direto


                                No entanto falta

                                 Ter um sujeito

                                   Pra ter afeto


                              Mas se é um sujeito

                                  Que se sujeita

                                  Ainda é objeto


                                Todo barbarismo

                                   É o português

                                  Que se repeliu


                                  O neologismo

                                  É uma palavra

                                Que não se ouviu


                                    Já o idiotismo

                               É tudo que a língua

                                     Não traduziu


                                 Mas tem idiotismo

                                  Também na fala

                                    De um imbecil 


(Composição: Sandra Peres e Luiz Tatti Palavra Cantada. Álbum: Canções Curiosas, 1998. Disponível em: https://www.letras.mus.br › Infantil › Palavra Cantada › Gramática. Acesso em: 15 jan. 2018)

A seguir, são trazidas conceituações da gramática tradicional para as categorias linguísticas abordadas na música:


I - “O substantivo: É o substituto do conteúdo.”

Substantivo é classe de palavras com que se denominam os seres, animados ou inanimados, concretos ou abstratos, os estados, as qualidades, as ações. Classe que nomeia ou categoriza tudo que existe. 


II - “O adjetivo / É a nossa impressão sobre quase tudo”

Adjetivo é vocábulo que serve para modificar um substantivo, acrescentando uma qualidade, uma extensão ou uma quantidade àquilo que ele nomeia; diz-se de palavra, locução, oração, pronome. Pode ser atributivo (objetivo) ou interpretativo (subjetivo). 


III - “O diminutivo / É o que aperta o mundo / E deixa miúdo.”

Diminutivo: diz-se do grau do substantivo; trata-se de morfema (sufixo) que, unindo-se à base de um substantivo, indica o grau diminutivo; designa o ser diminuído em relação ao normal, ou com a significação atenuada ou valorizada afetivamente. 


IV - “O imperativo / É o que aperta os outros e deixa mudo.” 

Imperativo: modo verbal que indica ordem, pedido, exortação etc.; determinação de autoridade; ordem, mando, imposição; adjetivo que acentua o caráter de mando, de autoridade, ou que exprime uma ordem; autoritário.


Os autores embasaram-se na dimensão referencial para definirem poeticamente, por meio de pará-frase, as categorias constantes em: 

Alternativas
Comentários
  • Gabarito letra B

  • Não concordo!!!! Adjetivo trás quantidade????????????? não seria o numeral???

  • Gabarito letra B

     

    Tati, achei essa explicação:

     

     Adjetivo numeral:

     

    . indica a sucessão ou a ordem que o nome tem numa série;

    . corresponde aos designados numerais ordinais;

    . ocorre, geralmente, em posição pré-nominal, antecedido de um determinante artigo(1), demonstrativo(2), possessivo(3) ou de um quantificador(4):

    (1) O primeiro aluno a chegar à meta foi o Aristófanes.

    (2) Aquele terceiro tiro foi fatal.

    (3) meu último ano de faculdade foi extraordinário.

    (4) As três primeiras obras de Saramago ainda são desconhecidas do leitor comum.

    . não flexiona (varia) em grau:

    exemplos:

    primeiro

    segundo

    terceiro

    último

    centésimo etc...

     

    Fonte: http://portugues-fcr.blogspot.com/2014/01/subclasses-do-adjetivo.html

     

  • Essa banca é podre

  • Gaba: B



    Quanto ao adjetivo ser numeral é o seguinte: o papel do adjetivo é se referir a um substantivo, modificando-o.


    Conheço duas moças. ==> duas é um numeral adjetivo, pois acompanha moças.



    Fonte: Aulas do Prof. Felipe Luccas, Estratégia

  • Para responder esta questão, exige-se conhecimento em classe de palavras. O candidato deve saber quais assertivas estão corretas. Vejamos:

    I - Correta.

    “O substantivo: É o substituto do conteúdo.”

    Substantivo é a classe de palavras com que se denominam os seres, animados ou inanimados, concretos ou abstratos, os estados, as qualidades, as ações. Classe que nomeia ou categoriza tudo que existe. 

    Ex:

    Homem⇢ ser e animado respectivamente porque é uma espécie e tem vida própria.

    Plástico⇢ inanimado e concreto, respectivamente porque não tem vida e pode tocar

    Tristeza⇢ abstrato e estado respectivamente porque não tem vida própria e é intangível e é como alguém está no momento.

    II - Correta.

    “O adjetivo / É a nossa impressão sobre quase tudo”

    Adjetivo é vocábulo que serve para modificar um substantivo, acrescentando uma qualidade, uma extensão ou uma quantidade àquilo que ele nomeia; diz-se de palavra, locução, oração, pronome. Pode ser atributivo (objetivo) ou interpretativo (subjetivo). 

    Ex: O menino alto foi embora.

    III - Correta.

    “O diminutivo / É o que aperta o mundo / E deixa miúdo.”

    Diminutivo: diz-se do grau do substantivo; trata-se de morfema (sufixo) que, unindo-se à base de um substantivo, indica o grau diminutivo; designa o ser diminuído em relação ao normal, ou com a significação atenuada ou valorizada afetivamente. 

    Ex: O menininho saiu (diminutivo de menino)

    IV - Correta.

    “O imperativo / É o que aperta os outros e deixa mudo.” 

    Imperativo: modo verbal que indica ordem, pedido, exortação etc.; determinação de autoridade; ordem, mando, imposição; adjetivo que acentua o caráter de mando, de autoridade, ou que exprime uma ordem; autoritário.

    Ex: Faça essa questão (ordem).

    Portanto, todas as assertivas estão corretas.

    Gabarito do monitor: B

  • O x da questão foi o seguinte. "Paráfrase" era só se atentar a esse significado. Quem soubesse mataria a questão.

    Paráfrase: Interpretação, explicação ou nova apresentação de um texto (entrecho, obra etc.) que visa torná-lo mais inteligível ou que sugere novo enfoque para o seu sentido.

    Todas os itens apresentaram paráfrase.


ID
2688868
Banca
FUMARC
Órgão
SEE-MG
Ano
2018
Provas
Disciplina
Português
Assuntos

Leia a letra da música “Gramática”, a respeito da qual serão feitas a questão. 


TEXTO XII


                                         Gramática

                                                                              Sandra Peres e Luiz Tatti


                                       O substantivo

                            É o substituto do conteúdo

 

                                          O adjetivo

                    É a nossa impressão sobre quase tudo


                                       O diminutivo

                               É o que aperta o mundo

                                      E deixa miúdo


                                       O imperativo

                   É o que aperta os outros e deixa mudo


                               Um homem de letras

                                   Dizendo ideias

                                Sempre se inflama


                              Um homem de ideias

                                  Nem usa letras

                                  Faz ideograma


                               Se altera as letras

                              E esconde o nome

                                   Faz anagrama


                           Mas se mostro o nome

                              Com poucas letras

                                É um telegrama


                               Nosso verbo ser

                              É uma identidade

                               Mas sem projeto


                               E se temos verbo

                                   Com objeto

                              É bem mais direto


                                No entanto falta

                                 Ter um sujeito

                                   Pra ter afeto


                              Mas se é um sujeito

                                  Que se sujeita

                                  Ainda é objeto


                                Todo barbarismo

                                   É o português

                                  Que se repeliu


                                  O neologismo

                                  É uma palavra

                                Que não se ouviu


                                    Já o idiotismo

                               É tudo que a língua

                                     Não traduziu


                                 Mas tem idiotismo

                                  Também na fala

                                    De um imbecil 


(Composição: Sandra Peres e Luiz Tatti Palavra Cantada. Álbum: Canções Curiosas, 1998. Disponível em: https://www.letras.mus.br › Infantil › Palavra Cantada › Gramática. Acesso em: 15 jan. 2018)

Assinale a afirmativa INCORRETA sobre as estrofes destacadas:

Alternativas
Comentários
  • b), porque não faz nenhum sentido a parte de política.

  • acho que faz sentido sim, a parte política... o que eu acho que seja o erro da B é a parte: "Não se verifica nela, porém, informação interpretável sobre as noções gramaticais de sujeito e objeto"; porque, há também a possibilidade de interpretar gramaticalmente a mudança de classe do sujeito que "se sujeita", como na voa passiva, por exemplo...

  • Que questão estranha. Fumarc e suas viagens...

  • O sujeito da voz passiva vira objeto na voz ativa.

     O animal   foi alimentado   pela criança. (voz passiva)

    A criança   alimentou   o animal. ( voz ativa)

    O animal se sujeitou, aceitou ser alimentado pela criança.


ID
2688871
Banca
FUMARC
Órgão
SEE-MG
Ano
2018
Provas
Disciplina
Português
Assuntos

Leia a letra da música “Gramática”, a respeito da qual serão feitas a questão. 


TEXTO XII


                                         Gramática

                                                                              Sandra Peres e Luiz Tatti


                                       O substantivo

                            É o substituto do conteúdo

 

                                          O adjetivo

                    É a nossa impressão sobre quase tudo


                                       O diminutivo

                               É o que aperta o mundo

                                      E deixa miúdo


                                       O imperativo

                   É o que aperta os outros e deixa mudo


                               Um homem de letras

                                   Dizendo ideias

                                Sempre se inflama


                              Um homem de ideias

                                  Nem usa letras

                                  Faz ideograma


                               Se altera as letras

                              E esconde o nome

                                   Faz anagrama


                           Mas se mostro o nome

                              Com poucas letras

                                É um telegrama


                               Nosso verbo ser

                              É uma identidade

                               Mas sem projeto


                               E se temos verbo

                                   Com objeto

                              É bem mais direto


                                No entanto falta

                                 Ter um sujeito

                                   Pra ter afeto


                              Mas se é um sujeito

                                  Que se sujeita

                                  Ainda é objeto


                                Todo barbarismo

                                   É o português

                                  Que se repeliu


                                  O neologismo

                                  É uma palavra

                                Que não se ouviu


                                    Já o idiotismo

                               É tudo que a língua

                                     Não traduziu


                                 Mas tem idiotismo

                                  Também na fala

                                    De um imbecil 


(Composição: Sandra Peres e Luiz Tatti Palavra Cantada. Álbum: Canções Curiosas, 1998. Disponível em: https://www.letras.mus.br › Infantil › Palavra Cantada › Gramática. Acesso em: 15 jan. 2018)

Uma das competências específicas do Ensino de Língua Portuguesa preconizadas pela BNCC a ser desenvolvida na Educação Básica foi transcrita abaixo:


4. Compreender o fenômeno da variação linguística, demonstrando atitude respeitosa diante de variedades linguísticas e rejeitando preconceitos linguísticos.

Disponível em: http://basenacionalcomum.mec.gov.br/images/BNCC_20dez_site.pdf. Acesso em: 19 fev. 2018


Um dos grandes desafios aos docentes na contemporaneidade é lidar (e ensinar a lidar) com a diversidade e pluralidade, em todas as suas manifestações – uma delas é a variabilidade linguística.


Analise a metalinguagem adotada pelos compositores (Sandra Peres e Luiz Tatti) e o que destacam os verbetes a seguir, no dicionário:


I - “Já o idiotismo / É tudo que a língua / Não traduziu”

II - “Mas tem idiotismo / Também na fala / De um imbecil”


Idiotismo

Substantivo masculino 1. m.q. IDIOTICE. 1.2 – ling. traço ou construção peculiar a uma determinada língua, que não se encontra na maioria dos outros idiomas (p.ex., o infinitivo pessoal do português, ou a resposta afirmativa com o próprio verbo da pergunta, como: -́Você vai? - Vou ); idiomatismo. - locução própria de uma língua, cuja tradução literal não faz sentido numa outra língua de estrutura análoga, ger. por ter um significado não dedutível da simples combinação dos significados dos elementos que a constituem (p.ex., [estar] com a cachorra '[estar] irado, de mau humor'); modismo.


III - “Todo barbarismo / É o português / Que se repeliu”


Barbarismo

substantivo masculino - 1.1 – estado ou condição de povo bárbaro; barbárie; 1.2. ato de grande crueldade; barbaria, barbaridade; 1.3. gram uso de formas vocabulares contrárias à norma culta da língua, seja do ponto de vista ortoépico (p.ex., rúbrica no de rubrica ), ortográfico (p.ex., excessão por exceção ), gramatical (p.ex., a construção quando eu ver por quando eu vir; menas palavras por ... menos palavras ), ou semântico (p.ex., o uso da loc. ir de encontro a ['chocar-se com'] no lugar de ir ao encontro de ['estar conforme']).


IV – “O neologismo / É uma palavra / Que não se ouviu”


Neologismo

Substantivo masculino. Ling. 1.1 emprego de palavras novas, derivadas ou formadas de outras já existentes, na mesma língua ou não; 1.2. atribuição de novos sentidos a palavras já existentes na língua; 1. 3. unidade léxica criada por esses processos.


Ainda que poeticamente, os autores espelham a existência de uma acepção negativa, preconceituosa, no que se refere:

Alternativas
Comentários
  • Resposta:B


ID
2688883
Banca
FUMARC
Órgão
SEE-MG
Ano
2018
Provas
Disciplina
Português
Assuntos

No documento “Orientações curriculares para o ensino médio: linguagem, código e suas tecnologias” (2006), aborda-se a piada a seguir, de forma a permitir uma reflexão sobre os diferentes fatores e conhecimentos implicados no processo de produção de sentido.


TEXTO XX


Chegando à fazenda dos avós, para visitá-los, o neto se dirige ao avô, que está na sala:

– Firme, vô?

– Não, fio, Sírvio Santos.

Disponível em: http://portal.mec.gov.br/seb/arquivos/pdf/book_volume_01_internet.pdf. Acesso em: 2 fev. 2018.


Assinale a alternativa que apresenta consideração INADEQUADA sobre a piada, tendo em vista o processo de compreensão.

Alternativas
Comentários
  •  Resposta: E

  • Questãozinha bizarra...

    16% de acertos..

    Ou seja, um chute aleatório tem mais chances de acertar essa questão do que usando a "lógica".

  • É... muito complicado... concordo que a "E" seja realmente inadequada. Agora falar que o avô foi retratado de forma não positiva é tão estranho. Como assim?????????


    Tem que FUMARc demais msm

  • Alguma dica de como resolver questões de Português da FUMARC? Banca muito estranha.

  • CA-RA-LIO

  • Se você acertou essa questão, volte a estudar.

  • Acertei no chutômetro

    Tentei analisar me enrolei toda.

    frustrante!!!!

  • Erraria cem vezes novamente... O avô não é retratado negativamente em lugar nenhum

  • Nada com nada!!!!

  • Esta bnaca E muito Estranha so por Deus!!

  • drogas pesadas na hr de elaborar a questao

  • acertei por eliminação, vi quais estavam certas e marquei a que estava errada. O avô retrata de forma não positiva, quando o neto pergunta se ele está firme, o avô responde Não, fio. quer dizer de forma negativa ( o avô não entende a pergunta do neto)

  • "levam o leitor entender que as formas “vô”, “Sírvio” e “fio” remetem a uma pronúncia característica de certas localidades do interior do país".

    Veja bem, essas palavras do elaborador extrapolam as informações do texto com opinião pessoal dele sobre o senso comum de que essas palavras são ditas no interior, mas essas informações não podem ser extraídas do texto.

  • "O avô é retratado no texto de forma não positiva"

    ???????????????????????

    O texto em nenhum momento coloca o vocábulo e a maneira de se expressar do avô como inferiores. Aliás, o objetivo é RIGOROSAMENTE o contrário.

    Questão absurda. Essa prova de português, com um monte de texto errado, fugindo da norma padrão, é vergonhosa.

    To desaprendendo.

  • "Os conhecimentos sobre variação dialetal são importantes para a compreensão da piada,"

    SIM, SÃO IMPORTANTÍSSIMOS.

    "os quais levam o leitor entender que as formas “vô”, “Sírvio” e “fio” remetem a uma pronúncia característica de certas localidades do interior do país."

    Perfeito. O texto inclusive inicia indicando que eles estão em UMA FAZENDA. Questão E está correta.

    Gabarito injustificável. Absurdo.

  • kara ! será que antes de imprimir a prova não existe outro professor para revisar as questões não ?????? É ridículo tantas questões ruins, subjetivas, sem nexo ... Com varias possibilidade de assertivas corretas..


ID
2688886
Banca
FUMARC
Órgão
SEE-MG
Ano
2018
Provas
Disciplina
Português
Assuntos

Chegando à fazenda dos avós, para visitá-los, o neto se dirige ao avô, que está na sala:
– Firme, vô?
– Não, fio, Sírvio Santos.
Disponível em: http://portal.mec.gov.br/seb/arquivos/pdf/book_volume_01_internet.pdf. Acesso em: 2 fev. 2018.

Pode-se dizer que a informação trazida pela oração adjetiva explicativa na piada é relevante para que o leitor signifique adequadamente:

Alternativas
Comentários
  •  Resposta: D- A resposta do avô.

  • (oração adjetiva explicativa )que está na sala: – Firme, vô? – Não, fio, Sírvio Santos.



    Se não tivesse essa oração avisando que está na sala,não iriamos identificar que o avô quis dizer que não estava assistindo FILME e sim "Sírvio Santos." até porque é de costume alguém assitir alguma coisa na sala.


    Gabarito D

  • Nao entendi o comentário do Diogo Cordeiro. Alguém ajuda ?

  • A questão centra-se na resposta do avô, que não entendeu a informalidade "giria" do neto, achou que o neto perguntou se ele estava assistindo ao "firme"

     

  • Pode ser qualquer m*rda.

    Eu to é desaprendendo com essas questões de professor de português.

  • Tentando explicar de outra forma:

    "Pode-se dizer que a informação trazida pela oração adjetiva explicativa na piada é relevante para que o leitor signifique adequadamente:"

    Informação trazida pela oração adjetiva explicativa: "...que está na sala"

    é relevante para que o leitor signifique: " a resposta do avô"


ID
2688889
Banca
FUMARC
Órgão
SEE-MG
Ano
2018
Provas
Disciplina
Português
Assuntos

Atente para as informações a seguir e responda à questão.


O texto XXII foi produzido por uma aluna do 7º ano do ensino fundamental II, como resposta à proposição de sua professora, exibida no texto XXI, abaixo.


TEXTO XXI


O texto a seguir apresenta apenas a introdução de um enredo narrativo, conforme estamos estudando em sala. Será sua tarefa continuar a história, com coerência, criando conflito, clímax e desfecho, de acordo com sua imaginação. Procure dar coerência à sua narrativa.

Naquela amanhã, acordei feliz. Era domingo, dia de clube, picolé, vôlei. Enfim, apenas alegria. Estranhei muito que meus pais não estivessem ainda de pé. Nem meu irmão. Fiquei apreensiva. Foi então que.....


TEXTO XXII


      Foi então que pensei: nossa, esta tarde demais. Porque será que ninguém acordou ainda? Teve ter acontecido algo muito ruim aqui em casa. Aí pus meu biquine e telefonei para a minha madrinha, e pidi pra ela me levá no clube com o carro dela, para jogar vole no domingo que era o dia de folga dela no hospital. Era gratuito tudo lá.

      Foi muito legal, nós tomamos sorvete e, jogamos bola com outras meninas, ninguém queria jogar vole comigo. Ela disse:

      – Que droga! O sorvete sujou meu cabelo todo.

      Então eu falei que não era culpa minha.

      Eu ri muito dela por que é super-engraçado quando alguém fica sujo sem querer.

      Nesse momento nós pegamos as sacolas e fomos para o vestuario do clube para tomar banho.

      O dia foi muito lindo. Eu adorei tudo lá. Quando voltamos vou contá tudo para meus pais. Eles gostaram muito. 

Considerada a proposta da professora, assinale a alternativa que traz consideração INADEQUADA sobre o texto da aluna:

Alternativas
Comentários
  • Resposta: C

  • Essa banca é a mais doid a q já vi..........não tem uma metodologia para fazer questão...como as outras...cada questão uma surpresa

  • A questão pede para marcar a alternativa inadequada.


    Então como o gabarito é a letra C a letra A está adequada?

  • Questão confusa e mal elaborada !

  • Gabarito para a BANCA LETRA C

     

    mas vejo duas respostas incorretas, uma vez que: a letra "A" diz que:

     

     a) No texto, não há nenhum problema que contrarie as inovações trazidas pelo Acordo Ortográfico de 1990.

     

    -------> No texto há a frase:  " Eu ri muito dela por que é super-engraçado quando alguém fica sujo sem querer."

     

    De acordo com o novo arcordo ortográfico, Usa-se hífen com palavras iniciadas com prefixo “super” se o termo posterior começar com H ou R.

    Ex: Super-homem, super-herói, super-habilidade, super-requintado, super-real, super-resistente, etc.

     

    e NÃO se usa hífen nos demais casos
    Ex: Supersalário, superbactéria, superfaturado, supermercado, superlotado, supermãe, superaquecido, etc.

     

    Por isso, entendo que cabe recurso e que a questão deveria ser anulada. 

    ---------------------------------------------------------------------------------------------------------------------------------------------------------------------------------------------

     

    O engraçado é que na mesma prova a própria banca reconhece o erro do hífen. Vejam:

     

    Q896295 - Português -  Redação - Reescritura de texto,  Interpretação de Textos -Ano: 2018- Banca: FUMARC- Órgão: SEE-MG

    Prova: Professor de Educação Básica - Língua Portuguesa

     

    Abaixo se apresentam problemas formais do texto que deveriam ser apontados pela professora, visando à sua reescrita, EXCETO:

     a) biquine. (é o gabarito)

     b) contá.

     c) super-engraçado. 

     d) teve.

     e) vestuario.

     

     

     

     

     

  • Prova pra professor de português... Alguém avisa a FUMARC que os professores do estado de MG não podem ser piores do que os seus examinadores...

  • Pessoal, atentar que a questão está falando do Acordo Ortográfico de 1990!! Prestem atenção!!

    Foi uma prova pra professor de Português

  • ☠️Danielle ☠️

    não há nenhum problema que contrarie as inovações trazidas pelo Acordo Ortográfico de 1990.

    Talvez no Acordo Ortográfico de 1990 usava hífen em super-engraçado. Mas, no Acordo de 2009, só se usa hífen em super + h ou r.

  • Engraçado que a banca se contradisse na mesma prova. Na questão , deu como incorreta a palavra super-engraçado.

    O Acordo Ortográfico da Língua Portuguesa (1990) entrou em vigor no início de 2009 no Brasil e em 13 de maio de 2009 em Portugal. Ou seja, não são dois acordos, mas apenas um.


ID
2688892
Banca
FUMARC
Órgão
SEE-MG
Ano
2018
Provas
Disciplina
Português
Assuntos

Atente para as informações a seguir e responda à questão.


O texto XXII foi produzido por uma aluna do 7º ano do ensino fundamental II, como resposta à proposição de sua professora, exibida no texto XXI, abaixo.


TEXTO XXI


O texto a seguir apresenta apenas a introdução de um enredo narrativo, conforme estamos estudando em sala. Será sua tarefa continuar a história, com coerência, criando conflito, clímax e desfecho, de acordo com sua imaginação. Procure dar coerência à sua narrativa.

Naquela amanhã, acordei feliz. Era domingo, dia de clube, picolé, vôlei. Enfim, apenas alegria. Estranhei muito que meus pais não estivessem ainda de pé. Nem meu irmão. Fiquei apreensiva. Foi então que.....


TEXTO XXII


      Foi então que pensei: nossa, esta tarde demais. Porque será que ninguém acordou ainda? Teve ter acontecido algo muito ruim aqui em casa. Aí pus meu biquine e telefonei para a minha madrinha, e pidi pra ela me levá no clube com o carro dela, para jogar vole no domingo que era o dia de folga dela no hospital. Era gratuito tudo lá.

      Foi muito legal, nós tomamos sorvete e, jogamos bola com outras meninas, ninguém queria jogar vole comigo. Ela disse:

      – Que droga! O sorvete sujou meu cabelo todo.

      Então eu falei que não era culpa minha.

      Eu ri muito dela por que é super-engraçado quando alguém fica sujo sem querer.

      Nesse momento nós pegamos as sacolas e fomos para o vestuario do clube para tomar banho.

      O dia foi muito lindo. Eu adorei tudo lá. Quando voltamos vou contá tudo para meus pais. Eles gostaram muito. 

Abaixo se apresentam problemas formais do texto que deveriam ser apontados pela professora, visando à sua reescrita, EXCETO:

Alternativas
Comentários
  • SUPERENGRAÇADO => Prefixos: hiper, inter, super: quando a palavra seguinte começar com h ou com r usará hifen:
    Ex. super-homem, inter-regional, hiper-real etc.

    Em todos os demais casos, é junto: hiperinflação, supersônico, internacional etc.

    VESTUÁRIO

    BIQUINE-BIQUÍNI => VOLP

    CONTA

    TEVÊ (televisão)

     

  • Ana, a correção quanto a alrernativa "d" se trata da do termo "deve" (Deve ter acontecido algo muito ruim aqui em casa.)

  • A C S C, a correção de "contá" não é CONTA, e sim, CONTAR!!


    Sobre biquine, isso tá certo??

  • Não entendi a questão. "biquine" está errado...deveria, também, ser apontado pela professora.

  • GAB:A

  • Sinceramente não dá pra entender essa banca, nunca escrevi BIQUINE e sim BIQUÍNI

  • Tb não entendi ... o certo seria superengraçado e biquíni?????
  • FONTE: PROFESSOR FABIANO SALES (TECCONCURSOS)

    Gabarito: LETRA A.

    A banca apontou a opção (A) como resposta. No entanto, a palavra "biquine" não é registrada nos dicionários. Os dicionaristas registram apenas a forma "biquíni", grafada com acento agudo e com a vogal final [i]. Portanto, a questão deveria ter sido anulada, o que lamentavelmente não foi feito pela banca examinadora.

     

    Nas demais opções:

    b) a forma verbal "contá" é marcada pela supressão da desinência de infinitivo "r"; deve, portanto, ser reescrita e substituída por "contar".

    c) o composto "super-engraçado" deve ser escrito sem hífen: superengraçado.

    d) a forma "teve" precisa ser substituída por "deve", formando a locução "Deve ter acontecido".

    e) a palavra "vestuário" é graficamente acentuada por ser paroxítona finalizada em ditongo crescente.

  • pelo o que tenho observado a FUMARC tem várias questões com mais de uma alternativa correta e, nesse caso não tem nenhuma.

    biquíni

    contar

    superengraçado

    deve

    vestuário

  • biquíni não é proparoxítona ? Via de regra, todas são acentuadas... não entendi essa questão


ID
2688895
Banca
FUMARC
Órgão
SEE-MG
Ano
2018
Provas
Disciplina
Português
Assuntos

Atente para as informações a seguir e responda à questão.


O texto XXII foi produzido por uma aluna do 7º ano do ensino fundamental II, como resposta à proposição de sua professora, exibida no texto XXI, abaixo.


TEXTO XXI


O texto a seguir apresenta apenas a introdução de um enredo narrativo, conforme estamos estudando em sala. Será sua tarefa continuar a história, com coerência, criando conflito, clímax e desfecho, de acordo com sua imaginação. Procure dar coerência à sua narrativa.

Naquela amanhã, acordei feliz. Era domingo, dia de clube, picolé, vôlei. Enfim, apenas alegria. Estranhei muito que meus pais não estivessem ainda de pé. Nem meu irmão. Fiquei apreensiva. Foi então que.....


TEXTO XXII


      Foi então que pensei: nossa, esta tarde demais. Porque será que ninguém acordou ainda? Teve ter acontecido algo muito ruim aqui em casa. Aí pus meu biquine e telefonei para a minha madrinha, e pidi pra ela me levá no clube com o carro dela, para jogar vole no domingo que era o dia de folga dela no hospital. Era gratuito tudo lá.

      Foi muito legal, nós tomamos sorvete e, jogamos bola com outras meninas, ninguém queria jogar vole comigo. Ela disse:

      – Que droga! O sorvete sujou meu cabelo todo.

      Então eu falei que não era culpa minha.

      Eu ri muito dela por que é super-engraçado quando alguém fica sujo sem querer.

      Nesse momento nós pegamos as sacolas e fomos para o vestuario do clube para tomar banho.

      O dia foi muito lindo. Eu adorei tudo lá. Quando voltamos vou contá tudo para meus pais. Eles gostaram muito. 

Assinale a alternativa que ilustra correção ADEQUADA de problemas de pontuação do texto, em uma possível reescrita pela aluna.

Alternativas
Comentários
  • para jogar vole no domingo, termo explicativo vem isolado por virgulas

    gab C

  • "Aí pus meu biquine e telefonei para a minha madrinha( )e pidi pra ela me levá no clube com o carro dela, para jogar vole no domingo( , ) que era o dia de folga dela no hospital."

     

    Na primeira vírgula, em vermelho, a qual foi retirada, a referida retirada se deveu ao fato de não caber vírgula antes de conjunção aditiva (notadamente E) e o caso não se enquadrar em nenhuma exceção (a saber: ênfase, sujeitos diferentes, valor adversativo, valor valor conclusivo e polissíndeto).

     

    Na segunda vírgula, em azul, a qual foi introduzida, a introdução se deveu ao fato de, com base no contexto, a expressão ser explicativa (todos os domingos a tia tem folga), não restritiva.

  • Nossa! Como é difícil corrigir pontuação em um texto com tantos erros.

  • Esses textos cheios de erros pra corrigir só a pontuação são osso viu, só pra professor mesmo kkk.

  • Que questão p0-rca! Não há gabrito, visto que a vírgula é obrigatória em casa polissíndeto(repetição de conectivo)

  • quem leva, leva algo em algum lugar, por isso penso que eles consideraram a letra C e não a letra D

  • Não é possível realizar a correção da pontuação em um texto com palavras inexistentes. A prova ser para professor não justifica. Não existe a palavra "pidi" na norma padrão da língua, de modo que não podemos identificá-la como verbo. A identificação do verbo é crucial para a pontuação.

    Ah não ser que o edital cobrasse adivinhação de novilíngua, a questão é absurda.

  • Questão difícil e confusa pra karai kkk. Nem sei como acertei kkk...


ID
2688898
Banca
FUMARC
Órgão
SEE-MG
Ano
2018
Provas
Disciplina
Português
Assuntos

Examine as seguintes passagens do texto XXII e as considerações que as seguem:


1. Porque será que ninguém acordou ainda? Teve ter acontecido algo muito ruim aqui em casa.

2. Eu ri muito dela por que é super-engraçado quando alguém fica sujo sem querer.

3. Foi muito legal, nós tomamos sorvete e, jogamos bola com outras meninas, ninguém queria jogar vole comigo.

4. Quando voltamos vou contá tudo para meus pais. Eles gostaram muito.


I. Em 3, a inserção de uma conjunção adversativa poderia ajudar o leitor na compreensão das relações entre suas orações.

II. Em 2, seria necessário que a aluna justificasse, no texto, a real razão do riso do narrador, pois isso é lacunar no texto.

III. Em 4, o tempo das formas verbais é fator determinante para o problema de incoerência existente nessa parte do texto.

IV. Em 1, manifesta-se um problema formal responsável pela primeira ocorrência de quebra de expectativa no texto.


São ADEQUADAS as considerações em:

Alternativas
Comentários
  • I. Em 3, a inserção de uma conjunção adversativa poderia ajudar o leitor na compreensão das relações entre suas orações.(verdadeira)

    III. Em 4, o tempo das formas verbais é fator determinante para o problema de incoerência existente nessa parte do texto.(verdadeira)

    GAB:B

  • Oi, eu sou o goku! 

    Gab. B

    Conjunção adversativa: Mas, porém, todavia, contudo, no entanto, entretanto.

    "Foi muito legal, nós tomamos sorvete e, jogamos bola com outras meninas, mas ninguém queria jogar vole comigo".

     

     

  • Alguém pode explicar melhor essa questão?Obrigado.

  • I. Em 3, a inserção de uma conjunção adversativa poderia ajudar o leitor na compreensão das relações entre suas orações. CERTO

    3. Foi muito legal, nós tomamos sorvete e, jogamos bola com outras meninas, MAS ninguém queria jogar vole comigo.

    II. Em 2, seria necessário que a aluna justificasse, no texto, a real razão do riso do narrador, pois isso é lacunar no texto. ERRADO, pois há a justificativa.

    2. Eu ri muito dela por que é super-engraçado quando alguém fica sujo sem querer.

    III. Em 4, o tempo das formas verbais é fator determinante para o problema de incoerência existente nessa parte do texto. CERTO

    4. Quando voltaRmos vou contAR tudo AOS meus pais. Eles gostarAO muito.

    IV. Em 1, manifesta-se um problema formal responsável pela primeira ocorrência de quebra de expectativa no texto.

    1. Porque será que ninguém acordou ainda? Teve ter acontecido algo muito ruim aqui em casa.

    Não sei explicar a IV, mas a alternativa deve ter as respostas I e III como certas e não deve ter a II.

    Logo chegamos ao gabarito letra B.

  • Na IV acontece um problema verbal... Teve no lugar de Deve fazendo assim a quebra da leitura

  • Eu sinto que deveria ter um texto aí, não?

  • IV - erro ortográfico e não formal Teve - Deve (creio ser esse o erro)


ID
2688901
Banca
FUMARC
Órgão
SEE-MG
Ano
2018
Provas
Disciplina
Português
Assuntos

TEXTO XXIII


Noivo é preso suspeito de matar mulher atropelada no casamento

17 de abril de 2011 • 19h33 • atualizado às 19h36


Um noivo foi preso na madrugada deste domingo em Santo André (SP) suspeito de atropelar uma mulher durante a sua própria festa de casamento. Segundo o 2º Distrito Policial da cidade, ocorriam duas festas de casamento no bairro Campestre, uma ao lado da outra, e, quando o noivo ia embora, por volta das 2h, ele atropelou a convidada da outra festa, que morreu.

      A polícia afirma que, antes do atropelamento, ocorreu uma briga entre convidados dos dois casamentos, com agressões físicas entre eles. O noivo, então, teria entrado em um carro, atropelado a vítima e fugido para sua casa, onde foi preso.

      O noivo foi indiciado por homicídio qualificado – devido a motivo fútil – e passou a noite de núpcias na cadeia.

Disponível em:  https://www.terra.com.br/noticias/brasil/policia/noivo-e-preso-suspeito-de-matar-mulher-atropelada-no-casamento,e4ba0970847ea310Vgn

CLD200000bbcceb0aRCRD.html. Acesso em: 19 fev. 2018.

Examine os sintagmas abaixo:


I. mulher atropelada no casamento.

II. mulher no casamento.

III. mulher atropelada.

IV. mulher.


Tendo em vista a organização sintática da manchete, mas sem levar em conta as informações trazidas pelo corpo da notícia, pode(m) ser tomado(s) como complemento do verbo matar apenas:

Alternativas
Comentários
  • Please!!! alguém me explica????? 

  • entendi nd

  • Armaria.....que banca e essa !!!! 

     

  • Gab.: D

    Realmente essa questão está bem confusa, pois o verbo matar é VTD, ou seja, pede complemento direto sem preposição, então a meu ver todas as opções estaria corretas, pois todas são complemento direto, e a expressão atropelada e atropelada no casamento é circunstância é uma invormação a mais que veio ali, a não ser que a banca quis que fosse uma reescritura sem mudança de sentido aí sim, mulher e mulher no casamento iriam alterar op sentido, mas não incorre em erro gramatical.

  • O nome dessa banca diz tudo: FUMARc, eh mta brisa!!!

  • Error 404 not found

  • No casamento também há morte rsrs


    resposta 1 e 3

  • ????????

  • Não sou de reclamar das bancas, até porque devemos resolver muitas questões para nos adaptarmos ao estilo da banca, mas a FUMARc nossa francamente... várias questões subjetivas, com duas alternativas corretas ou às vezes nenhuma correta. Fico me perguntando como ela consegue ganhar as licitações de concursos grandes como a PC de MG com tantas bancas boas no mercado... O pior é que ela não admite o erro, já vi questões claramentes equivocadas, mas que ela manteve o gabarito. Lamantável. Já passou da hora de haver uma lei que regulamente concurso Público. As bancas podem tudo e nós concurseiros só temos obrigações, mas direito zero. 

  • GABARITO D


    Pessoal, o que eu entendi ao fazer a questão: ela pede qual das alternativas dadas faz sentido de para complementar o verbo MATAR de acordo com a manchete. Vejamos a manchete: Noivo é preso suspeito de matar mulher atropelada no casamento. Reescrevam "Noivo é preso suspeito de matar (e aqui substituam as alternativas dadas na questão e vejam quais terão o mesmo sentido)"

    As alternativas dadas:

    I. mulher atropelada no casamento. Aqui faz sentido porque fica igual à manchete. Noivo mata mulher atropelada no casamento. Então já marcamos essa de cara.

    II. mulher no casamento. Aqui, se colocássemos "mata mulher no casamento", daria a entender que ele matou a noiva ou a esposa, sei la. Mas não teria o sentido real da manchete. Já descartamos essa.

    III. mulher atropelada. Aqui, ficaria correta, uma vez que o noivo matou uma mulher (qualquer) atropelada. Não fala qual mulher, como na manchete e ao mesmo tempo não da a entender ser a sua mulher, como no alternativa anterior.

    IV. mulher. Aqui seria a mesma coisa da alternativa II, daria a entender que o noivo matou sua mulher e não qualquer mulher.


    bons estudos

  • Gabarito: D)

    Pessoal, a questão pediu análise SINTÁTICA e não semântica. Realmente, o raciocínio da colega Débora Oliveira tem lógica, mas ao meu entender é baseado tão somente na análise semântica (sentido) do trecho.

    Eu consegui resolver a questão corretamente, analisando da seguinte forma:

    Trecho: "Noivo é preso suspeito de matar mulher atropelada no casamento".

    O enunciado da questão quer saber o que pode ser COMPLEMENTO VERBAL.

    ** MATAR (VTD - Verbo Transitivo Direto): quem mata mata alguém (OD - Objeto Direto, que seria o Complemento do verbo).

    Sendo assim, qual seria o Objeto Direto nesse trecho? R = "mulher atropelada no casamento" (tudo isso é o OD) e já responde parcialmente ao que foi pedido no enunciado da questão.

    Entretanto, se destrincharmos por partes, podemos analisar a sintaxe de cada termo dentro desse OD:

    - mulher (núcleo do objeto direto)

    - atropelada (morfologicamente é um adjetivo mas, sintaticamente, é um adjunto adnominal)

    - no casamento (sintaticamente, trata-se de adjunto adverbial de lugar. Aqui está o ponto crucial para responder a questão, visto que ADJUNTO ADVERBIAL NÃO É COMPLEMENTO!!)


    Portanto, somente "mulher atropelada" poderia ser considerado Complemento Verbal (qual seja, OD) do verbo MATAR, juntamente com "mulher atropelada no casamento".


    Corrijam-me se estiver equivocado em algum ponto :)

    Simbora!!

  • ????????????? ²

  • Danielle falou tudo.

    Faço questões há anos e nunca vi questões como essas da FUMARC, e o pior que o TJMMG terá ela como banca.

    Todas as demais parecem que mantêm um padrão, mas ela sai fora de todas, deus me livre.

  • Como dito no enunciado, analisemos UNICAMENTE a manchete sem levar em conta o contexto em que ele se insere.

    Noivo é preso suspeito de matar mulher atropelada no casamento.

    Fica claro que o sujeito de "matar" é "noivo".

    O verbo "matar" é transitivo direto.

    E o núcleo do objeto direto é "mulher". Isso posto, precisamos checar quais os elementos que estão subordinados a esse núcleo.

    O termo "atropelada" está subordinado a "mulher", pois este é o único com quem pode concordar.

    Dessa forma, "atropelada" necessariamente compõe o objeto direto.

    Já o o termo "no casamento" pode estar subordinado a "atropelada" ou a "matar".

    No primeiro caso, compõe o objeto direto "mulher atropelada no casamento".

    No segundo caso, atua como adjunto adverbial de "matar" e o objeto direto passa a ser "mulher atropelada".

    Dessa forma, os sintagmas que podem atuar como objeto direto são "mulher atropelada" (III) ou "mulher atropelada no casamento" (I).

    Resposta: Letra D

  • Qual o sentido dessa questão? Alguém sabe dizer?

    Danielle, concordo plenamente com você. A forma como as coisas são fere claramente a transparência. Deve sim ser aprovada uma lei que regule melhor e, se possível, determine que junto ao gabarito venha a resolução embasada da questão. Questões como essas são um desrespeito.

    Alan, entendi seu raciocínio; mas então, a opção IV - mulher também deveria ser resposta para a questão.

  • A IV-Mulher não poderia ser pois "atropelada" é adjunto adnominal de " mulher". Adjuntos Adnominais são termos internos de um outro termo maior, nesse caso o Objeto Direto.

    Se houver alguma informação errada peço que me corrijam.

  • Meu Deus!

  • Que questão horrível!

  • Deu bug total na mente agora. Já passou da hora dessa Fumarc sair de cena.


ID
2688904
Banca
FUMARC
Órgão
SEE-MG
Ano
2018
Provas
Disciplina
Português
Assuntos

TEXTO XXIII


Noivo é preso suspeito de matar mulher atropelada no casamento

17 de abril de 2011 • 19h33 • atualizado às 19h36


Um noivo foi preso na madrugada deste domingo em Santo André (SP) suspeito de atropelar uma mulher durante a sua própria festa de casamento. Segundo o 2º Distrito Policial da cidade, ocorriam duas festas de casamento no bairro Campestre, uma ao lado da outra, e, quando o noivo ia embora, por volta das 2h, ele atropelou a convidada da outra festa, que morreu.

      A polícia afirma que, antes do atropelamento, ocorreu uma briga entre convidados dos dois casamentos, com agressões físicas entre eles. O noivo, então, teria entrado em um carro, atropelado a vítima e fugido para sua casa, onde foi preso.

      O noivo foi indiciado por homicídio qualificado – devido a motivo fútil – e passou a noite de núpcias na cadeia.

Disponível em:  https://www.terra.com.br/noticias/brasil/policia/noivo-e-preso-suspeito-de-matar-mulher-atropelada-no-casamento,e4ba0970847ea310Vgn

CLD200000bbcceb0aRCRD.html. Acesso em: 19 fev. 2018.

Abaixo, apresentam-se termos e orações, sublinhados na notícia, os quais exercem a mesma função sintática, EXCETO:

Alternativas
Comentários
  • B - motivo fútil - complemento...

    o resto é sujeito

  • HÁ DOIS GABARITOS

     b) motivo fútil. complemento

    e) uma briga entre convidados dos dois casamentos.   A polícia afirma que, antes do atropelamento, ocorreu uma briga entre convidados dos dois casamentos

    Polícia = SUJE

    Afirma= VTD

    que ocorreu uma briga entre convidados dos dois = OI

  • Concurseiro Federal, a questão sublinha o QUE (que morreu) e não é esse que vc citou. Sugiro que releia. 

  • Entendi foi nada...

  • Olá, pessoal!

    Para fazer esse tipo de questão, aconselho colocar os termos na ordem direta. Assim, será mais fácil descobrir sua função sintática. Fiquem também atentos às funções sintáticas exercidas pelo relativo QUE. Este conhecimento é essencial para a resolução de questões desse tipo.

    a) (...)duas festas de casamento ocorriam - SUJEITO

    b) (...) devido a motivo fútil - NÚCLEO DO ADJUNTO ADVERBIAL DE CAUSA - RESPOSTA CORRETA!

    c) (...) o noivo ia - SUJEITO

    d) (...) que morreu - QUE é pronome relativo. Todo pronome relativo exerce uma função sintática na oração de que participa. Para reconhecê-la, basta substituir o QUE pela palavra retomada no texto. Assim, ficará muito mais fácil identificar a função desse QUE na oração em que está inserido. No presente caso, QUE retoma "convidada". Então, "A CONVIDADA morreu". Se a convidada exerce papel de sujeito, logo o QUE também fará o papel de sujeito.

    e) (...) uma briga entre convidados dos dois casamentos ocorreu. SUJEITO


    Obs.: Cuidado! "motivo fútil" não é complemento verbal ou nominal. Isso porque "devido a" é uma locução prepositiva que incia o adjunto adverbial de causa "devido a motivo fútil". Assim, não há que se falar em complemento, mas sim em núcleo do adjunto. Caso tenham dúvidas em relação à classificação de "devido a", consultem o Dicionário Houaiss. ;=)


    QUER MAIS DICAS DE PORTUGUÊS? MANDE-ME UM OI PELO WHATSAPP (31) 9 84974272 E ME SIGA PELO INSTAGRAM @CRISORZIL. BONS ESTUDOS!

  • Gabarito letra B

     

    Sempre, antes de qualquer análise, pergunte pelo sujeito. Facilita também se você colocar a frase na ordem direta (SUJEITO, VERBO E COMPLEMENTO). Todas são sujeitos, exceto a B que é adjunto adverbial de causa. 

     

    Abaixo, apresentam-se termos e orações, sublinhados na notícia, os quais exercem a mesma função sintática, EXCETO:

     

      a) duas festas de casamento. Pergunte pelo sujeito: o que ocorria? resposta é o sujeito:duas festas

     

    b) motivo fútil. adjunto adverbial de causa, facilita se vc inserir na frase : por CAUSA do motivo fútil o noivo foi indiciado. 

     

    c) o noivo. Quem ia embora? resposta é o sujeito, o noivo

     

    d) que - essa era a mais complicada e que poderia gerar dúvidas - Ele atropelou a convidade de outra festa, que morreu. Quem morreu? A convidada, sendo assim o que é pronome relativo, está no lugar do sujeito, exercendo assim, sua função. 

     

    e) uma briga entre convidados dos dois casamentos. o que ocorreu? sujeito : uma briga entre convidados dos dois casamentos

  • QUE MORREU

    QUE SUJEITO

  • O termo "duas festas de casamento" exerce a função de sujeito da forma verbal "ocorriam".

    O termo "o noivo" é sujeito do predicado "ia embora".

    O pronome relativo "que", que retoma "convidada", atua como sujeito de "morreu".

    O termo "uma briga entre convidados dos dois casamentos" exerce função de sujeito da forma verbal "ocorria".

    Já o termo "a motivo fútil" atua como complemento do nome "devido".

    Resposta: Letra B

  • Toda pergunta a baixo a resposta "=" é o sujeito blz? ok.

    o que ocorria ? = duas festa

    quem ia embora ? = o noivo

    quem morreu ? = a convidada

    que (m) foi indiciado ? = o noivo (repare que aqui o sujeito é "o noivo", o motivo fútil é um "adjunto adverbial de causa")

    Pegou ? Estimo que sim.

    Bons estudos !


ID
2688907
Banca
FUMARC
Órgão
SEE-MG
Ano
2018
Provas
Disciplina
Português
Assuntos

Leia a seguir uma das habilidades previstas pela Base Nacional Curricular Comum (BNCC) para o 8º e o 9º anos e responda à questão.


TEXTO XXIV


Analisar a modalização realizada em textos noticiosos e argumentativos, por meio das modalidades apreciativas, viabilizadas por classes e estruturas gramaticais como adjetivos, locuções adjetivas, advérbios, locuções adverbiais, orações adjetivas e adverbiais, orações relativas restritivas e explicativas etc., de maneira a perceber a apreciação ideológica sobre os fatos noticiados ou as posições implícitas ou assumidas.

Disponível em: http://basenacionalcomum.mec.gov.br/images/BNCC_20dez_site.pdf. Acesso em: 19 fev. 2018

Considerando a habilidade descrita pela BNCC, o estudante deve ser levado a perceber que a manchete publicada pelo jornal O Globo, em 29/1/2009 – “Lula amplia Bolsa Família um dia após cortar orçamento” – traz o ponto de vista do jornal sobre o fato noticiado. Esse ponto de vista se evidencia:


I. pelo uso do verbo “ampliar”, no lugar de “aumentar”, que seria mais neutro;

II. pelo uso da forma “Lula”, no lugar de “presidente do Brasil”, que indicaria respeito;

III. pela inserção de informação temporal, que não é central à natureza do fato relatado.


Tendo em vista o exposto acima:

Alternativas
Comentários
  • Apenas o item III estar correto.

     

  • " um dia após " .....item III

  • Ampliar e aumentar são sinônimos, a troca de um termo por outro não torna o texto mais neutro.


    Eliminando o item I restam apenas as letras C e D.


    Gabarito D

  • Claro que a questão temporal é central no fato narrado se levarmos em consideração a intenção do texto !! discordo totalmente do Gabarito

  • A FUMARC é medonha!!! Meu Deus!! brinca com o concurseiro.

    Claro que a informação é central a natureza do relato.

    Item III ERRADO.

  •  “Lula amplia Bolsa Família um dia após cortar orçamento”

    I. pelo uso do verbo “ampliar”, no lugar de “aumentar”, que seria mais neutro; errado

    Ampliar quer dizer a adesão de mais pessoas. Aumentar seria aumentar o valor do benefício já existente.

    No contexto, aumentar não é mais neutro nem sinônimo de ampliar.

    II. pelo uso da forma “Lula”, no lugar de “presidente do Brasil”, que indicaria respeito; errada

    Lula é o nome social que o próprio Ex-Presidente se identifica. Não é mais ou menos respeitoso o chamar de Lula.

    III. pela inserção de informação temporal, que não é central à natureza do fato relatado. certa

    "um dia após" A notícia quis dar ênfase ao paradoxo dos acontecimentos através do caráter temporal.

  • Não é central. A informação é "Lula amplia Bolsa Família"

    A "apreciação ideológica sobre os fatos noticiados ou as posições implícitas ou assumidas" vem exatamente com a informação temporal. O repórter quis informar o fato mas colocá-lo em oposição a um acontecimento independente.

    "O técnico do Flamengo barra Gabigol um dia após o centroavante criticar Bolsonaro nas redes"

    É clara a intenção aqui em colocar duas coisas independentes como causa e consequência uma da outra através de uma relação temporal.

    Capiche?


ID
2688910
Banca
FUMARC
Órgão
SEE-MG
Ano
2018
Provas
Disciplina
Português
Assuntos

Abaixo, apresentam-se ocorrências do português que, embora frequentes na língua, não são, de modo geral, abonadas pelas nossas gramáticas normativas. Examine-as e, em seguida, leia as considerações apresentadas sobre elas.


I. Ele queixou com a professora.

II. Esta casa bate muito sol; valerá a pena comprá-la.

III. Felizmente, estudei todos os meus filhos.

IV. Na minha gestão, construiu-se pontes e viadutos modernos.


Assinale a alternativa que traz consideração INADEQUADA.

Alternativas
Comentários
  • Banca Cabulosa!!!!!!!!

  • Acredito que a alternativa C seja a correta porque o item IV a partícula SE é apassivadora e a partícula SE do item C é indice de indeterminação.

  • Marquei a letra C por ter comparado a sentença "Trata-se de assuntos nevrálgicos", que está correta, com o item IV: "Na minha gestão, construiu-se pontes e viadutos modernos", que está errada (Construíram-se - devido à partícula apassivadora), logo, não se trata de um exemplo como o que a questão diz:


    "Meu pai responsabilizou pelo assunto”, “O carro furou pneu” e “Trata-se de assuntos nevrálgicos” são dois exemplos que materializam fenômenos trazidos, respectivamente, em I, II e IV.

  • Marquei a letra C por ter comparado a sentença "Trata-se de assuntos nevrálgicos", que está correta, com o item IV: "Na minha gestão, construiu-se pontes e viadutos modernos", que está errada (Construíram-se - devido à partícula apassivadora), logo, não se trata de um exemplo como o que a questão diz:


    "Meu pai responsabilizou pelo assunto”, “O carro furou pneu” e “Trata-se de assuntos nevrálgicos” são dois exemplos que materializam fenômenos trazidos, respectivamente, em I, II e IV.

  • VI) o verbo era para estar flexionado..pois na voz passiva...o que parece objeto é sujeito !

  • NÃO GOSTEI.

  • FUMARC faz uma questões muito esquisitas...

  • A gente gasta muito tempo tentando entender o que eles estão dizendo...

  • O ruim de questões como esta é que a gente gasta tempo tentando resolver, erra e não aprende nada.

    Vamos para a próxima, até passar.

  • Fumarc consegue ser mais lixo a cada prova

  • No dia em que "esta casa" for sujeito do verbo "bater" em "Esta casa bate muito sol", como afirmado pela banca na alternativa B, eu largo meus estudos e começo a vender balas no sinal.

  • Essa foi uma das mais complexas que já vi mesmo, mas é para professor também, relevemos.

    Deus, ajudai-nos no TJMMG kkkk

  • GAB:C

  • Essa questão foi difícil, mas não impossível de resolver. O que eu temo na Fumarc são aquelas questões de interpretação impossíveis, com gabarito ambíguo.

  • Meus Deus, a alternativa E não faz absolutamente sentido nenhum.

  • Lucas Ivan meu raciocínio foi o mesmo que o seu.

  • Meu Deus que enigma é esse? to sem entende r até agora